Exam 2

Pataasin ang iyong marka sa homework at exams ngayon gamit ang Quizwiz!

A (Children with asthma usually have these chronic symptoms. Pneumonia appears with an acute onset and fever and general malaise. Bronchiolitis is an acute condition caused by respiratory syncytial virus. Foreign body in the trachea will manifest with acute respiratory distress or failure and maybe stridor.)

A child has a chronic, nonproductive cough and diffuse wheezing during the expiratory phase of respiration. This suggests: a. Asthma. b. Pneumonia. c. Bronchiolitis. d. Foreign body in the trachea.

A (Children often undergo separation anxiety when they are separated from their parents. This separation anxiety manifests in different stages such as protest, despair, and detachment. Protest is the first stage of separation anxiety, during which the child screams, cries, or hits the other person for separating him or her from the parents. After this stage, the child enters the stage of despair, where the child begins to withdraw from others and stay depressed. During this stage, the child starts wetting the bed and sucking the thumb because of fear and anxiety. After the stage of despair, the child enters the stage of detachment. The child starts interacting with strangers and takes an increased interest in the surroundings or sits in a corner and plays with a toy.)

A child has recently been admitted to the hospital. The child's parents have not yet arrived at the hospital. What behavior is the child exhibiting that leads the nurse to believe the child is exhibiting the stage of protest? The child: A. Screams and hits the nurse. B. Is withdrawn from others. C. Has the habit of bed-wetting. D. Sits in a corner with a toy.

C (Visual acuity in one eye despite appropriate optical correction is amblyopia. Myopia is nearsightedness, which is the ability to see objects up close but not clearly at a distance. Hyperopia is farsightedness, which is the ability to see distant objects clearly but not those up close. Astigmatism is an alteration in vision caused by unequal curvature in the refractive apparatus of the eye.)

A child in the clinic exhibits reduced visual acuity in one eye despite appropriate optical correction. The nurse expects the child's health care provider to diagnosis the child with: A. myopia. B. hyperopia. C. amblyopia. D. astigmatism.

A, C, E (a. Discuss dietary restrictions. c. Send a pain scale home with the family. e. Discuss complications that may occur.)

A child is being discharged from an ambulatory care center after an inguinal hernia repair. Which discharge interventions should the nurse implement (select all that apply)? a. Discuss dietary restrictions. b. Hold any analgesic medications until the child is home. c. Send a pain scale home with the family. d. Suggest the parents fill the prescriptions on the way home. e. Discuss complications that may occur.

D (Hospitalized children undergo depression when they are separated from their parents. As they go through the stages of separation anxiety, children eventually detach from their parents and develop new and shallow relationships. Children interact with others and develop new relationships as a result of resignation, not contentment. Children who are detached begin to show increased interest in their surroundings. They are also not developing their social interaction skills. Children try not to think about the separation; hence, they start developing new interactions.)

A child is hospitalized for a chronic illness. Initially, the child showed symptoms of depression but later started interacting with others. What does the nurse infer from the patient's behavior? The child is: A. Content with the care provided. B. Showing improved social skills. C. Getting used to the surroundings. D. Detached from both parents.

C (Children who undergo orofacial surgery should not use a straw for drinking fluids because it can damage the surgical site. Therefore the parents should not use cup with a lid and a straw for giving fluids to the child. The parents should bring a blanket and pillow for the child in the car so that the child can sit or sleep properly. Parents should bring a plastic bag, which will be helpful if the child becomes nauseated or vomits. The parents should give prescribed pain medication to the child before leaving the facility for relieving pain.)

A child who has undergone orofacial surgery is getting discharged. The nurse teaches the parents about how to safely transport the child on the way home. Which statement made by the parents indicates a need for additional teaching? "We should: A. Have a blanket and pillow for our child for the car ride home." B. Have a plastic bag for our child in case of nausea and vomiting." C. Use a cup with a lid and a straw for giving fluids to our child." D. Make sure our child has pain medication before discharge."

B (Child's routine habits and preferences are maintained.)

A child with autism is hospitalized with asthma. The nurse should plan care so that the: a. Parents' expectations are met. b. Child's routine habits and preferences are maintained. c. Child is supported through the autistic crisis. d. Parents need not be at the hospital.

A (Maintain a structured routine and keep stimulation to a minimum.)

A child with autism spectrum disorder (ASD) is admitted to the hospital with pneumonia. The nurse should plan which priority intervention when caring for the child? a. Maintain a structured routine and keep stimulation to a minimum. b. Place the child in a room with a roommate of the same age. c. Maintain frequent touch and eye contact with the child. d. Take the child frequently to the playroom to play with other children.

A (Bronchodilators should be given before CPT to open bronchi and make expectoration easier. Aerosolized bronchodilator medications are not helpful when used after CPT. Oxygen administration is necessary only in acute episodes with caution because of chronic carbon dioxide retention.)

A child with cystic fibrosis (CF) receives aerosolized bronchodilator medication. When should this medication be administered? a. Before chest physiotherapy (CPT) b. After CPT c. Before receiving 100% oxygen d. After receiving 100% oxygen

A (Popcorn is a high-fiber food. Pancakes and muffins do not have significant fiber unless made with fruit or bran. Raw fruits, especially those with skins and seeds, other than ripe bananas and avocados are high in fiber.)

A high-fiber food that the nurse could recommend for a child with chronic constipation is: a. Popcorn. b. Muffins. c. Pancakes. d. Ripe bananas.

D (The size of the droplets is too large to liquefy secretions. No additional oxygen is provided with humidified air. The humidity has no effect on ventilation. By humidifying the inspired air, the membranes inflamed by the infection and dry air are soothed.)

A humidified atmosphere is recommended for a young child with an upper respiratory tract infection because this environment facilitates: A. liquefying secretions. B. improving oxygenation. C. promoting ventilation. D. soothing inflamed mucous membrane.

A (Regression is seen during hospitalization.)

A previously "potty-trained" 30-month-old child has reverted to wearing diapers while hospitalized. The nurse should reassure the parents that this is normal because: a. Regression is seen during hospitalization. b. Developmental delays occur because of the hospitalization. c. The child is experiencing urinary urgency because of hospitalization. d. The child was too young to be "potty-trained."

B (Changing furniture positions can result in accidents, so this must be avoided. A small stool or a desk should be placed near the bed to support the child so that he or she can climb into bed easily. The child is a patient in the hospital and should not be asked to work while there. Educating the child about the procedures that will be carried out for the treatment will help the child understand and mentally prepare for them.)

A visually impaired child is hospitalized for eye surgery. What nursing intervention should be included in the plan of care to encourage the child to be independent? The nurse: A. Does not keep a stool or small desk near the bed. B. Instructs the cleaner not to move the furniture around. C. Gives the child work to do while the child is in the hospital. D. Does not educate the child about the treatment procedures.

C (Moderately cognitively impaired but trainable.)

A young child who has an intelligence quotient (IQ) of 45 would be described as: a. Within the lower limits of the range of normal intelligence. b. Mildly cognitively impaired but educable. c. Moderately cognitively impaired but trainable. d. Severely cognitively impaired and completely dependent on others for care.

3. In the fourth stage of labor, a full bladder increases the risk for A. Hemorrhage B. Dissesminated intravascular coagulation C. Infection D. Shock

A. Hemorrhage A full bladder displaces the uterus and prevents contraction of the uterus and uterine atony is the primary cause of postpartum hemorrhage. Shock, infection and DIC are not related to bladder distention

An appropriate nursing intervention when providing comfort and support for a child whose death is imminent is to: a. Limit care to essentials. b. Avoid playing music near the child. c. Explain to the child the need for constant measurement of vital signs. d. Whisper to the child instead of using a normal voice.

ANS: A When death is imminent, care should be limited to interventions for palliative care. Music may be used to provide comfort for the child. Vital signs do not need to be measured frequently. The nurse should speak to the child in a clear, distinct voice.

26. The nerve block used in labor that provides anesthesia to the lower vagina and perineum is called: a. an epidural. b. a pudendal. c. a local. d. a spinal block.

ANS: B A pudendal block anesthetizes the lower vagina and perineum to provide anesthesia for an episiotomy and use of low forceps if needed. An epidural provides anesthesia for the uterus, perineum, and legs. A local provides anesthesia for the perineum at the site of the episiotomy. A spinal block provides anesthesia for the uterus, perineum, and down the legs.

The nurse comes into the room of a child who was just diagnosed with a chronic disability. The child's parents begin to yell at the nurse about a variety of concerns. The nurse's best response is: a. "What is really wrong?" b. "Being angry is only natural." c. "Yelling at me will not change things." d. "I will come back when you settle down."

ANS: B Parental anger after the diagnosis of a child with a chronic disability is a common response. One of the most common targets for parental anger is members of the staff. The nurse should recognize the common response of anger to the diagnosis and allow the family to ventilate. "What is really wrong?" "Yelling at me will not change things," and "I will come back when you settle down" are all possible responses, but they are not the likely reasons for this anger.

25. A child is receiving total parenteral nutrition (TPN; hyperalimentation). At the end of 8 hours, the nurse observes the solution and notes that 200 mL/8 hr is being infused rather than the ordered amount of 300 mL/8 hr. The nurse should adjust the rate so that how much will infuse during the next 8 hours? a. 200 mL b. 300 mL c. 350 mL d. 400 mL

ANS: B The TPN infusion rate should not be increased or decreased without the practitioner being informed because alterations in rate can cause hyperglycemia or hypoglycemia. Knowing this will result in the infusion rate being set to the original prescribed flow rate.

19. Guidelines for intramuscular administration of medication in school-age children include what instruction? a. Inject medication as rapidly as possible. b. Insert the needle quickly, using a dart-like motion. c. Penetrate the skin immediately after cleansing the site, before skin has dried. d. Have the child stand, if possible, and if he or she is cooperative.

ANS: B The needle should be inserted quickly in a dart-like motion at a 90-degree angle unless contraindicated. Inject medications slowly. Allow skin preparation to dry completely before skin is penetrated. Place the child in a lying or sitting position.

27. What nursing action is appropriate for specimen collection? a. Follow sterile technique for specimen collection. b. Sterile gloves are worn if the nurse plans to touch the specimen. c. Use Standard Precautions when handling body fluids. d. Avoid wearing gloves in front of the child and family.

ANS: C Standard Precautions should always be used when handling body fluids. Specimen collection is not always a sterile procedure. Gloves should be worn if there is a chance the nurse will be contaminated. The choice of sterile or clean gloves will vary according to the procedure or specimen. The child and family should be educated in the purpose of glove use, including the fact that gloves are used with every patient, so that they will not be offended or frightened.

24. What intervention should the nurse implement when suctioning a child with a tracheostomy? a. Encouraging the child to cough to raise the secretions before suctioning. b. Selecting a catheter with a diameter three-fourths as large as the diameter of the tracheostomy tube. c. Ensuring that each pass of the suction catheter take no longer than 10 seconds. d. Allowing the child to rest after every 5 times the suction catheter is passed.

ANS: C Suctioning should require no longer than 10 seconds per pass. Otherwise the airway may be occluded for too long. If the child is able to cough up secretions, suctioning may not be indicated. The catheter should have a diameter one-half the size of the tracheostomy tube. If it is too large, it might block the child's airway. The child is allowed to rest for 30 to 60 seconds after each aspiration to allow oxygen tension to return to normal. Then the process is repeated until the trachea is clear.

Chris, age 9 years, has several physical disabilities. His father explains to the nurse that his son concentrates on what he can rather than cannot do and is as independent as possible. The nurse's best interpretation of this is: a. The father is experiencing denial. b. The father is expressing his own views. c. Chris is using an adaptive coping style. d. Chris is using a maladaptive coping style.

ANS: C The father is describing a well-adapted child who has learned to accept physical limitations. These children function well at home, at school, and with peers. They have an understanding of their disorder that allows them to accept their limitations, assume responsibility for care, and assist in treatment and rehabilitation. The father is not denying the child's limitations or expressing his own views. This is descriptive of an adaptive coping style.

At what developmental period do children have the most difficulty coping with death, particularly if it is their own? a. Toddlerhood c. School-age b. Preschool d. Adolescence

ANS: D Because of their mature understanding of death, remnants of guilt and shame, and issues with deviations from normal, adolescents have the most difficulty coping with death. Toddlers and preschoolers are too young to have difficulty coping with their own death. They will fear separation from parents. School-age children will fear the unknown, such as the consequences of the illness and the threat to their sense of security.

32. What procedure is recommended to facilitate a heelstick on an ill neonate to obtain a blood sample? a. Apply cool, moist compresses. b. Apply a tourniquet to the ankle. c. Elevate the foot for 5 minutes. d. Wrap foot in a warm washcloth.

ANS: D Before the blood sample is taken, the heel is heated with warm moist compresses for 5 to 10 minutes to dilate the blood vessels in the area. Cooling causes vasoconstriction, making blood collection more difficult. A tourniquet is used to constrict superficial veins. It will have an insignificant effect on capillaries. Elevating the foot will decrease the blood in the foot available for collection.

Most parents of children with special needs tend to experience chronic sorrow. This is characterized by: a. Lack of acceptance of the child's limitation. b. Lack of available support to prevent sorrow. c. Periods of intensified sorrow when experiencing anger and guilt. d. Periods of intensified sorrow and loss that occur in waves over time.

ANS: D Chronic sorrow is manifested by feelings of sorrow and loss that recur in waves over time. The sorrow is in response to the recognition of the child's limitations. The family should be assessed in an ongoing manner to provide appropriate support as the needs of the family change. The sorrow is not preventable. The chronic sorrow occurs during the reintegration and acknowledgment stage.

16. In the current practice of childbirth preparation, emphasis is placed on: a. the Dick-Read (natural) childbirth method. b. the Lamaze (psychoprophylactic) method. c. the Bradley (husband-coached) method. d. having expectant parents attend childbirth preparation in any or no specific method.

ANS: D Encouraging expectant parents to attend childbirth preparation class is most important because preparation increases a woman's confidence and thus her ability to cope with labor and birth. Although still popular, the "method" format of classes is being replaced with other offerings such as Hypnobirthing and Birthing from Within.

34. Which is the preferred site for intramuscular injections in infants? a. Deltoid b. Dorsogluteal c. Rectus femoris d. Vastus lateralis

ANS: D The preferred site for infants is the vastus lateralis. The deltoid and dorsogluteal sites are used for older children and adults. The rectus femoris is not a recommended site.

A preschooler is found digging up a pet bird that was recently buried after it died. The best explanation for this behavior is that: a. He has a morbid preoccupation with death. b. He is looking to see if a ghost took it away. c. The loss is not yet resolved, and professional counseling is needed. d. Reassurance is needed that the pet has not gone somewhere else.

ANS: D The preschooler can recognize that the pet has died but has difficulties with the permanence. Digging up the bird gives reassurance that the bird is still present. A morbid preoccupation with death and the child looking to see if a ghost took it away are expected responses. If they persist, intervention may be required.

B (Identity)

According to Erikson, the psychosocial task of adolescence is developing: a. Intimacy. b. Identity. c. Initiative. d. Independence.

D (Erosion of tooth enamel)

An adolescent teen has bulimia. Which assessment finding should the nurse expect? a. Diarrhea b. Amenorrhea c. Cold intolerance d. Erosion of tooth enamel

A, D (Antibiotics are indicated for a bacterial pneumonia. Often the child will have decreased pulmonary reserve, and the clustering of care is essential. Round-the-clock antitussive agents and strict intake and output are not included in the care of the child with pneumonia. Mist tents are no longer utilized for pediatric respiratory care.)

An infant has developed staphylococcal pneumonia. Nursing care of the child with pneumonia includes which of the following? (Select all that apply). a. Cluster care to conserve energy b. Round-the-clock administration of antitussive agents c. Strict intake and output to avoid congestive heart failure d. Administration of antibiotics e. Placement in a mist tent

D (The etiology of anorexia remains unclear, but a distinct psychologic component is present. The diagnosis is based primarily on psychologic and behavioral criteria. Anorexia nervosa is observed more commonly in adolescent girls and young women. It does not occur most frequently in adolescents from a lower socioeconomic group. In reality, anorexic adolescents are often from families of means who have high parental expectations for achievement. Anorexia is a psychiatric disorder.)

Anorexia nervosa may best be described as: a. Occurring most frequently in adolescent males. b. Occurring most frequently in adolescents from lower socioeconomic groups. c. Resulting from a posterior pituitary disorder. d. Resulting in severe weight loss in the absence of obvious physical causes.

B (Provide peer experiences such as Special Olympics when older.)

Appropriate interventions to facilitate socialization of the cognitively impaired child include to: a. Provide age-appropriate toys and play activities. b. Provide peer experiences such as Special Olympics when older. c. Avoid exposure to strangers who may not understand cognitive development. d. Emphasize mastery of physical skills because they are delayed more often than verbal skills.

B (Viral illnesses cause inflammation that causes increased airway reactivity in asthma. Medications such as aspirin, nonsteroidal antiinflammatory drugs, and antibiotics may aggravate asthma, but not frequently in infants. Exposure to cold air may exacerbate already existing asthma. Allergy is associated with asthma, but 20% to 40% of children with asthma have no evidence of allergic disease.)

Asthma in infants is usually triggered by: a. Medications. b. A viral infection. c. Exposure to cold air. d. Allergy to dust or dust mites.

A, C, D (The peak expiratory flow rate is one of the diagnostic criteria for classifying severity. The frequency of symptoms is one of the diagnostic criteria for classifying severity. The frequency and severity of exacerbations are two of the diagnostic criteria for classifying severity. The clinical features that distinguish the categories of asthma do not include other allergies.)

Asthma is classified into four categories: mild intermittent, mild persistent, moderate persistent, and severe persistent. Clinical features used to determine these categories include: (Select all that apply.) A. Lung function. B. Associated allergies. C. Frequency of symptoms. D. Frequency and severity of exacerbations.

A primary nursing responsibility when caring for a woman experiencing an obstetric hemorrhage associated with uterine atony is to: A. Establish venous access B. Perform fundal massage C. Prepare the woman for surgical intervention D. Catheterize the bladder

B. Perform fundal massage The initial management of excessive postpartum bleeding is firm massage of the uterine fundus.

A nulliparous woman who has just begun the second stage of her labor would most likely: a. Experience a strong urge to bear down. b. Show perineal bulging. c. Feel tired yet relieved that the worst is over. d. Show an increase in bright red bloody show.

C (Common maternal behaviors during the latent phase of the second stage of labor include feeling a sense of accomplishment and optimism because "the worst is over." During the latent phase of the second stage of labor, the urge to bear down often is absent or only slight during the acme of contractions. Perineal bulging occurs during the transition phase of the second stage of labor, not at the beginning of the second stage. An increase in bright red bloody show occurs during the descent phase of the second stage of labor.)

At 1 minute after birth, the nurse assesses the newborn to assign an Apgar score. The apical heart rate is 110 bpm, and the infant is crying vigorously with the limbs flexed. The infant's trunk is pink, but the hands and feet are blue. What is the correct Apgar score for this infant? a. 7 b. 8 c. 9 d. 10

C (The Apgar score is 9 because 1 point is deducted from the total score of 10 for the infant's blue hands and feet. The baby received 2 points for each of the categories except color. Because the infant's hands and feet were blue, this category is given a grade of 1.)

D (A sweat chloride test result greater than 60 mEq/L is diagnostic of CF. Although bronchoscopy is helpful for identifying bacterial infection in children with CF, it is not diagnostic. Serum calcium is normal in children with CF. Urine creatinine is not diagnostic of CF.)

Cystic fibrosis (CF) is suspected in a toddler. Which test is essential in establishing this diagnosis? a. Bronchoscopy b. Serum calcium c. Urine creatinine d. Sweat chloride test

B (Sensorineural)

Distortion of sound and problems in discrimination are characteristic of which type of hearing loss? a. Conductive b. Sensorineural c. Mixed conductive-sensorineural d. Central auditory imperceptive

C (The nurse should remove restraints periodically, while supervising the infant, to allow him or her to exercise arms and to provide cuddling and tactile stimulation. The infant should not be left in the crib, but should be removed for appropriate holding and stimulation. Analgesia and sedation are administered for pain. Heavy sedation is not indicated. The child should not be placed in the prone position.)

During the first few days after surgery for cleft lip, which intervention should the nurse do? a. Leave infant in crib at all times to prevent suture strain. b. Keep infant heavily sedated to prevent suture strain. c. Remove restraints periodically to cuddle infant. d. Alternate position from prone to side-lying to supine.

D (Breast development.)

In girls, the initial indication of puberty is: a. Menarche. b. Growth spurt. c. Growth of pubic hair d. Breast development.

C (The growth of children on long-term inhaled steroids should be assessed frequently to assess for systemic effects of these drugs. Cough is prevented by inhaled steroids. No evidence exists that inhaled steroids cause osteoporosis. Cushing's syndrome is caused by long-term systemic steroids.)

It is now recommended that children with asthma who are taking long-term inhaled steroids should be assessed frequently because they may develop: a. Cough. b. Osteoporosis. c. Slowed growth. d. Cushing's syndrome.

A (Being involved in immunization clinics for children.)

Prevention of hearing impairment in children is a major goal for the nurse. This can be achieved through: a. Being involved in immunization clinics for children. b. Assessing a newborn for hearing loss. c. Answering parents' questions about hearing aids. d. Participating in hearing screening in the community.

D (The inability to speak indicates a foreign-body airway obstruction of the larynx. Abdominal thrusts are needed for treatment of the choking child. Gagging indicates irritation at the back of the throat, not obstruction. Coughing does not indicate a complete airway obstruction. Tachycardia may be present for many reasons.)

The mother of a toddler yells to the nurse, "Help! He is choking to death on his food." The nurse determines that lifesaving measures are necessary based on: a. Gagging. b. Coughing. c. Pulse over 100 beats/min. d. Inability to speak.

C (Claiming)

The nurse hears a primiparous woman talking to her son and telling him that his chin is just like his dad's chin. This woman's statement reflects: A. Mutuality B. Synchrony C. Claiming D. Reciprocity

C, D, E (Loss of established motor skill and bowel and bladder control indicate spinal cord compression and must be reported immediately. The child with Down syndrome may have persistent neck pain caused by spinal cord compression. These children do not have impaired pain sensation. Children with Down syndrome are not aggressive. Loss of impulse control is not seen in such children.)

The nurse is assessing a child with Down syndrome. What findings in the child should alert the nurse to report to the health care provider immediately? Select all that apply. A. Loss of pain sensation B. Loss of impulse control C. Loss of established motor skill D. Loss of established bowel control E. Loss of established bladder control

D (The hallmark of autism is an inability to maintain eye contact with another person. Limited functional play may be seen in children with autism, but it is not a hallmark of autism. Autistic children also avoid body contact, but it is not a hallmark finding. Language delay at an early age is not a hallmark of autism; however, children with autism may exhibit language delay at an early age.)

The nurse is evaluating a child for suspected autism. Which finding in the child suggests autism? A. Limited functional play B. Avoidance of body contact C. Language delay at an early age D. Inability to maintain eye contact

A (When parents speak, infants tend to get excited and dance in tune with the parent's voice. This indicates that the infant is responding well to the patient. The newborn should ideally look at the parent when the parent tries to communicate. If the newborn looks toward the wall, it indicates that the newborn is not responding well to the parent. If the newborn keeps the upper and lower limbs still when the parent interacts, it indicates that the newborn is not responding well to the patient.)

The nurse observes that the newborn is responding well to the parent. Which behavior from the newborn did the nurse observe to come to this conclusion? The newborn: A. Was dancing in tune with the parent's voice. B. Looked at the wall upon hearing the parent's voice. C. Was not kicking its legs in tune to the parent's voice. D. Was not waving its arms in tune to the parent's voice.

C (Lack of physical connection to the hospital)

What is the primary disadvantage associated with outpatient and day facility care? a. Increased cost b. Increased risk of infection c. Lack of physical connection to the hospital d. Longer separation of the child from family

B (Corticosteroids such as prednisone and prednisolone are used in short bursts to suppress the inflammatory response in inflammatory bowel disease. Antacids and antidiarrheals are not drugs of choice to treat the inflammatory process of inflammatory bowel disease. Antibiotics may be used as adjunctive therapy to treat complications.)

What is used to treat moderate-to-severe inflammatory bowel disease? a. Antacids b. Corticosteroids c. Antibiotics d. Antidiarrheal medications

C (Congenital heart disease.)

When caring for a newborn with Down syndrome, the nurse should be aware that the most common congenital anomaly associated with Down syndrome is: a. Hypospadias. b. Pyloric stenosis. c. Congenital heart disease. d. Congenital hip dysplasia.

D (Preventing dehydration by small frequent feedings is an important intervention in the febrile child. Tepid water baths may induce shivering, which raises temperature. Food should not be forced; it may result in the child vomiting. The febrile child should be dressed in light, loose clothing.)

When caring for an infant with an upper respiratory tract infection and elevated temperature, an appropriate nursing intervention is to: a. Give tepid water baths to reduce fever. b. Encourage food intake to maintain caloric needs. c. Have child wear heavy clothing to prevent chilling. d. Give small amounts of favorite fluids frequently to prevent dehydration.

A (Mutuality)

When the infant's behavior and characteristics call forth a corresponding set of maternal behaviors and characteristics, this is called: a) Mutuality b) Bonding c) Claiming d) Acquaintance

A (Speaking at an even rate)

Which action best facilitates lipreading by the hearing-impaired child? a. Speaking at an even rate b. Exaggerating pronunciation of words c. Avoiding using facial expressions d. Repeating in exactly the same way if child does not understand

A, B, D (High-arched, narrow palate Protruding tongue Transverse palmar crease)

Which assessment findings indicate to the nurse a child has Down syndrome (select all that apply)? a. High-arched, narrow palate b. Protruding tongue c. Long, slender fingers d. Transverse palmar crease e. Hypertonic muscle tone

A (Cigarettes are considered a gateway drug. Teenagers who smoke are 11.4 times more likely to use an illicit drug. Teens who refrain from smoking often have a desire to succeed in athletics. If a parent smokes, it is more likely that the teen will smoke. Cigarette smoking has declined among all groups since the 1990s.)

Which statement is true about smoking in adolescence? a. Smoking is related to other high-risk behaviors. b. Smoking is more common among athletes. c. Smoking is less common when the adolescent's parent(s) smokes. d. Smoking among adolescents is becoming more prevalent.

C (Individuals with anorexia nervosa are described as perfectionist, academically high achievers, conforming, and conscientious. Independent, disruptive, and low achieving are not part of the behavioral characteristics of anorexia nervosa.)

Young people with anorexia nervosa are often described as being: a. Independent. b. Disruptive. c. Conforming. d. Low achieving.

If nonsurgical treatment for subinvolution is ineffective, which surgical procedure is appropriate to correct the cause of this condition? a. Hysterectomy b. Laparoscopy c. Laparotomy d. D&C

d. D&C D&C allows examination of the uterine contents and removal of any retained placenta or membranes.

Which condition is a transient, self-limiting mood disorder that affects new mothers after childbirth? a. Postpartum depression b. Postpartum psychosis c. Postpartum bipolar disorder d. Postpartum blues

d. Postpartum blues Postpartum blues affects 50% to 70% of new mothers. It is believed to be related to hormonal fluctuations after childbirth.

When planning care for a laboring woman whose membranes have ruptured, the nurse recognizes that the woman's risk for _________________________ has increased. a. Intrauterine infection b. Precipitous labor c. Hemorrhage d. Supine hypotension

A (When the membranes rupture, microorganisms from the vagina can ascend into the amniotic sac and cause chorioamnionitis and placentitis. Rupture of membranes (ROM) is not associated with fetal or maternal bleeding. Although ROM may increase the intensity of contractions and facilitate active labor, it does not result in precipitous labor. ROM has no correlation with supine hypotension.)

D (Infantile glaucoma.)

A 2-year-old girl has excessive tearing and corneal haziness. The nurse knows that these symptoms may indicate: a. Viral conjunctivitis. b. Paralytic strabismus. c. Congenital cataract. d. Infantile glaucoma.

A (Dyspnea is labored breathing. Tachypnea is rapid breathing. Hypopnea is breathing that is too shallow. Orthopnea is difficulty breathing except in upright position.)

A nurse is charting that a hospitalized child has labored breathing. Which describes labored breathing? a. Dyspnea b. Tachypnea c. Hypopnea d. Orthopnea

A (The Heimlich maneuver is recommended for airway obstruction in children older than 1 year. In children younger than 1 year, back blows and chest thrusts are administered.)

Abdominal thrusts (the Heimlich maneuver) are recommended for airway obstruction in children older than: a. 1 year. b. 4 years. c. 8 years. d. 12 years.

C (Risk for Injury.)

An appropriate nursing diagnosis for a child with a cognitive dysfunction who has a limited ability to anticipate danger is: a. Impaired Social Interaction. b. Deficient Knowledge. c. Risk for Injury. d. Ineffective Coping.

B (Cochlear implant.)

An implanted ear prosthesis for children with sensorineural hearing loss is a(n): a. Hearing aid. b. Cochlear implant. c. Auditory implant. d. Amplification device.

What laboratory marker is indicative of disseminated intravascular coagulation (DIC)? A. Bleeding time of 10 minutes B. Presence of fibrin split products C. Thrombocytopenia D. Hyperfibrinogenemia

B. Presence of fibrin split products Degradation of fibrin leads to the accumulation of fibrin split products in the blood.

A (This combination of drug therapy is effective in the treatment and eradication of H. pylori.)

Bismuth subsalicylate, clarithromycin, and metronidazole are prescribed for a child with a peptic ulcer to: a. Eradicate Helicobacter pylori. b. Treat epigastric pain. c. Coat gastric mucosa. d. Reduce gastric acid production.

Because the risk for childbirth complications may be revealed, nurses should know that the point of maximal intensity (PMI) of the fetal heart tone (FHT) is: a. Usually directly over the fetal abdomen. b. In a vertex position heard above the mother's umbilicus. c. Heard lower and closer to the midline of the mother's abdomen as the fetus descends and rotates internally. d. In a breech position heard below the mother's umbilicus.

C (Nurses should be prepared for the shift. The PMI of the FHT usually is directly over the fetal back. In a vertex position it is heard below the mother's umbilicus. In a breech position it is heard above the mother's umbilicus.)

A woman who has recently given birth complains of pain and tenderness in her leg. Upon physical examination, the nurse notices warmth and redness over an enlarged, hardened area. The nurse should suspect _____ and should confirm the diagnosis by _____. A. Disseminated intravascular coagulation; asking for laboratory tests B. von Willebrand disease; noting whether bleeding times have been extended C. Thrombophlebitis; using real time and color Doppler ultrasound D. Coagulopathies; drawing blood for laboratory analysis

C. Thrombophlebitis; using real time and color Doppler ultrasound Pain and tenderness in the extremities, which show warmth, redness, and hardness, is likely thrombophlebitis. A Doppler ultrasound is a common noninvasive way to confirm diagnosis.

A (The appealing facial expression of the infant is a normal behavioral pattern and is indicative of the infant's desire to seek the parents' attention. If the infant seeks attention from other adults, it indicates that the infant is not getting required attention from the parents. Jerky movements upon touching indicate that the infant is not reacting in a normal way to the parents and may suggest that the parents are not comfortable while handling the infant. If the infant does not respond properly to the care giving tasks of the parents, it indicates that the infant lacks adequate attachment with the parents.)

During a home visit, the nurse observes that a newborn baby is well attached to the parents. Which infant behavior did the nurse most likely observe in the baby? A. The baby used appealing facial expressions. B. The baby sought attention from other adults. C. The baby's body movements were jerky when touched. D. The baby was unresponsive to the parents' caregiving.

B (The ability to hear sounds is essential for the development of speech. Babies imitate the sounds that they hear. The child will have greater difficulty learning to read, but the primary issue of concern is the effect on speech. Relationships with peers and performance at school will be affected by the child's lack of hearing. The effect will be augmented by difficulties with oral communication.)

Early detection of a hearing impairment is critical because of its effect on areas of a child's life. The nurse should evaluate further for effects of the hearing impairment on: A. Reading development. B. Speech development. C. Relationships with peers. D. Performance at school

____________________ is the most common postpartum infection.

Endometritis

A (Children with CF require a well-balanced, high-protein, high-calorie diet because of impaired intestinal absorption. Enzyme supplementation helps digest foods; other modifications are not necessary. A well-balanced diet containing fruits and vegetables is important. Fats and proteins are a necessary part of a well-balanced diet.)

In providing nourishment for a child with cystic fibrosis (CF), which factor should the nurse keep in mind? a. Diet should be high in carbohydrates and protein. b. Diet should be high in easily digested carbohydrates and fats. c. Most fruits and vegetables are not well tolerated. d. Fats and proteins must be greatly curtailed.

A (Washing both the infant's face and the mother's face.)

In the United States, the en face position is preferred immediately after birth. Nurses can facilitate this process by all of these actions except: A. Washing both the infant's face and the mother's face B. Placing the infant on the mother's abdomen or breast with their heads on the same plane C. Dimming the lights D. Delaying the instillation of prophylactic antibiotic ointment in the infant's eyes

A, D, E (a. Peer pressure d. A belief that they are invulnerable e. Impulsivity)

Injuries claim many lives during adolescence. Which factors contribute to early adolescents engaging in risk-taking behaviors (select all that apply)? a. Peer pressure b. A desire to master their environment c. Engagement in the process of separation from their parents d. A belief that they are invulnerable e. Impulsivity

A (Have an extremely developed skill in a particular area.)

Parents have learned that their 6-year-old child has autism. The nurse may help the parents to cope by explaining that the child may: a. Have an extremely developed skill in a particular area. b. Outgrow the condition by early adulthood. c. Have average social skills. d. Have age-appropriate language skills.

C (White eye reflex.)

The most common clinical manifestation of retinoblastoma is: a. Glaucoma. c. White eye reflex. b. Amblyopia. d. Sunken eye socket.

A (Pointing out that the infant turned at the sound of his voice)

The nurse can help a father in his transition to parenthood by: A. Pointing out that the infant turned at the sound of his voice B. Encouraging him to go home to get some sleep C. Telling him to tape the infant's diaper a different way D. Suggesting that he let the infant sleep in the bassinet

A (Passage of a normal brown stool indicates that the intussusception has reduced itself. This is immediately reported to the practitioner, who may choose to alter the diagnostic/therapeutic plan of care.)

The nurse is caring for a boy with probable intussusception. He had diarrhea before admission but, while waiting for administration of air pressure to reduce the intussusception, he passes a normal brown stool. The most appropriate nursing action is to: a. Notify the practitioner. b. Measure abdominal girth. c. Auscultate for bowel sounds. d. Take vital signs, including blood pressure.

B (Arterial blood gases and COHb levels are the best way to monitor CO poisoning. PaO2 monitored with pulse oximetry may be normal in the case of CO poisoning. Oxygen at 100% should be given as quickly as possible, not only if respiratory distress or other symptoms develop.)

The nurse is caring for a child with carbon monoxide (CO) poisoning associated with smoke inhalation. What is essential in this child's care? a. Monitor pulse oximetry. b. Monitor arterial blood gases. c. Administer oxygen if respiratory distress develops. d. Administer oxygen if child's lips become bright, cherry red.

A ("I am going to request a referral to a hearing specialist.")

The nurse is talking to the parent of a 13-month-old child. The mother states, "My child does not make noises like 'da' or 'na' like my sister's baby, who is only 9 months old." Which statement by the nurse would be most appropriate to make? a. "I am going to request a referral to a hearing specialist." b. "You should not compare your child to your sister's child." c. "I think your child is fine, but we will check again in 3 months." d. "You should ask other parents what noises their children made at this age."

C (Suggest that he reinsert the hearing aid.)

The nurse is talking with a 10-year-old boy who wears bilateral hearing aids. The left hearing aid is making an annoying whistling sound that the child cannot hear. The most appropriate nursing action is to: a. Ignore the sound. b. Ask him to reverse the hearing aids in his ears. c. Suggest that he reinsert the hearing aid. d. Suggest that he raise the volume of the hearing aid.

C, E (Maternal age more than 35 years increases the risk of having babies with Down syndrome. Down syndrome is caused by the presence of an extra autosomal chromosome. Down syndrome is not caused by a mutation of chromosomes. Advanced paternal age is not a risk factor for Down syndrome. There is no extra sex chromosome in children with Down syndrome.)

The nursing instructor is explaining the risk factors and pathogenesis of Down syndrome to a group of nursing students. What information should the nurse include in the explanation? Select all that apply. A. It is caused by a mutation of chromosomes. B. It is more likely to occur if the paternal age is more than 35 years. C. It is more likely to occur if the maternal age is more than 35 years. D. It is caused by acquisition of an extra sex chromosome. E. It is caused by acquisition of an extra autosomal chromosome.

B (Applying a Fox shield to the affected eye and any type of patch to the other eye.)

The school nurse is caring for a child with a penetrating eye injury. Emergency treatment includes: a. Applying a regular eye patch. b. Applying a Fox shield to the affected eye and any type of patch to the other eye. c. Applying ice until the physician is seen. d. Irrigating the eye copiously with a sterile saline solution.

C ( Testicular tumors during adolescence are generally malignant.)

The school nurse is discussing testicular self-examination with adolescent boys. Why a. Epididymitis is common during adolescence. b. Asymptomatic sexually transmitted diseases may be present. c. Testicular tumors during adolescence are generally malignant. d. Testicular tumors, although usually benign, are common during adolescence.

A (The child with celiac disease is unable to fully digest gluten, the protein found in wheat, barley, rye, and oats. Oatmeal contains gluten and is not an appropriate food selection. Rice is an appropriate choice because it does not contain gluten. Corn is digestible because it does not contain gluten. Meats do not contain gluten and can be included in the diet of a child with celiac disease.)

What food choice by the parent of a 2-year-old child with celiac disease indicates a need for further teaching? a. Oatmeal c. Corn muffin b. Rice cake d. Meat patty

C (The child may exhibit monotone speech and echolalia.)

What should the nurse keep in mind when planning to communicate with a child who has autism? a. The child has normal verbal communication. b. The child is expected to use sign language. c. The child may exhibit monotone speech and echolalia. d. The child is not listening if she is not looking at the nurse.

A (As young as possible.)

When should children with cognitive impairment be referred for stimulation and educational programs? a. As young as possible b. As soon as they have the ability to communicate in some way c. At age 3 years, when schools are required to provide services d. At age 5 or 6 years, when schools are required to provide services

B (Visually impaired mothers cannot overcome the infant's need for eye-to-eye contact)

When working with parents who have some form of sensory impairment, nurses should realize that all of these statements are true except: a) One of the major difficulties visually impaired parents experience is the skepticism of health care professionals b) Visually impaired mothers cannot overcome the infant's need for eye-to-eye contact c) The best approach for the nurse is to assess the parents' capabilities rather than focusing on their disabilities d) Technologic advances, including the Internet, can provide deaf parents with a full range of parenting activities and information

a. Social isolation

Which is the most significant factor in distinguishing those who commit suicide from those who make suicidal attempts or threats? a. Social isolation c. Degree of depression b. Level of stress d. Desire to punish others

C ("When the eye drainage improves, we'll stop giving the antibiotic ointment.")

Which statement by a parent about a child's conjunctivitis indicates that further teaching is needed? a. "I'll have separate towels and washcloths for each family member." b. "I'll notify my doctor if the eye gets redder or the drainage increases." c. "When the eye drainage improves, we'll stop giving the antibiotic ointment." d. "After taking the antibiotic for 24 hours, my child can return to school."

C (Blood stools are often a presenting sign of Meckel's diverticulum. It is associated with mild-to-profuse intestinal bleeding. It is twice as common in males as in females, and complications are more frequent in males. Meckel's diverticulum is the most common congenital malformation of the gastrointestinal tract and is present in 1% to 4% of the general population. The standard therapy is surgical removal of the diverticulum.)

Which statement is most descriptive of Meckel's diverticulum? a. It is more common in females than in males. b. It is acquired during childhood. c. Intestinal bleeding may be mild or profuse. d. Medical interventions are usually sufficient to treat the problem.

The mother-baby nurse must be able to recognize what sign of thrombophlebitis? a. Visible varicose veins b. Positive Homans' sign c. Local tenderness, heat, and swelling d. Pedal edema in the affected leg

c. Local tenderness, heat, and swelling Tenderness, heat, and swelling are classic signs of thrombophlebitis that appear at the site of the inflammation.

Nursing measures that help prevent postpartum urinary tract infection include: a. promoting bed rest for 12 hours after delivery. b. discouraging voiding until the sensation of a full bladder is present. c. forcing fluids to at least 3000 mL/day. d. encouraging the intake of orange, grapefruit, or apple juice.

c. forcing fluids to at least 3000 mL/day. Adequate fluid intake prevents urinary stasis, dilutes urine, and flushes out waste products.

At the time of a child's death, the nurse tells his mother, "We will miss him so much." The best interpretation of this is that the nurse is: a. Pretending to be experiencing grief. b. Expressing personal feelings of loss. c. Denying the mother's sense of loss. d. Talking when listening would be better.

ANS: B The death of a patient is one of the most stressful aspects of a critical care or oncology nurse. Nurses experience reactions similar to those of family members because of their involvement with the child and family during the illness. Nurses often have feelings of personal loss when a patient dies. The nurse is experiencing a normal grief response to the death of a patient. There is no implication that the mother's loss is minimized. The nurse is validating the worth of the child.

28. What information should the nurse include when teaching parents how to care for a child's gastrostomy tube at home? a. Never turn the gastrostomy button. b. Clean around the insertion site daily with soap and water. c. Expect some leakage around the button. d. Remove the tube for cleaning once a week.

ANS: B The skin around the tube insertion site should be cleaned with soap and water once or twice daily. The gastrostomy button should be rotated in a full circle during cleaning. Leakage around the tube should be reported to the physician. A gastrostomy tube is placed surgically. It is not removed for cleaning.

7. The nurse is caring. What skin care interventions for an unconscious child should be included in the plan of care? a. Avoiding use of pressure reduction on the bed. b. Massaging reddened bony prominences to prevent deep tissue damage. c. Using drawsheet to move child in bed to reduce friction and shearing injuries. d. Avoiding rinsing skin after cleansing with mild antibacterial soap to provide a protective barrier.

ANS: C A drawsheet should be used to move the child in the bed or onto a gurney to reduce friction and shearing injuries. Do not drag the child from under the arms. Bony prominences should not be massaged if reddened. Deep tissue damage can occur. Pressure-reduction devices should be used to redistribute weight instead. The skin should be cleansed with mild nonalkaline soap or soap-free cleaning agents for routine bathing.

The nurse case manager is planning a care conference about a young child who has complex health care needs and will soon be discharged home. Whom should the nurse invite to the conference? a. Family and nursing staff b. Social worker, nursing staff, and primary care physician c. Family and key health professionals involved in child's care d. Primary care physician and key health professionals involved in child's care

ANS: C A multidisciplinary conference is necessary for coordination of care for children with complex health needs. The family and key health professionals who are involved in the child's care are included. The nursing staff can address the nursing care needs of the child with the family, but other involved disciplines must be included. The family must be included in the discharge conferences, which allow them to determine what education they will require and the resources needed at home. A member of the nursing staff must be included to review the nursing needs of the child.

3. A laboring woman received an opioid agonist (meperidine) intravenously 90 minutes before she gave birth. Which medication should be available to reduce the postnatal effects of Demerol on the neonate? a. Fentanyl (Sublimaze) b. Promethazine (Phenergan) c. Naloxone (Narcan) d. Nalbuphine (Nubain)

ANS: C An opioid antagonist can be given to the newborn as one part of the treatment for neonatal narcosis, which is a state of central nervous system (CNS) depression in the newborn produced by an opioid. Opioid antagonists such as naloxone (Narcan) can promptly reverse the CNS depressant effects, especially respiratory depression. Fentanyl, promethazine, and nalbuphine do not act as opioid antagonists to reduce the postnatal effects of Demerol on the neonate. Although meperidine (Demerol) is a low-cost medication and readily available, the use of Demerol in labor has been controversial because of its effects on the neonate.

At what age do most children have an adult concept of death as being inevitable, universal, and irreversible? a. 4 to 5 years c. 9 to 11 years b. 6 to 8 years d. 12 to 16 years

ANS: C By age 9 or 10 years, children have an adult concept of death. They realize that it is inevitable, universal, and irreversible. Preschoolers and young school-age children are too young to have an adult concept of death. Adolescents have a mature understanding of death.

13. Nurses with an understanding of cultural differences regarding likely reactions to pain may be better able to help patients. Nurses should know that _____ women may be stoic until late in labor, when they may become vocal and request pain relief. a. Chinese b. Arab or Middle Eastern c. Hispanic d. African-American

ANS: C Hispanic women may be stoic early and more vocal and ready for medications later. Chinese women may not show reactions to pain. Medical interventions must be offered more than once. Arab or Middle Eastern women may be vocal in response to labor pain from the start. They may prefer pain medications. African-American women may express pain openly; use of medications for pain is more likely to vary with the individual.

C (Students with moderate cognitive impairment (IQ of 50-55) have difficulty with functional reading and arithmetic calculations. The student can perform simple manual skills, such as copying information from the board, learning safe and healthy habits, and communicating with classmates.)

A 10-year-old child has moderate cognitive impairment. With which activity would a teacher expect the child to need help? A. Copying information from the board B. Learning safe and healthy habits C. Performing arithmetic calculations D. Communicating with classmates

C (Demonstrate a dressing change on a doll.)

A nurse is preparing to perform a dressing change on a 6-year-old child with mild cognitive impairment (CI) who sustained a minor burn. Which strategy should the nurse use to prepare the child for this procedure? a. Verbally explain what will be done. b. Have the child watch a video on dressing changes. c. Demonstrate a dressing change on a doll. d. Explain the importance of keeping the burn area clean.

What PPH conditions are considered medical emergencies that require immediate treatment? A. Inversion of the uterus and hypovolemic shock B. Hypotonic uterus and coagulopathies C. Subinvolution of the uterus and idiopathic thrombocytopenic purpura D. Uterine atony and disseminated intravascular coagulation

A. Inversion of the uterus and hypovolemic shock Inversion of the uterus and hypovolemic shock are considered medical emergencies.

6. The nurse assesses a client who delivered 24 hours ago. Which of the following suggests the need for further assessment? A. Scant lochia rubra B. Chills C. Thirst and fatigue D. A temperature of 100.2oF (37.9oC)

A. Scant lochia rubra During the early postpartum period, lochia rubra should be moderate to significant. Scant lochia may indicate that large clots are blocking the flow. Thirst, fatigue and a temperature up to 100.4oF (38oC) are normal within the first 24 hours. Immediately after delivery, vasomotor changes may cause a shaking chill.

25. The nurse providing newborn stabilization must be aware that the primary side effect of maternal narcotic analgesia in the newborn is: a. respiratory depression. b. bradycardia. c. acrocyanosis. d. tachypnea.

ANS: A An infant delivered within 1 to 4 hours of maternal analgesic administration is at risk for respiratory depression from the sedative effects of the narcotic. Bradycardia is not the anticipated side effect of maternal analgesics. Acrocyanosis is an expected finding in a newborn and is not related to maternal analgesics. The infant who is having a side effect to maternal analgesics normally would have a decrease in respirations, not an increase.

33. After collecting blood by venipuncture in the antecubital fossa, what intervention should the nurse implement in order to assure control of any bleeding? a. Keep arm extended while applying a bandage to the site. b. Keep arm extended, and apply pressure to the site for a few minutes. c. Apply a bandage to the site, and keep the arm flexed for 10 minutes. d. Apply a gauze pad or cotton ball to the site, and keep the arm flexed for several minutes.

ANS: B Applying pressure to the site of venipuncture stops the bleeding and aids in coagulation. Pressure should be applied before a bandage is applied.

13. Frequent urine testing for specific gravity and glucose are required on a 6-month-old infant. How should the nurse collect small amounts of urine for these tests? a. Apply a urine-collection bag to the perineal area. b. Tape a small medicine cup to the inside of the diaper. c. Aspirate urine from cotton balls inside the diaper with a syringe. d. Aspirate urine from a superabsorbent disposable diaper with a syringe.

ANS: C To obtain small amounts of urine, use a syringe without a needle to aspirate urine directly from the diaper. If diapers with absorbent material are used, place a small gauze dressing or cotton balls inside the diaper to collect the urine, and aspirate the urine with a syringe. For frequent urine sampling, the collection bag would be too irritating to the child's skin. Taping a small medicine cup to the inside of the diaper is not feasible; the urine will spill from the cup. Diapers with superabsorbent gels absorb the urine, so there is nothing to aspirate.

The nurse recognizes that a woman is in true labor when she states: a. "I passed some thick, pink mucus when I urinated this morning." b. "My bag of waters just broke." c. "The contractions in my uterus are getting stronger and closer together." d. "My baby dropped, and I have to urinate more frequently now."

C (Regular, strong contractions with the presence of cervical change indicate that the woman is experiencing true labor. Loss of the mucous plug (operculum) often occurs during the first stage of labor or before the onset of labor, but it is not the indicator of true labor. Spontaneous rupture of membranes often occurs during the first stage of labor, but it is not the indicator of true labor. The presenting part of the fetus typically becomes engaged in the pelvis at the onset of labor, but this is not the indicator of true labor.)

For the labor nurse, care of the expectant mother begins with any or all of these situations, with the exception of: a. The onset of progressive, regular contractions. b. The bloody, or pink, show. c. The spontaneous rupture of membranes. d. Formulation of the woman's plan of care for labor.

D (Labor care begins when progressive, regular contractions begin; the blood-tinged mucoid vaginal discharge appears; or fluid is discharged from the vagina. The woman and nurse can formulate their plan of care before labor or during treatment.)

D (The infant seeks attention from any adult in the room.)

In follow-up appointments or visits with parents and their new baby, it is useful if the nurse can identify infant behaviors that can either facilitate or inhibit attachment. What is an inhibiting behavior? A. The infant cries only when hungry or wet. B. The infant's activity is somewhat predictable. C. The infant clings to the parents. D. The infant seeks attention from any adult in the room.

B (The parents hover around the infant, directing attention to and pointing at the infant)

In follow-up appointments or visits with parents and their new baby, it may be useful if the nurse can identify parental behaviours that can either facilitate or inhibit attachment. What is a facilitating behavior? a) The parents have difficulty naming the infant b) The parents hover around the infant, directing attention to and pointment at the infant c) The parents make no effort to interpret the actions or needs of the infant d) The parents do not move from fingertip touch to palmar contact and holding

B (If a child has acute epiglottitis, examination of the throat may cause complete obstruction and should be performed only when immediate intubation can take place. Stridor is aggravated when a child with epiglottitis is supine. Sore throat and pain on swallowing are early signs of epiglottitis. Epiglottitis is caused byHaemophilus influenzae in the respiratory tract.)

The nurse is assessing a child with acute epiglottitis. Examining the child's throat by using a tongue depressor might precipitate which symptom or condition? a. Inspiratory stridor b. Complete obstruction c. Sore throat d. Respiratory tract infection

A, B, D (The nurse has to follow a few guidelines while admitting the child to the hospital. The nurse should apply an identification band on the child's wrist. This helps in providing appropriate care to the child. The nurse should take the nursing admission history in order to help to identify needs of the child. The nurse should explain to the parents as well as the child about the inpatient facilities. This would make them comfortable within the hospital. Specimens should be collected and may even be ordered for other specimens upon admission. The nurse should orient the parents and the child about the hospital regulations and schedules like visiting hours and food timings.)

What are the various guidelines that the nurse has to follow for the admission of a child into the hospital? Select all that apply. A. Apply an identification band on the child's wrist. B. Obtain the nursing admission history of the child. C. Specimens for lab tests should not be taken. D. Orient the parents about the inpatient facilities. E. Hospital regulations should not be disclosed.

B (Allowing teenagers to choose between realistic options and offering consistent and structured discipline typically enhances cooperation and decreases rebelliousness. Structure helps adolescents to feel more secure and assists them in the decision-making process. Setting stricter limits typically does not decrease rebelliousness or decrease testing of parental limits. Increasing peer involvement does not typically increase self-esteem.)

Which statement is the most appropriate advice to give parents of a 16-year-old girl who is rebellious? a. "You need to be stricter so that your teen stops trying to test the limits." b. "You need to collaborate with your daughter and set limits that are perceived as being reasonable." c. "Increasing your teen's involvement with her peers will improve her self-esteem." d. "Allow your teenager to choose the type of discipline that is used in your home."

B, C, D, E (The vaccine elicits the formation of an antibody to the hepatitis B surface antigen, which is protective against hepatitis B. Hepatitis B can be transferred to an infant of a breastfeeding mother, especially if the mother's nipples are cracked. The onset of hepatitis B is insidious. Immunity develops after one exposure to hepatitis B. Hepatitis B can exist in a carrier state.)

Which statements regarding hepatitis B are correct (Select all that apply)? a. Hepatitis B cannot exist in a carrier state. b. Hepatitis B can be prevented by hepatitis B virus vaccine. c. Hepatitis B can be transferred to an infant of a breastfeeding mother. d. The onset of hepatitis B is insidious. e. Immunity to hepatitis B occurs after one attack.

A laboring woman is lying in the supine position. The most appropriate nursing action at this time is to: a. Ask her to turn to one side. b. Elevate her feet and legs. c. Take her blood pressure. d. Determine whether fetal tachycardia is present.

A (The woman's supine position may cause the heavy uterus to compress her inferior vena cava, thus reducing blood return to her heart and reducing placental blood flow. Elevating her legs will not relieve the pressure from the inferior vena cava. If the woman is allowed to stay in the supine position and blood flow to the placental is reduced significantly, fetal tachycardia may occur. The most appropriate nursing action is to prevent this from occurring by turning the woman to her side. Blood pressure readings may be obtained when the patient is in the appropriate and safest position.)

C (Although the exact cause of ASD is not known, the nurse should always help parents understand that they are not responsible for the child's condition. There are many theories about the cause of ASD, but nothing is definitive. High intake of proteins is necessary during pregnancy because it promotes proper growth and development of the fetus. Vaccines containing thimerosal are not associated with ASD. Thimerosal is a preservative found in some vaccines. Consumption of alcohol during pregnancy leads to fetal alcohol syndrome, not autism.)

A 10-year-old child is diagnosed with an autism spectrum disorder (ASD). The parents ask the nurse about the cause of the disorder. Which answer given by the nurse is most appropriate? A. "Autism is caused by a high intake of proteins during pregnancy." B. "The disorder is caused by vaccines that contain thimerosal." C. "The exact cause of autism spectrum disorders is unknown." D. "Alcohol consumption during pregnancy is linked to autism."

A (This school-age child is attempting to maintain control. The nurse should provide the girl with structured choices about when the IV will be inserted. This can be characteristic behavior when an individual needs to maintain some control over a situation. The child is trying to have some control in the hospital experience.)

A 10-year-old girl needs to have another intravenous (IV) line started. She keeps telling the nurse, "Wait a minute," and, "I'm not ready." The nurse should recognize that: a. This is normal behavior for a school-age child. b. This behavior is usually not seen past the preschool years. c. The child thinks the nurse is punishing her. d. The child has successfully manipulated the nurse in the past.

C (Able to independently care for the lenses in a responsible manner.)

A 10-year-old patient is talking to the nurse about wanting to try contact lenses instead of wearing glasses. She states that the other children at her school call her "four-eyes." Contact lenses should be prescribed for a child who is: a. At least 12 years of age. b. Able to read all the written information and instructions. c. Able to independently care for the lenses in a responsible manner. d. Confident that she really wants contact lenses.

A (The male breast responds to hormone changes. Some degree of bilateral or unilateral breast enlargement occurs frequently in boys during puberty. This is not a manifestation of Klinefelter's syndrome. Administration of estrogen or testosterone will have no effect on the reduction of breast tissue and may aggravate the condition.)

A 14-year-old boy and his parents are concerned about bilateral breast enlargement. The nurse's discussion of this should be based on knowing that: a. This is usually benign and temporary. b. This is usually caused by Klinefelter's syndrome. c. Administration of estrogen effectively reduces gynecomastia. d. Administration of testosterone effectively reduces gynecomastia.

C (A 15-year-old boy admitted with a vaso-occlusive sickle cell crisis.)

A 14-year-old boy is being admitted to the hospital for an appendectomy. Which roommate should the nurse assign with this patient? a. A 4-year-old boy who is first day post-appendectomy surgery b. A 6-year-old boy with pneumonia c. A 15-year-old boy admitted with a vaso-occlusive sickle cell crisis d. A 12-year-old boy with cellulitis

B (Moral development)

A 17-year-old tells the nurse that he is not having sex because it would make his parents very angry. This response indicates that the adolescent has a developmental lag in which area? a. Cognitive development b. Moral development c. Psychosocial development d. Psychosexual development

B (The appropriate moral development for a 17-year-old would include evidence that the teenager has internalized a value system and does not depend on parents to determine right and wrong behaviors. Adolescents who remain concrete thinkers may never advance beyond conformity to please others and avoid punishment. Cognitive development is related to moral development, but it is not the pivotal point in determining right and wrong behaviors. Identity formation is the psychosocial development task. Energy is focused within the adolescent, who exhibits behavior that is self-absorbed and egocentric. Although a task during adolescence is the development of a sexual identity, the teenager's dependence on the parents' sanctioning of right or wrong behavior is more appropriately related to moral development.)

A 17-year-old tells the nurse that he is not having sex because it would make his parents very angry. This response indicates that the adolescent has a developmental lag in which area? a. Cognitive development b. Moral development c. Psychosocial development d. Psychosexual development

C (The child's age dictates the type and extent of psychologic preparation. When a colostomy is performed, the child who is at least preschool age is told about the procedure and what to expect in concrete terms with the use of visual aids. It is necessary to prepare this age child for procedures. The preschooler is not yet concerned with body image.)

A 3-year-old child with Hirschsprung's disease is hospitalized for surgery. A temporary colostomy will be necessary. The nurse should recognize that preparing this child psychologically is: a. Not necessary because of child's age. b. Not necessary because the colostomy is temporary. c. Necessary because it will be an adjustment. d. Necessary because the child must deal with a negative body image.

D (Realize that the regression in habits and behaviors in the older child is a typical reaction and that he needs extra love and attention at this time)

A 30-year-old multiparous woman has a boy who is 2 1/2 years old and now an infant girl. She tells the nurse, "I don't know how I'll ever manage both children when I get home." Which suggestion would best help this woman alleviate sibling rivalry? A. Tell the older child that he is a big boy now and should love his new sister B. Let the older child stay with his grandparents for the first 6 weeks to allow him to adjust to the newborn C. Ask friends and relatives not to bring gifts to the older sibling because you do not want to spoil him D. Realize that the regression in habits and behaviors in the older child is a typical reaction and that he needs extra love and attention at this time

B (Giving small frequent feedings of formula combined with 1 teaspoon to 1 tablespoon of rice cereal per ounce of formula has been recommended. Milk thickening agents have been shown to decrease the number of episodes of vomiting and increase the caloric density of the formula. This may benefit infants who are underweight as a result of GERD. Placing the child in Trendelenburg position would increase the reflux. Continuous nasogastric feedings are reserved for infants with severe reflux and failure to thrive. Smaller, more frequent feedings are recommended in reflux.)

A 4-month-old infant has gastroesophageal reflux disease (GERD) but is thriving without other complications. What should the nurse suggest to minimize reflux? a. Place in Trendelenburg position after eating. b. Thicken formula with rice cereal. c. Give continuous nasogastric tube feedings. d. Give larger, less frequent feedings.

A (The medication in a metered-dose inhaler is sprayed into the spacer. The child can then inhale the medication without having to coordinate the spraying and breathing. A nebulizer is a mechanism used to administer medications, but it cannot be used with metered-dose inhalers. A peak expiratory flow meter is a measure of pulmonary function not related to medication administration. Chest physiotherapy is unrelated to medication administration.)

A 4-year-old boy needs to use a metered-dose inhaler to treat asthma. He cannot coordinate the breathing to use it effectively. The nurse should suggest that he use a: A. Spacer. B. Nebulizer. C. Peak expiratory flow meter. D. Trial of chest physiotherapy.

A, C, D (A child with a hearing impairment yells or screeches in pleasure because the child cannot hear how loud these sounds are. The child also does not respond to loud sounds and prefers nonverbal communication such as pointing. A child who has difficulty reading a book may have a visual impairment. Rapid speech with stuttering and speech delay are symptoms of fragile X syndrome.)

A 4-year-old child is seen in a clinic for a hearing impairment. What action does the nurse observe in the child to confirm hearing impairment? Select all that apply. The child: A. Screeches happily when looking at a toy. B. Has difficulty trying to read a book. C. Does not respond when an alarm sounds. D. Points at his tummy to indicate hunger. E. Speaks fast, stutters, and has speech delay.

D (Examination of the oral pharynx may cause total obstruction. The child assumes a tripod position to facilitate breathing. Forcing the child to lie down will increase the respiratory distress and anxiety. Preparation should be made to care for her if an obstruction occurs. Sitting upright, drooling, agitation, and a froglike cough indicate epiglottitis. This is a medical emergency, and tracheostomy or intubation may be necessary.)

A 4-year-old girl is brought to the emergency room. She has a "froglike" croaking sound on inspiration, is agitated, and is drooling. She insists on sitting upright. The nurse should: A. Examine her oral pharynx and report to the physician. B. Make her lie down and rest quietly. C. Auscultate her lungs and make preparations for placement in a mist tent. D. Notify the physician immediately and be prepared to assist with a tracheostomy or intubation.

A, C, D, E (Vital signs should always be taken as a part of the assessment. Medical history is important in assisting with the diagnosis in addition to knowing immunization status. Assessment of breath sounds is important in assisting with the diagnosis. Suprasternal and substernal retractions may be noted. Emergency airway equipment must be readily available in case the airway becomes obstructed. Throat culture should never be done when diagnosis of epiglottis is suspected. Manipulation of the throat can stimulate the gag reflex in an already inflamed airway and cause laryngeal spasm that will cause occlusion of the airway.)

A 5-year-old child is brought to the Emergency Department with abrupt onset of sore throat, pain with swallowing, fever, and sitting upright and forward. Acute epiglottitis is suspected. What are the most appropriate nursing interventions? (Select all that apply.) A. Vital signs B. Throat culture C. Medical history D. Assessment of breath sounds E. Emergency airway equipment readily available

D (Sinusitis is characterized by signs and symptoms of a cold that do not improve after 14 days, a low-grade fever, nasal congestion and purulent nasal discharge, headache, tenderness, a feeling of fullness over the affected sinuses, halitosis, and a cough that increases when the child is lying down. The classic symptoms of allergic rhinitis are watery rhinorrhea; itchy nose, eyes, ears, and palate; and sneezing. Symptoms occur as long as the child is exposed to the allergen. Bronchitis is characterized by a gradual onset of rhinitis and a cough that is initially nonproductive but may change to a loose cough. The manifestations of asthma may vary, with wheezing being a classic sign. The symptoms presented in the question do not suggest asthma.)

A child has had cold symptoms for more than 2 weeks, a headache, nasal congestion with purulent nasal drainage, facial tenderness, and a cough that increases during sleep. The nurse recognizes that these symptoms are characteristic of which respiratory condition? a. Allergic rhinitis b. Bronchitis c. Asthma d. Sinusitis

C (Amantadine hydrochloride may reduce symptoms related to influenza type A if administered within 24 to 48 hours of onset. It is ineffective against type B or C. A clear liquid diet is not necessary for influenza, but maintaining hydration is important. Aspirin is not recommended in children because of increased risk of Reye's syndrome. Acetaminophen or ibuprofen is a better choice. Preventive antibiotics are not indicated for influenza unless there is evidence of a secondary bacterial infection.)

A child is diagnosed with influenza, probably type A disease. Management includes: a. Clear liquid diet for hydration. b. Aspirin to control fever. c. Amantadine hydrochloride to reduce symptoms. d. Antibiotics to prevent bacterial infection.

A (The child's behavior indicates that the child is in the protest stage of separation anxiety. The child is less able to cope with separation because of stress from the illness and wants to stay with the parents. The child expresses anger indirectly by showing behavioral changes. These behavioral changes are observed in the protest stage of separation anxiety. In the despair stage, the child appears less active, depressed, and uninterested in play and refuses to eat food. The denial stage is also called the detachment stage. In this stage the child is interested in the surroundings, plays with others, and forms new but superficial relationships with others.)

A child is hospitalized for treatment of the flu. Once the child's parents leave, the child starts crying, looks for parents, attempts to leave, refuses to take medicine, hits other children, and breaks toys. What should the nurse conclude from the child's behavior? The child is in the: A. Protest stage. B. Despair stage. C. Denial stage. D. Detachment stage.

C (The playroom is a safe haven for children, free from medical or nursing procedures. The child can be returned to his or her room for the blood pressure and then escorted back to the playroom. The exam room is reserved for painful procedures that should not be performed in the child's hospital bed. Documenting that the blood pressure was not obtained because the child was in the playroom is inappropriate.)

A child is playing in the playroom. The nurse needs to take a blood pressure on the child. Which is the appropriate procedure for obtaining the blood pressure? a. Take the blood pressure in the playroom. b. Ask the child to come to the exam room to obtain the blood pressure. c. Ask the child to return to his or her room for the blood pressure, then escort the child back to the playroom. d. Document that the blood pressure was not obtained because the child was in the playroom.

A (The child does not have enough knowledge about the tonsillectomy. Therefore the child may have fear about the surgery. The nurse should explain to the child that once the tonsils are removed, they do not need "fixing" again. It helps relieve the child's fear about the operation, and the child may feel comfortable. Once the tonsillectomy has been done in the child, a second operation is not required after another throat infection. There will actually not be a need for repeating the operation at any age. The child needs to be instructed that there may be other sore throats in the future. However, the child needs to be reassured that future sore throats will not require surgery.)

A child is scheduled for a tonsillectomy and is afraid of the surgery. The child asks the nurse, "Will I need another operation when I have a sore throat again?" Which response should the nurse give to the child? A. "Once your tonsils are taken out, you will not need the surgery again." B. "You will need to repeat the surgery when you have another infection." C. "You will need to have another surgery when you turn 14 years old." D. "Once your tonsils are fixed, you will not have any more sore throats."

C (The nurse helps the child become familiar with the room so that the child knows the layout in order to avoid injury while moving around the room. Explaining to the child about the hospital departments is not necessary to make the child feel comfortable. Understanding the child's daily routine is necessary to plan activities for the child but does not increase comfort. The cleaning personnel are asked to maintain the décor of the room to avoid accidents; therefore changes should be avoided.)

A child with a temporary visual impairment was admitted to the hospital for treatment. What nursing intervention would make the child feel most comfortable in the hospital? The nurse: A. Explains the different departments of the hospital. B. Understands the child's behavior and daily routine. C. Describes the surroundings of the room and the unit. D. Asks the cleaner to move the furnishings around

D (Diagnosis of asthma is made on the basis of clinical manifestations, history, and physical examination. The causes of asthma are inflammation, bronchospasm, and obstruction. Some of the triggers of asthma are identified with allergy testing. The PEFR measures the maximum amount of air that can be forcefully exhaled in 1 minute. This can provide an objective measure of pulmonary function when compared to the child's baseline.)

A child with asthma is having pulmonary function tests. The purpose of the peak expiratory flow rate (PEFR) is to: A. Confirm the diagnosis of asthma. B. Determine the cause of asthma. C. Identify "triggers" of asthma. D. Assess the severity of asthma.

A (Children with autism spectrum disorders often are uncomfortable in a new environment and may not like to be with strangers. Therefore children with an autism spectrum disorder must be accompanied by their parents during hospitalization. While caring for a visually impaired child, the nurse modify the room according to the needs of the child. This helps prevent accidents. Because the child is not visually impaired, the nurse need not explain the surroundings of the room. Children with autism spectrum disorders often do not like assistance and prefer to perform their daily chores by themselves. Therefore the nurse should not help the child with such activities.)

A child with autism spectrum disorder is hospitalized for a treatment that will last about 1 week. How should the nurse make the child comfortable? A. Ask the parents to accompany the child. B. Modify the room according to the child's needs. C. Explain the surroundings of the room. D. Help the child perform daily routine tasks.

B (Two of the only adverse effects of rhDNase are voice alterations and laryngitis. rhDNase decreases viscosity of mucus, is given in an aerosolized form, and is safe for children younger than 12 years of age)

A child with cystic fibrosis is receiving recombinant human deoxyribonuclease (rhDNase). This drug: a. May cause mucus to thicken. b. May cause voice alterations. c. Is given subcutaneously. d. Is not indicated for children younger than 12 years.

C (Children with fragile X syndrome have impaired cognitive development and may be prescribed clonidine (Catapres) to improve attention span and decrease hyperactivity. Other interventions that can improve cognitive ability in these children include speech and language therapies, occupational therapy, and special educational programs. Aromatherapy and hydrotherapy are useful for reducing stress and anxiety. Protein and gene replacement involves replacing the defected gene. It does not improve cognitive ability. Hormone therapy can be given to treat endocrine disorders. Biotherapy is given to strengthen the patient's immunity.)

A child with fragile X syndrome was prescribed clonidine (Catapres) to improve attention and decrease hyperactivity. What other intervention may improve the child's cognitive ability? A. Aromatherapy and hydrotherapy B. Protein replacement and gene therapy C. Language and occupational therapy D. Hormone and biologic therapy

C (While caring for a child with strabismus, the nurse should cover the unaffected eye with an occlusive patch because it helps stimulate vision and movement in the weaker eye. The main reason for applying an ocular patch is to improve vision in the left eye, not to protect the right eye from dust. Applying an ocular patch on the right eye does not reduce intraocular pressure in the left eye; antiglaucoma medications can be used to reduce intraocular pressure. Applying an occlusion patch will not prevent the child from rubbing his or her eyes. The nurse should explain to the child that rubbing the eyes may cause further damage.)

A child with strabismus is undergoing treatment for impaired vision of the left eye. The nurse covers the child's right eye with an occlusion patch. Why does the nurse do so? A. To protect the right eye from dust B. To reduce intraocular pressure in the left eye C. To increase vision in the left eye D. To prevent the child from rubbing the right eye

C, D (Fragile X syndrome is an X-linked dominant syndrome with reduced penetrance. About 50% of daughters with fathers affected by fragile X syndrome will be affected because the dominant X chromosome can be from the affected father. All daughters with an affected father will be carriers. The sons get Y chromosomes from the father, so they are not necessarily carriers of the syndrome or affected by the syndrome. The sons can be carriers or affected if the syndrome is passed from the mother.)

A couple visits the hospital for a prenatal checkup. On reviewing the genetic analysis report, the nurse finds that the male partner has fragile X syndrome. What should the nurse interpret from these findings? Select all that apply. A. All of their sons will have a 50% chance of being affected. B. All of their sons will be carriers for fragile X syndrome. C. The chance of a daughter being affected is 50%. D. All daughters will be carriers for fragile X syndrome. E. All sons will be carriers and will have fragile X syndrome.

C (Irrigate eyes copiously with tap water for 20 minutes)

A father calls the emergency department nurse saying that his daughter's eyes burn after getting some dishwasher detergent in them. The nurse recommends that the child be seen in the emergency department or by an ophthalmologist. What should the nurse recommend before the child is transported? a. Keep the eyes closed. b. Apply cold compresses. c. Irrigate eyes copiously with tap water for 20 minutes. d. Prepare a normal saline solution (salt and water) and irrigate eyes for 20 minutes.

B (The mechanism of action of histamine receptor antagonists is to reduce the amount of acid present in gastric contents and may prevent esophagitis. Preventing reflux and hematemesis and increasing gastric acid production are not the modes of action of histamine receptor antagonists.)

A histamine receptor antagonist such as cimetidine (Tagamet) or ranitidine (Zantac) is ordered for an infant with gastroesophageal reflux. The purpose of this is to: a. Prevent reflux. b. Reduce gastric acid production. c. Prevent hematemesis. d. Increase gastric acid production.

A, C, E (The parents should use a basin or plastic bag for managing vomiting in the child. A blanket and pillow should be kept in the car to provide comfort. Pain medication can be administered before leaving to provide a pain-free journey home. The use of a car safety restraint system should be encouraged for the child's safety. Also, the use of a straw for drinking fluids should be encouraged except for children with oral facial surgeries.)

A hospitalized child is being released for home health care. What suggestions should the nurse provide to prepare the family for transporting the child home? Select all that apply. A. Take a basin in case of vomiting B. Avoid using the restraint system C. Keep a blanket and pillow in the car D. Discourage the use of a straw for drinking fluids E. Administer prescribed pain medication before leaving

A (Antidiarrheal medications are not recommended for the treatment of acute infectious diarrhea. These medications have adverse effects and toxicity, such as worsening of the diarrhea because of slowing of motility and ileus, or a decrease in diarrhea with continuing fluid losses and dehydration. Antidiarrheal medications are not recommended in infants and small children.)

A mother calls the clinic nurse about her 4-year-old son who has acute diarrhea. She has been giving him the antidiarrheal drug loperamide (Imodium A-D). The nurse's response should be based on knowledge that this drug is: a. Not indicated. b. Indicated because it slows intestinal motility. c. Indicated because it decreases diarrhea. d. Indicated because it decreases fluid and electrolyte losses.

A (Because toddlers have a limited concept of time, the nurse should translate the mother's statement about being back around noon by linking the arrival time to a familiar activity that takes place at that time. Saying that the child's mother will always return does not give the child any information about when his mother will visit. Twelve noon is a meaningless concept for a toddler. Saying generally that the child's mother will visit does not give the child specific information about when his mother will visit.)

A mother tells the nurse that she will visit her 2-year-old son tomorrow about noon. During the child's bath, he asks for mommy. The nurse's best reply is: A. "Mommy will be here after lunch." B. "Mommy always comes back to see you." C. "Your mommy told me yesterday that she would be here today about noon." D. "Mommy had to go home for a while, but she will be here today."

B (The depressed phase after pregnancy and delivery is known as postpartum blues, or "baby blues," where the mother may experience restlessness, insomnia, and headache. Episiotomy would cause other symptoms related to pain, such as restlessness and insomnia, but it would not cause the mother to experience headaches. The pink phase of the postpartum period is the period where the woman experiences a sense of heightened joy and feelings of well-being. Anesthesia is not given during vaginal delivery, so this is not a factor in her postpartum symptoms.)

A mother who had a vaginal delivery reports being restless and having insomnia and regular episodes of headache 4 days after childbirth. What would the nurse infer about the mother's clinical condition from these symptoms? The mother: A. Is experiencing pain from the episiotomy. B. Likely has postpartum blues. C. Is in the pink postpartum period. D. Was given spinal anesthesia during the delivery process.

B, D (The mother who wishes to breastfeed may need encouragement and support because the defect does present some logistical issues. The nipple must be positioned and stabilized well back in the infant's oral cavity so that the tongue action facilitates milk expression. The suction required to stimulate milk, absent initially, may be useful before nursing to stimulate the let-down reflex. Because breastfeeding is an option, if the mother wishes to breastfeed, medications should not be given to suppress lactation. Because breastfeeding can usually be accomplished, gavage feedings are not indicated.)

A mother who intended to breastfeed has given birth to an infant with a cleft palate. Nursing interventions should include (Select all that apply): a. Giving medication to suppress lactation. b. Encouraging and helping mother to breastfeed. c. Teaching mother to feed breast milk by gavage. d. Recommending use of a breast pump to maintain lactation until infant can suck.

B (Down syndrome)

A newborn assessment shows separated sagittal suture, oblique palpebral fissures, depressed nasal bridge, protruding tongue, and transverse palmar creases. These findings are most suggestive of: a. Microcephaly. b. Down syndrome. c. Cerebral palsy. d. Fragile X syndrome.

B (For parents, cleft lip and cleft palate deformities are particularly disturbing. The nurse must place emphasis not only on the infant's physical needs but also on the parents' emotional needs. The mother needs to be able to express her feelings before the acceptance of her child can occur. Although discussing plastic surgery will be addressed, it is not part of the initial therapeutic approach. As the mother expresses her feelings, the nurse's actions should convey to the parents that the infant is a precious human being. The child's normalcy is emphasized, and the mother is assisted to recognize the child's uniqueness. A focus on abnormal maternal-infant attachment would be inappropriate at this time.)

A newborn was admitted to the nursery with a complete bilateral cleft lip and palate. The physician explained the plan of therapy and its expected good results. However, the mother refuses to see or hold her baby. Initial therapeutic approach to the mother should be to: a. Restate what the physician has told her about plastic surgery. b. Encourage her to express her feelings. c. Emphasize the normalcy of her baby and the baby's need for mothering. d. Recognize that negative feelings toward the child continue throughout childhood.

B (Allow the child to hold the digital thermometer while taking the child's blood pressure.)

A nurse in the emergency department is assessing a 5-year-old child with symptoms of pneumonia and a fever of 102° F. Which intervention can the nurse implement to promote a sense of control for the child? a. None, this is an emergency and the child should not participate in care. b. Allow the child to hold the digital thermometer while taking the child's blood pressure. c. Ask the child if it is OK to take a temperature in the ear. d. Have parents wait in the waiting room.

A (In bronchial asthma, spasm of the smooth muscle of the bronchi and bronchioles causes constriction, producing impaired respiratory function. In bronchial asthma, there is increased resistance in the airway. There are multiple causes of asthma, including allergens, irritants, exercise, cold air, infections, medications, medical conditions, and endocrine factors. Atopy or development of an immunoglobulin E (IgE)-mediated response is inherited but is not the only cause of asthma.)

A nurse is conducting an in-service on asthma. Which statement is the most descriptive of bronchial asthma? a. There is heightened airway reactivity. b. There is decreased resistance in the airway. c. The single cause of asthma is an allergic hypersensitivity. d. It is inherited.

C, D, E (High-fiber foods include whole grain breads, bran pancakes, and raw carrots. Unrefined (brown) rice is high in fiber but white rice is not. Raw fruits, especially those with skins or seeds, other than ripe banana or avocados are high in fiber.)

A nurse is conducting dietary teaching on high-fiber foods for parents of a child with constipation. Which foods should the nurse include as being high in fiber (Select all that apply)? a. White rice b. Avocados c. Whole grain breads d. Bran pancakes e. Raw carrots

A (Clinical evidence of a positive TST in children receiving immunosuppressive therapy, including immunosuppressive doses of steroids, or who have immunosuppressive conditions, including HIV infection, is an induration of 5 mm. Children younger than 4 years of age (a) with other medical risk conditions, including Hodgkin's disease, lymphoma, diabetes mellitus, chronic renal failure, or malnutrition; (b) born or whose parents were born in high-prevalence tuberculosis (TB) regions of the world; (c) frequently exposed to adults who are HIV infected, homeless, users of illicit drugs, residents of nursing homes, incarcerated or institutionalized, or migrant farm workers; and (d) who travel to high-prevalence TB regions of the world are positive when the induration is 10 mm. Children 4 years of age or older without any risk factors are positive when the induration is 20 mm.)

A nurse is interpreting the results of a tuberculin skin test (TST) on an adolescent who is human immunodeficiency virus (HIV) positive. Which induration size indicates a positive result for this child 48 to 72 hours after the test? a. 5 mm c. 15 mm b. 10 mm d. 20 mm

B, C, D (b. Administration of chamomile tea at bedtime c. Hypnotherapy for relief of pain d. Acupressure to relieve headaches)

A nurse is interviewing the parents of a toddler about use of complementary or alternative medical practices. The parents share several practices they use in their household. Which should the nurse document as complementary or alternative medical practices (select all that apply)? a. Use of acetaminophen (Tylenol) for fever b. Administration of chamomile tea at bedtime c. Hypnotherapy for relief of pain d. Acupressure to relieve headaches e. Cool mist vaporizer at the bedside for "stuffiness"

D (Chronic otitis media.)

A nurse is preparing a teaching session for parents on prevention of childhood hearing loss. The nurse should include that the most common cause of hearing impairment in children is: a. Auditory nerve damage. b. Congenital ear defects. c. Congenital rubella. d. Chronic otitis media.

C (Perform the exam while the child is on the parent's lap.)

A nurse is preparing to complete an admission assessment on a 2-year-old child. The child is sitting on the parent's lap. Which technique should the nurse implement to complete the physical exam? a. Ask the parent to place the child in the hospital crib. b. Take the child and parent to the exam room. c. Perform the exam while the child is on the parent's lap. d. Ask the child to stand by the parent while completing the exam.

B ("The onset of autism usually occurs before 3 years of age.")

A nurse is providing a parent information regarding autism. Which statement made by the parent indicates understanding of the teaching? a. "Autism is characterized by periods of remission and exacerbation." b. "The onset of autism usually occurs before 3 years of age." c. "Children with autism have imitation and gesturing skills." d. "Autism can be treated effectively with medication."

C (Androgen.)

A nurse is reviewing hormone changes that occur during adolescence. The hormone that is responsible for the growth of beard, mustache, and body hair in the male is: a. Estrogen. b. Pituitary hormone. c. Androgen. d. Progesterone.

C (Beard, mustache, and body hair on the chest, upward along the linea alba, and sometimes on other areas (e.g., back and shoulders) appears in males and is androgen dependent. Estrogen and progesterone are produced by the ovaries in the female and do not contribute to body hair appearance in the male. The pituitary hormone does not have any relationship to body hair appearance in the male.)

A nurse is reviewing hormone changes that occur during adolescence. The hormone that is responsible for the growth of beard, mustache, and body hair in the male is: a. Estrogen. b. Pituitary hormone. c. Androgen. d. Progesterone.

A (The first sign of pubertal changes in boys is testicular enlargement in response to testosterone secretion, which usually occurs in Tanner stage 2. Slight pubic hair is present and the smooth skin texture of the scrotum is somewhat altered. As testosterone secretion increases, the penis, testes, and scrotum enlarge. During Tanner stages 4 and 5, rising levels of testosterone cause the voice to deepen and facial hair appears at the corners of the upper lip and chin.)

A nurse is teaching adolescent boys about pubertal changes. The first sign of pubertal change seen with boys is: a. Testicular enlargement. b. Facial hair. c. Scrotal enlargement. d. Voice deepens.

Z (Testicular enlargement.)

A nurse is teaching adolescent boys about pubertal changes. The first sign of pubertal change seen with boys is: a. Testicular enlargement. b. Facial hair. c. Scrotal enlargement. d. Voice deepens.

B, C, D (Allows the child to express feelings)

A nurse plans therapeutic play time for a hospitalized child. Which are the benefits of therapeutic play (select all that apply)? a. Serves as method to assist disturbed children b. Allows the child to express feelings c. The nurse can gain insight into the child's feelings d. The child can deal with concerns and feelings e. Gives the child a structured play environment

A (Excessive rubbing of the eyes.)

A nurse would suspect possible visual impairment in a child who displays: a. Excessive rubbing of the eyes. b. Rapid lateral movement of the eyes. c. Delay in speech development. d. Lack of interest in casual conversation with peers.

B (Giardia is a parasite that represents 15% of nondysenteric illness in the United States. Shigella, Salmonella, and E. coli are bacterial pathogens.)

A parasite that causes acute diarrhea is: a. Shigella organisms. b. Giardia lamblia. c. Salmonella organisms. d. Escherichia coli.

B (The best method for early detection of cognitive disorders.)

A parent asks the nurse why a developmental assessment is being conducted for a child during a routine well-child visit. The nurse answers based on the knowledge that routine developmental assessments during well-child visits are: a. Not necessary unless the parents request them. b. The best method for early detection of cognitive disorders. c. Frightening to parents and children and should be avoided. d. Valuable in measuring intelligence in children.

B (May be caused by a variety of factors.)

A parent whose child has been diagnosed with a cognitive deficit should be counseled that intellectual impairment: a. Is usually due to a genetic defect. b. May be caused by a variety of factors. c. Is rarely due to first-trimester events. d. Is usually caused by parental intellectual impairment.

C (Swimming is well tolerated in children with asthma because they are breathing air fully saturated with moisture and because of the type of breathing required in swimming. Exercise-induced bronchospasm is more common in sports that involve endurance, such as soccer, running, and basketball. Prophylaxis with medications may be necessary.)

A parent whose two school-age children have asthma asks the nurse in what sports, if any, they can participate. The nurse should recommend: a. Soccer. b. Running. c. Swimming. d. Basketball.

A, B, C (The patient reports having problems with lactation. Thus the nurse should assist in establishing and maintaining a successful breastfeeding process for the patient. To do this, the nurse should teach the patient about the infant's rooting and sucking reflexes. The patient should be asked to immediately breastfeed the child after birth. Then the nurse has to explain frequent milk expression and kangaroo care to the patient for optimized milk supply. Placing the newborn skin-to-skin with the parent immediately after birth is not useful for breastfeeding. The nurse has to talk to the patient about the infant's capabilities for interaction while teaching the patient how to nurture the infant in the first year of life, but these interventions are not in response to problems with lactation.)

A patient reports having difficulty ejecting milk while breastfeeding the infant. Which nursing interventions would help the patient breastfeed the newborn with greater ease? Select all that apply. A. Teach the patient about the infant's rooting and sucking reflexes. B. Ask the patient to breastfeed the newborn immediately after birth. C. Explain frequent milk expression and kangaroo care to the patient. D. Talk to the patient about the newborn's capabilities for interaction. E. Ensure the newborn has good contact with the patient immediately after birth.

A, C (Rubella infections during pregnancy may cause hearing and visual loss in the newborn. However, these impairments may disappear as the child grows. Rubella infections do not cause growth retardation. Growth hormone deficiency or Turner syndrome can lead to growth impairment. Respiratory disorders or allergic reactions can result from hypersensitivities and can cause difficulty breathing in the newborn. A decreased ability to concentrate indicates impaired cognition. It usually results from inadequate intake of omega-3 fatty acids by the mother during pregnancy.)

A pregnant woman is diagnosed with a rubella infection during a prenatal checkup. What does the nurse expect the health care provider will tell the patient? Select all that apply. "The newborn may: A. Have vision difficulties." B. Have growth impairment." C. Have difficulty hearing." D. Develop breathing problems." E. Not be able to concentrate."

B (Women express a need to review their childbirth experience and evaluate their performance. After the mother's needs are met, she is more able to take an active role, not only in her own care but also in the care of her newborn. Short teaching sessions (using written materials to reinforce the content presented) are a more effective approach. The focus of the taking-in stage is nurturing the new mother by meeting her dependency needs for rest, comfort, hygiene, and nutrition. Test-Taking Tip: After choosing an answer, go back and reread the question stem along with your chosen answer. Does it fit correctly? The choice that grammatically fits the stem and contains the correct information is the best choice.)

A primiparous woman is in the taking-in stage of psychosocial recovery and adjustment following birth. Recognizing the needs of women during this stage, the nurse should: A. Foster an active role in the baby's care. B. Provide time for the mother to reflect on the events of and her behavior during childbirth. C. Recognize the woman's limited attention span by giving her written materials to read when she gets home rather than doing a teaching session now. D. Promote maternal independence by encouraging her to meet her own hygiene and comfort needs.

B (A significant number of teenage suicides occur among homosexual youths. Gay and lesbian adolescents who live in families or communities that do not accept homosexuality are likely to suffer low self-esteem, self-loathing, depression, and hopelessness as a result of a lack of acceptance from their family or community. At-risk teenagers include those who are depressed, have poor problem-solving skills, or use drugs and alcohol. History of previous suicide attempt is a serious indicator for possible suicide completion in the future.)

A school nurse is conducting a class with adolescents on suicide. Which true statement about suicide should the nurse include in the teaching session? a. A sense of hopelessness and despair are a normal part of adolescence. b. Gay and lesbian adolescents are at a particularly high risk for suicide. c. Problem-solving skills are of limited value to the suicidal adolescent. d. Previous suicide attempts are not an indication of risk for completed suicides.

A (Bronchitis is characterized by these symptoms and occurs in children older than 6 years. Bronchiolitis is rare in children older than 2 years. Asthma is a chronic inflammation of the airways that may be exacerbated by a virus. Acute spasmodic laryngitis occurs in children between 3 months and 3 years.)

A school-age child has had an upper respiratory tract infection for several days and then began having a persistent dry, hacking cough that was worse at night. The cough has become productive in the past 24 hours. This is most suggestive of: a. Bronchitis. b. Bronchiolitis. c. Viral-induced asthma. d. Acute spasmodic laryngitis.

C (Create a schedule similar to the one the child follows at home.)

A school-age child, admitted for intravenous antibiotic therapy for osteomyelitis, reports difficulty in going to sleep at night. Which intervention should the nurse implement to assist the child in going to sleep at bedtime? a. Request a prescription for a sleeping pill. b. Allow the child to stay up late and sleep late in the morning. c. Create a schedule similar to the one the child follows at home. d. Plan passive activities in the morning and interactive activities right before bedtime.

D (Neutrophils and red blood cells in stool indicate bacterial gastroenteritis. Protein intolerance is suspected in the presence of eosinophils. Parasitic infection is indicated by eosinophils. Fat malabsorption is indicated by foul-smelling, greasy, bulky stools.)

A stool specimen from a child with diarrhea shows the presence of neutrophils and red blood cells. This is most suggestive of which condition? a. Protein intolerance b. Fat malabsorption c. Parasitic infection d. Bacterial gastroenteritis

B (Physical dependence is an involuntary response to the pharmacologic characteristics of drugs such as opioids or alcohol. A person can be physically dependent on a narcotic/drug without being addicted; for example, patients who use opioids to control pain need increasing doses to achieve the same effect. Dependence is a physiologic response; it is not culturally determined or subject to voluntary control.)

A teen asks a nurse, "What is physical dependence in substance abuse?" Which is the correct response by the nurse? a. Problem that occurs in conjunction with addiction b. Involuntary physiologic response to drug c. Culturally defined use of drugs for purposes other than accepted medical purposes d. Voluntary behavior based on psychosocial needs

A (One of the characteristics of Down syndrome is a short, broad neck. These children have an impaired immune system and are at risk for spinal cord compression. Physical features such as long and thin fingers, short and thin lips, and broad and long nose are all common in a normal child and do not indicate any abnormality.)

A week-old newborn is assessed for body weight, birth marks, and height. The birth weight is lower than what it should be for height. Which physical feature of the newborn makes the nurse conclude that the newborn is affected by Down syndrome? A. Short and broad neck B. Long and thin fingers C. Short and thin lips D. Broad and long nose

A (Growth in length of the extremities and neck precedes growth in other areas, and, because these parts are the first to reach adult length, the hands and feet appear larger than normal during adolescence. Increases in hip and chest breadth take place in a few months, followed several months later by an increase in shoulder width. These changes are followed by increases in length of the trunk and depth of the chest. This sequence of changes is responsible for the characteristic long-legged, gawky appearance of early adolescent children. The growth spurt occurs earlier in girls than in boys.)

A young adolescent boy tells the nurse he "feels gawky." The nurse should explain that this occurs in adolescents because: a. Growth of the extremities and neck precedes growth in other areas. b. Growth is in the trunk and chest. c. The hip and chest breadth increases. d. The growth spurt occurs earlier in boys than it does in girls.

A (Intravenous fluids are initiated in children with severe dehydration. ORS is acceptable therapy if the dehydration is not severe. Diarrhea is not managed by using clear liquids by mouth. These fluids have a high carbohydrate content, low electrolyte content, and high osmolality. Antidiarrheal medications are not recommended for the treatment of acute infectious diarrhea.)

A young child is brought to the emergency department with severe dehydration secondary to acute diarrhea and vomiting. Therapeutic management of this child will begin with: a. Intravenous fluids. b. Oral rehydration solution (ORS). c. Clear liquids, 1 to 2 ounces at a time. d. Administration of antidiarrheal medication.

Women who have participated in childbirth education classes often bring a "birth bag" or "Lamaze bag" with them to the hospital. These items often assist in reducing stress and providing comfort measures. The nurse caring for women in labor should be aware of common items that a client may bring, including (Select all that apply): a. Rolling pin. b. Tennis balls. c. Pillow. d. Stuffed animal or photo. e. Candles.

A, B, C, D (The rolling pin and tennis balls are used to provide counterpressure, especially if the woman is experiencing back labor. Although the facility has plenty of pillows, when the client brings her own, it is a reminder of home and provides added comfort. A stuffed animal or framed photo can be used to provide a focal point during contractions. Although many women find the presence of candles conducive to creating calm and relaxing surroundings, these are not suitable for a hospital birthing room environment. Oxygen may be in use, resulting in a fire hazard. Flameless candles are often sold in hospital gift shops. It is also important for the nurse to orient the patient and her family to the call bell and light switches to familiarize herself with the environment.)

In caring for the woman with disseminated intravascular coagulation (DIC), what order should the nurse anticipate? A. Administration of blood B. Preparation of the client for invasive hemodynamic monitoring C. Restriction of intravascular fluids D. Administration of steroids

A. Administration of blood Primary medical management in all cases of DIC involves correction of the underlying cause, volume replacement, blood component therapy, optimization of oxygenation and perfusion status, and continued reassessment of laboratory parameters.

In caring for an immediate postpartum client, you note petechiae and oozing from her IV site. You would monitor her closely for the clotting disorder: A. Disseminated intravascular coagulation B. Amniotic fluid embolism C. Hemorrhage D. HELLP syndrome

A. Disseminated intravascular coagulation The diagnosis of DIC is made according to clinical findings and laboratory markers. Physical examination reveals unusual bleeding. Petechiae may appear around a blood pressure cuff on the woman's arm. Excessive bleeding may occur from the site of a slight trauma, such as venipuncture sites.

Medications used to manage postpartum hemorrhage include (choose all that apply): A. Pitocin B. Methergine C. Terbutaline D. Hemabate E. Magnesium sulfate

A. Pitocin B. Methergine D. Hemabate

The perinatal nurse caring for the postpartum woman understands that late postpartum hemorrhage is most likely caused by: A. Subinvolution of the placental site B. Defective vascularity of the decidua C. Cervical lacerations D. Coagulation disorders

A. Subinvolution of the placental site Late PPH may be the result of subinvolution of the uterus, pelvic infection, or retained placental fragments.

8. A woman with a past history of varicose veins has just delivered and the nurse suspects she has developed a pulmonary embolism. Which of the data below would lead to this nursing judgment? A. Sudden dyspnea and confusion B. Hypertension C. Chills and fever D. Leg pain

A. Sudden dyspnea and confusion Rationale: Sudden dyspnea, diaphoresis and confusion are the classic signs of the dislodgment of a thrombus (stationary blood clot) from a varicose vein and its travel to and its becoming enlodged in the pulmonary circulation. Chills and fever would indicate infection. A person with a pulmonary embolism would be hypotensive and not hypertensive.

The perinatal nurse is caring for a woman in the immediate postbirth period. Assessment reveals that the woman is experiencing profuse bleeding. The most likely etiology for the bleeding is: A. Uterine atony B. Uterine inversion C. Vaginal hematoma D. Vaginal laceration

A. Uterine atony Uterine atony is marked hypotonia of the uterus. It is the leading cause of postpartum hemorrhage.

18. When liquid medication is given to a crying 10-month-old infant, which approach minimizes the possibility of aspiration? a. Administering the medication with a syringe (without needle) placed along the side of the infant's tongue. b. Administering the medication as rapidly as possible with the infant securely restrained. c. Mixing the medication with the infant's regular formula or juice and administering by bottle. d. Keeping the child upright with the nasal passages blocked for a minute after administration.

ANS: A Administer the medication with a syringe without needle placed alongside of the infant's tongue. The contents are administered slowly in small amounts, allowing the child to swallow between deposits. Medications should be given slowly to avoid aspiration. The medication should be mixed with only a small amount of food or liquid. If the child does not finish drinking/eating, it is difficult to determine how much medication was consumed. Essential foods also should not be used. The child may associate the altered taste with the food and refuse to eat in future. Holding the child's nasal passages increases the risk of aspiration.

12. The nurse gives an injection in a patient's room. Which method should the nurse use to dispose of the needle? a. Dispose of syringe and needle in a rigid, puncture-resistant container in patient's room. b. Dispose of syringe and needle in a rigid, puncture-resistant container in an area outside of patient's room. c. Cap needle immediately after giving injection and dispose of in proper container. d. Cap needle, break from syringe, and dispose of in proper container.

ANS: A All needles (uncapped and unbroken) are disposed of in a rigid, puncture-resistant container located near the site of use. Consequently, these containers should be installed in the patient's room. The uncapped needle should not be transported to an area distant from use.

The nurse is providing support to parents at the time their child is diagnosed with chronic disabilities. The nurse notices that the parents keep asking the same questions. The nurse should: a. Patiently continue to answer questions. b. Kindly refer them to someone else to answer their questions. c. Recognize that some parents cannot understand explanations. d. Suggest that they ask their questions when they are not upset.

ANS: A Diagnosis is one of the anticipated stress points for parents. The parents may not hear or remember all that is said to them. The nurse should continue to provide the kind of information that they desire. This is a particularly stressful time for the parents; the nurse can play a key role in providing necessary information. Parents should be provided with oral and written information. The nurse needs to work with the family to ensure understanding of the information. The parents require information at the time of diagnosis. Other questions will arise as they adjust to the information.

The parents of a child born with disabilities ask the nurse for advice about discipline. The nurse's response should be based on knowledge that discipline is: a. Essential for the child. b. Too difficult to implement with a special-needs child. c. Not needed unless the child becomes problematic. d. Best achieved with punishment for misbehavior.

ANS: A Discipline is essential for the children with disabilities. It provides boundaries within which to test their behavior and teaches them socially acceptable behaviors. It is not too difficult to implement discipline with a special-needs child. The nurse should teach the parents ways to manage the child's behavior before it becomes problematic. Punishment is not effective in managing behavior.

3. The most appropriate nursing action to implement when a preschooler being prepped for outpatient surgery refused to allow the parent to remove his/her underwear? a. Allow the child to wear their underpants. b. Discuss to the mother why this is important. c. Ask the mother to explain to her child why he/she must remove the underwear. d. Explain in a kind, matter-of-fact manner that this is hospital policy.

ANS: A It is appropriate for the child to leave his/her underpants on. This allows his/her some measure of control during the foot surgery. The mother should not be required to make the child more upset. The child is too young to understand what hospital policy means.

31. A 6-year-old child is hospitalized for intravenous (IV) antibiotic therapy. He eats little on his "regular diet" trays. He tells the nurse that all he wants to eat is pizza, tacos, and ice cream. Which is the best nursing action? a. Request these favorite foods for him. b. Identify healthier food choices that he likes. c. Explain that he needs fruits and vegetables. d. Reward him with ice cream at the end of every meal that he eats.

ANS: A Loss of appetite is a symptom common to most childhood illnesses. To encourage adequate nutrition, favorite foods should be requested for the child. Even though these substances are not nutritious, they can provide necessary fluid and calories and can be supplemented with additional fruits and vegetables. Ice cream and other desserts should not be used as rewards or punishment.

9. A 3 year old has a 102° F fever associated with a viral illness that has not responded to acetaminophen. The nurse's action should be based on what knowledge about fevers in children? a. Fevers such as this are common with viral illnesses. b. Seizures are common in children when antipyretics are ineffective. c. Fever over 102° F indicates greater severity of illness. d. Fever over 102° F indicates a probable bacterial infection.

ANS: A Most fevers are of brief duration, have limited consequences, and are viral. Little evidence supports the use of antipyretic drugs to prevent febrile seizures. Neither the increase in temperature nor its response to antipyretics indicates the severity or etiology of infection.

A common parental reaction to a child with special needs is parental overprotection. Parental behavior suggestive of this includes: a. Giving inconsistent discipline. b. Providing consistent, strict discipline. c. Forcing child to help self, even when not capable. d. Encouraging social and educational activities not appropriate to child's level of capability.

ANS: A Parental overprotection is manifested by the parents' fear of letting the child achieve any new skill, avoiding all discipline, and catering to the child's every desire to prevent frustration. The overprotective parents usually do not set limits and or institute discipline, and they usually prefer to remain in the role of total caregiver. They do not allow the child to perform self-care or encourage the child to try new activities.

20. When teaching a mother how to administer eyedrops, where should the nurse instruct to place them? a. In the conjunctival sac that is formed when the lower lid is pulled down. b. Carefully under the upper eyelid while it is gently pulled upward. c. On the sclera while the child looks to the side. d. Anywhere as long as drops contact the eye's surface.

ANS: A The lower lid is pulled down, forming a small conjunctival sac. The solution or ointment is applied to this area. The medication should not be administered directly on the eyeball.

The nurse is caring for a child who has just died. The parents ask to be left alone so that they can rock their child one more time. The nurse should: a. Grant their request. b. Assess why they feel that this is necessary. c. Discourage this because it will only prolong their grief. d. Kindly explain that they need to say good-bye to their child now and leave.

ANS: A The parents should be allowed to remain with their child after the death. The nurse can remove all of the tubes and equipment and offer the parents the option of preparing the body. This is an important part of the grieving process and should be allowed if the parents desire it. It is important for the nurse to ascertain if the family has any special needs.

What is most descriptive of a school-age child's reaction to death? a. Is very interested in funerals and burials b. Has little understanding of words such as forever c. Imagines the deceased person to be still alive d. Has an idealistic view of the world and criticizes funerals as barbaric

ANS: A The school-age child is very interested in postdeath services and may be inquisitive about what happens to the body. School-age children have an established concept of forever and have a deeper understanding of death in a concrete manner. Toddler may imagine the deceased person to still be alive. Adolescents may respond to death with an idealistic view of the world and criticize funerals as barbaric.

Which is the most appropriate nursing intervention to promote normalization in a school-age child with a chronic illness? a. Give child as much control as possible. b. Ask child's peer to make child feel normal. c. Convince child that nothing is wrong with him or her. d. Explain to parents that family rules for the child do not need to be the same as for healthy siblings.

ANS: A The school-age child who is ill may be forced into a period of dependency. To foster normalcy, the child should be given as much control as possible. It is unrealistic to expect one individual to make the child feel normal. The child has a chronic illness. It would be unacceptable to convince the child that nothing is wrong. The family rules should be similar for each of the children in a family. Resentment and hostility can arise if different standards are applied to each child.

The nurse is talking with the parents of a child who died 6 months ago. They sometimes still "hear" the child's voice and have trouble sleeping. They describe feeling "empty" and depressed. The nurse should recognize that: a. These are normal grief responses. b. The pain of the loss is usually less by this time. c. These grief responses are more typical of the early stages of grief. d. This grieving is essential until the pain is gone and the child is gradually forgotten.

ANS: A These are normal grief responses. The process of grief work is lengthy and resolution of grief may take years, with intensification during the early years. The child will never be forgotten by the parents.

What is a priority nursing diagnosis for the preschool child with chronic illness? a. Risk for Delayed Growth and Development related to chronic illness or disability b. Chronic Pain related to frequent injections c. Anticipatory Grieving related to impending death d. Anxiety related to frequent hospitalizations

ANS: A This is the priority nursing diagnosis that is appropriate for the majority of chronic illnesses. Pain is not associated with the majority of chronic illnesses. A chronic illness is one that does not have a cure. It does not mean the child will die prematurely. Frequent hospitalizations are not necessarily required for many chronic illnesses.

11. The nurse wore gloves during a dressing change. When the gloves are removed, the nurse should perform which initial action? a. Wash hands thoroughly. b. Check the gloves for leaks. c. Rinse gloves in disinfectant solution. d. Apply new gloves before touching the next patient.

ANS: A When gloves are worn, the hands are washed thoroughly after removing the gloves because both latex and vinyl gloves fail to provide complete protection. Gloves should be disposed of after use and hands should be thoroughly washed again before new gloves are applied.

30. What critical information should the nurse incorporate into care when using restraints on a child? a. Use the least restrictive type of restraint. b. Tie knots securely so they cannot be untied easily. c. Secure the ties to the mattress or side rails. d. Remove restraints every 4 hours to assess skin.

ANS: A When restraints are necessary, the nurse should institute the least restrictive type of restraint. Knots must be tied so that they can be easily undone for quick access to the child. The ties are never tied to the mattress or side rails. They should be secured to a stable device, such as the bed frame. Restraints are removed every 2 hours to allow for range of motion, position changes, and assessment of skin integrity.

2. The advantages of the ventrogluteal muscle as an injection site in young children include which of the following? (Select all that apply.) a. Less painful than vastus lateralis b. Free of important nerves and vascular structures c. Cannot be used when child reaches a weight of 20 lbs d. Increased subcutaneous fat, which increases drug absorption e. Easily identified by major landmarks

ANS: A, B, E Less painful, free of important nerves and vascular structures, and easily identifiable are advantages of the ventrogluteal muscle. The major disadvantage is lack of familiarity by health professionals and controversy over whether the site can be used before weight bearing. Cannot be used when a child is 20 lbs or more and increased subcutaneous fat are not advantages of the ventrogluteal muscle as an injection site in young children.

29. Which nursing action is the most appropriate when applying a face mask to a child prescribed oxygen therapy? a. Set the oxygen flow rate at less than 6 L/min. b. Make sure the mask fits properly. c. Keep the child warm. d. Remove the mask for 5 minutes every hour.

ANS: B A properly fitting face mask is essential for adequate oxygen delivery. The oxygen flow rate should be greater than 6 L/min to prevent rebreathing of exhaled carbon dioxide. Oxygen delivery through a face mask does not affect body temperature. A face mask used for oxygen therapy is not routinely removed.

Kelly, age 8 years, will soon be able to return to school after an injury that resulted in several severe, chronic disabilities. What is the most appropriate action by the school nurse? a. Recommending that Kelly's parents attend school at first to prevent teasing b. Preparing Kelly's classmates and teachers for changes they can expect c. Referring Kelly to a school where the children have chronic disabilities similar to hers d. Discussing with Kelly and her parents the fact that her classmates will not accept her as they did before

ANS: B Attendance at school is an important part of normalization for Kelly. The school nurse should prepare teachers and classmates about her condition, abilities, and special needs. A visit by the parents can be helpful, but unless the classmates are prepared for the changes, it alone will not prevent teasing. Kelly's school experience should be normalized as much as possible. Children need the opportunity to interact with healthy peers and engage in activities with groups or clubs composed of similarly affected persons. Children with special needs are encouraged to maintain and reestablish relationships with peers and participate according to their capabilities.

Which intervention will encourage a sense of autonomy in a toddler with disabilities? a. Avoiding separation from family during hospitalizations b. Encouraging independence in as many areas as possible c. Exposing child to pleasurable experiences as much as possible d. Helping parents learn special care needs of their child

ANS: B Encouraging the toddler to be independent encourages a sense of autonomy. The child can be given choices about feeding, dressing, and diversional activities, which will provide a sense of control. Avoiding separation from family during hospitalizations and helping parents learn special care needs of their child should be practiced as part of family-centered care. They do not particularly foster autonomy. Exposing the child to pleasurable experiences, especially sensory ones, is a supportive intervention. It does not particularly support autonomy.

Which intervention will encourage a sense of autonomy in a toddler with disabilities? a. Avoid separation from family during hospitalizations. b. Encourage independence in as many areas as possible. c. Expose child to pleasurable experiences as much as possible. d. Help parents learn special care needs of their child.

ANS: B Encouraging the toddler to be independent encourages a sense of autonomy. The child can be given choices about feeding, dressing, and diversional activities, which will provide a sense of control. Avoiding separation from family during hospitalizations, and helping parents learn special care needs of their child should be practiced as part of family-centered care. It does not necessarily foster autonomy. Exposing the child to pleasurable experiences, especially sensory ones, is a supportive intervention. It does not promote autonomy.

Families progress through various stages of reactions when a child is diagnosed with a chronic illness or disability. After the shock phase, a period of adjustment usually follows. This is often characterized by: a. Denial. c. Social reintegration. b. Guilt and anger. d. Acceptance of child's limitations.

ANS: B For most families, the adjustment phase is accompanied by several responses that are normally part of the adjustment process. Guilt, self-accusation, bitterness, and anger are common reactions. The initial diagnosis of a chronic illness or disability often is often met with intense emotion and characterized by shock and denial. Social reintegration and acceptance of the child's limitations is the culmination of the adjustment process.

15. What is the most appropriate statement for the nurse to make to a 5-year-old child who is undergoing a venipuncture? a. "You must hold still or I'll have someone hold you down. This is not going to hurt." b. "This will hurt like a pinch. I'll get someone to help hold your arm still so it will be over fast and hurt less." c. "Be a big boy and hold still. This will be over in just a second." d. "I'm sending your mother out so she won't be scared. You are big, so hold still and this will be over soon."

ANS: B Honesty is the best approach. Children should be told what sensation they will feel during a procedure. A 5-year-old child should not be expected to hold still, and assistance ensures safety to everyone. Telling the child that "This will be over in just a second" is not supportive or honest. Parents should be encouraged to remain with the child unless they are extremely uncomfortable doing so.

2. The nurse is planning how to best prepare a 4-year-old child for some diagnostic procedures. What guideline should the nurse consider when preparing a preschooler for a diagnostic procedure? a. Planning for a short teaching session of about 30 minutes. b. Telling the child that procedures are never a form of punishment. c. Keeping equipment out of the child's view. d. Using correct scientific and medical terminology in explanations.

ANS: B Illness and hospitalization may be viewed as punishment in preschoolers. Always state directly that procedures are never a form of punishment. Teaching sessions for this age-group should be 10 to 15 minutes in length. Demonstrate the use of equipment and allow the child to play with miniature or actual equipment. Explain the procedure and how it affects the child in simple terms.

21. A 2-year-old child comes to the emergency department demonstrating signs of dehydration and hypovolemic shock. Which best explains why an intraosseous infusion is started? a. It is less painful for small children. b. Rapid venous access is not possible. c. Antibiotics must be started immediately. d. Long-term central venous access is not possible.

ANS: B In situations in which rapid establishment of systemic access is vital and venous access is hampered, such as peripheral circulatory collapse and hypovolemic shock, intraosseous infusion provides a rapid, safe lifesaving alternative. The procedure is painful, and local anesthesia and systemic analgesia are given. Antibiotics could be given when vascular access is obtained. Long-term central venous access is time-consuming, and intraosseous infusion is used in an emergency situation.

26. In preparing to give "enemas until clear" to a young child, the nurse should select which solution? a. Tap water b. Normal saline c. Oil retention d. Fleet solution

ANS: B Isotonic solutions should be used in children. Saline is the solution of choice. Plain water is not used. This is a hypotonic solution and can cause rapid fluid shift, resulting in fluid overload. Oil-retention enemas will not achieve the "until clear" result. Fleet enemas are not advised for children because of the harsh action of the ingredients. The osmotic effects of the Fleet enema can result in diarrhea, which can lead to metabolic acidosis.

Which represents a common best practice in the provision of services to children with chronic or complex conditions? a. Care is focused on the child's chronologic age. b. Children with complex conditions are integrated into regular classrooms. c. Disabled children are less likely to be cared for by their families. d. Children with complex conditions are placed in residential treatment facilities.

ANS: B Normalization refers to behaviors and interventions for people with disabilities to integrate into society by living life as people without a disability would. For children, normalization includes attending school and being integrated into regular classrooms. This affords the child the advantages of learning with a wide group of peers. Care is necessarily focused on the child's developmental age. Home care by the family is considered best practice. The nurse can assist families by assessing social support systems, coping strategies, family cohesiveness, and family and community resources.

Lindsey, age 5 years, will be starting kindergarten next month. She has cerebral palsy, and it has been determined that she needs to be in a special education classroom. Her parents are tearful when telling the nurse about this and state that they did not realize that her disability was so severe. The best interpretation of this situation is that: a. This is a sign that parents are in denial. b. This is a normal anticipated time of parental stress. c. The parents need to learn more about cerebral palsy. d. The parents are used to having expectations that are too high.

ANS: B Parenting a child with a chronic illness can be very stressful for parents. There are anticipated times that parental stress increases. One of these identified times is when the child begins school. Nurses can help parents recognize and plan interventions to work through these stressful periods. The parents are not in denial; they are responding to the child's placement in school. The parents are not exhibiting signs of a knowledge deficit or expectations that are too high; this is their first interaction with the school system with this child.

4. Using knowledge of child development, what is the best approach when preparing a toddler for a procedure? a. Avoid asking the child to make choices. b. Demonstrate the procedure on a doll. c. Plan for the teaching session to last about 20 minutes. d. Show necessary equipment without allowing child to handle it.

ANS: B Prepare toddlers for procedures by using play. Demonstrate on a doll, but avoid the child's favorite doll because the toddler may think the doll is really "feeling" the procedure. In preparing a toddler for a procedure, the child is allowed to participate in care and help whenever possible. Teaching sessions for toddlers should be about 5 to 10 minutes. Use a small replica of the equipment and allow the child to handle it.

The feeling of guilt that the child "caused" the disability or illness is especially critical in which child? a. Toddler c. School-age child b. Preschooler d. Adolescent

ANS: B Preschoolers are most likely to be affected by feelings of guilt that they caused the illness/disability or are being punished for wrongdoings. Toddlers are focused on establishing their autonomy. The illness will foster dependency. The school-age child will have limited opportunities for achievement and may not be able to understand limitations. Adolescents are faced with the task of incorporating their disabilities into their changing self-concept.

14. What is an important nursing consideration when performing a bladder catheterization on a young boy? a. Use clean technique, not Standard Precautions. b. Insert 2% lidocaine lubricant into the urethra. c. Lubricate catheter with water-soluble lubricant such as K-Y Jelly. d. Delay catheterization for 20 minutes while anesthetic lubricant is absorbed.

ANS: B The anxiety, fear, and discomfort experienced during catheterization can be significantly decreased by preparing the child and parents, selecting the correct catheter, and using appropriate insertion technique. Generous lubrication of the urethra before catheterization and use of lubricant containing 2% lidocaine may reduce or eliminate the burning and discomfort associated with this procedure. Catheterization is a sterile procedure, and Standard Precautions for body-substance protection should be followed. Water-soluble lubricants do not provide appropriate local anesthesia. Catheterization should be delayed only 2 to 3 minutes. This provides sufficient local anesthesia for the procedure.

Approach behaviors are coping mechanisms that result in a family's movement toward adjustment and resolution of the crisis of having a child with a chronic illness or disability. What is considered an approach behavior in parents? a. Are unable to adjust to a progression of the disease or condition b. Anticipate future problems and seek guidance and answers c. Look for new cures without a perspective toward possible benefit d. Fail to recognize seriousness of child's condition despite physical evidence

ANS: B The parents who anticipate future problems and seek guidance and answers are demonstrating approach behaviors. They are demonstrating positive actions in caring for their child. Avoidance behaviors include being unable to adjust to a progression of the disease or condition, looking for new cures without a perspective toward possible benefit, and failing to recognize the seriousness of the child's condition despite physical evidence. These behaviors would suggest that the parents are moving away from adjustment or adaptation in the crisis of a child with chronic illness or disability.

6. The nurse monitoring a child for signs and symptoms of malignant hyperthermia should be alert for which early sign of this disorder? a. Apnea b. Bradycardia c. Muscle rigidity d. Decreased blood pressure

ANS: C Early signs of malignant hyperthermia include tachycardia, increasing blood pressure, tachypnea, mottled skin, and muscle rigidity. Apnea is not a sign of malignant hyperthermia. Tachycardia, not bradycardia, is an early sign of malignant hyperthermia. Increased, not decreased, blood pressure is characteristic of malignant hyperthermia.

10. What intervention is appropriate when administering tepid water or sponge baths prescribed for hyperthermia in children? a. Add isopropyl alcohol to the water. b. Direct a fan on the child in the bath. c. Stop the bath if the child begins to chill. d. Continue the bath for 5 minutes.

ANS: C Environmental measures such as sponge baths can be used to reduce temperature if tolerated by the child and if they do not induce shivering. Shivering is the body's way of maintaining the elevated set point. Compensatory shivering increases metabolic requirements above those already caused by the fever. Ice water and isopropyl alcohol are inappropriate, potentially dangerous solutions. Fans should not be used because of the risk of the child developing vasoconstriction, which defeats the purpose of the cooling measures. Little blood is carried to the skin surface, and the blood remains primarily in the viscera to become heated. The child is placed in a tub of tepid water for 20 to 30 minutes.

17. The nurse administering a bitter oral medication to an infant or small child should mix the medication with what substance? a. A bottle of formula or milk. b. Any food the child is going to eat. c. A teaspoon of jam or ice cream. d. Large amounts of water to dilute medication sufficiently.

ANS: C Mix the drug with a small amount (about 1 teaspoon) of sweet-tasting substance. This will make the medication more palatable to the child. The medication should be mixed with only a small amount of food or liquid. If the child does not finish drinking/eating, it is difficult to determine how much medication was consumed. Medication should not be mixed with essential foods and milk. The child may associate the altered taste with the food and refuse to eat in future.

35. What nursing consideration is related to the administration of oxygen (O2) in an infant? a. Humidify the oxygen if the infant can tolerate it. b. Assess the infant to determine how much oxygen should be given. c. Arterial oxygen saturation (SaO2) readings are used to guide O2 therapy. d. Direct the oxygen flow so that it blows directly into the infant's face in a hood.

ANS: C Pulse oximetry is a continuous, noninvasive method of determining arterial oxygen saturation (SaO2) to guide oxygen therapy. Oxygen is drying to the tissues. Oxygen should always be humidified when delivered to a patient. A child receiving oxygen therapy should have the oxygen saturation monitored at least as frequently as vital signs. Oxygen is a medication, and it is the responsibility of the practitioner to modify dosage as indicated. Humidified oxygen should not be blown directly into an infant's face.

16. What important consideration in providing atraumatic care should the nurse consider when preforming a venipuncture on a 6-year-old child? a. Use an 18-gauge needle if possible. b. If not successful after four attempts, have another nurse try. c. Restrain the child only as needed to perform venipuncture safely. d. Show the child equipment to be used before procedure.

ANS: C Restrain the child only as needed to perform the procedure safely; an alternative would be the use of therapeutic hugging. Use the smallest gauge needle that permits free flow of blood. A two-try-only policy is desirable, in which two operators each have only two attempts. If insertion is not successful after four punctures, alternative venous access should be considered. Keep all equipment out of sight until used.

A 16-year-old boy with a chronic illness has recently become rebellious and is taking risks such as missing doses of his medication. The nurse should explain to his parents that: a. He needs more discipline. b. He needs more socialization with peers. c. This is part of normal adolescence. d. This is how he is asking for more parental control.

ANS: C Risk taking, rebelliousness, and lack of cooperation are normal parts of adolescence. If the parents increase the amount of discipline, he will most likely be more rebellious. Socialization with peers should be encouraged as a part of adolescence. It is a normal part of adolescence during which the young adult is establishing independence.

A school-age child is diagnosed with a life-threatening illness. The parents want to protect their child from knowing the seriousness of the illness. The nurse should explain that: a. This will help the child cope effectively by denial. b. This attitude is helpful to give parents time to cope. c. Terminally ill children know when they are seriously ill. d. Terminally ill children usually choose not to discuss the seriousness of their illness.

ANS: C The child needs honest and accurate information about the illness, treatments, and prognosis. Children, even at a young age, realize that something is seriously wrong and that it involves them. The nurse should help parents understand the importance of honesty. The child will know that something is wrong because of the increased attention of health professionals. This would interfere with denial as a form of coping. Parents may need professional support and guidance from a nurse or social worker in this process. Children will usually tell others how much information they want about their condition.

5. The nurse is cleaning multiple facial abrasions on a 9-year-old who was brought to the emergency department by his/her mother. When the child begins crying and screaming loudly, what intervention should the nurse implement to best manage this situation? a. Calmly ask the child to be quieter. b. Suggest that his/her mother help the child to relax. c. Tell the child it is okay to cry and scream. d. Suggest that he/she talk to his/her mother as a form of distraction.

ANS: C The child should be allowed to express feelings of anger, anxiety, fear, frustration, or any other emotion. The child needs to know that it is all right to cry. There is no reason for him to be quieter. He is too upset and needs to be able to express his feelings.

1. What should the nurse consider when having consent forms signed for surgery and procedures on children? a. Only a parent or legal guardian can give consent. b. The person giving consent must be at least 18 years old. c. The risks and benefits of a procedure are part of the consent process. d. A mental age of 7 years or older is required for a consent to be considered "informed." \

ANS: C The informed consent must include the nature of the procedure, benefits and risks, and alternatives to the procedure. In special circumstances such as emancipated minors, the consent can be given by someone younger than 18 years without the parent or legal guardian. A mental age of 7 years is too young for consent to be informed.

What is the most appropriate response to a school-age child who asks if she can talk to her dying sister? a. "You need to speak loudly so she can hear you." b. "Holding her hand would be better because at this point she can't hear you." c. "Although she can't hear you, she can feel your presence so sit close to her." d. "Even though she will probably not answer you, she can still hear what you say to her."

ANS: D Hearing is the last sense to cease before death. Talking to the dying child is important both for the child and for the family. There is no evidence that the dying process decreases hearing acuity; therefore, the sister should speak at a normal volume. The sibling should be encouraged to speak to the child, as well as sit close to the bed and hold the child's hand.

23. It is important to make certain that sensory connectors and oximeters are compatible since wiring that is incompatible increases the risk of which injury? a. Hyperthermia b. Electrocution c. Pressure necrosis d. Burns under sensors

ANS: D It is important to make certain that sensor connectors and oximeters are compatible. Wiring that is incompatible can generate considerable heat at the tip of the sensor, causing second- and third-degree burns under the sensor. Incompatibility would cause a local irritation or burn, not hyperthermia. A low voltage is used, which should not present risk of electrocution. Pressure necrosis can occur from the sensor being attached too tightly, but this is not a problem of incompatibility.

36. When administering a gavage feeding to a school-age child, the nurse should implement what intervention to assure safety? a. Lubricate the tip of the feeding tube with Vaseline to facilitate passage. b. Check the placement of the tube by inserting 20 mL of sterile water. c. Administer feedings over 5 to 10 minutes. d. Position the child on the right side after administering the feeding.

ANS: D Position the child with the head elevated about 30 degrees and on the right side or abdomen for at least 1 hour. This is in the same manner as after any infant feeding to minimize the possibility of regurgitation and aspiration. Insert a tube that has been lubricated with sterile water or water-soluble lubricant. With a syringe, inject a small amount of air into the tube, while simultaneously listening with a stethoscope over the stomach area. Feedings should be administered via gravity flow and take from 15 to 30 minutes to complete.

8. What is an appropriate intervention to encourage food and fluid intake in a hospitalized child? a. Force child to eat and drink to combat caloric losses. b. Discourage participation in noneating activities until caloric intake is sufficient. c. Administer large quantities of flavored fluids at frequent intervals and during meals. d. Give high-quality foods and snacks whenever child expresses hunger.

ANS: D Small, frequent meals and nutritious snacks should be provided for the child. Favorite foods such as peanut butter and jelly sandwiches, fruit yogurt, cheese, pizza, macaroni, and cheese should be available. Forcing a child to eat only meets with rebellion and reinforces the behavior as a control mechanism. Large quantities of fluid may decrease the child's hunger and further inhibit food intake.

22. When caring for a child with an intravenous infusion, the nurse should include which intervention in the plan of care? a. Using a macrodropper to facilitate reaching the prescribed flow rate. b. Avoid restraining the child to prevent undue emotional stress. c. Changing the insertion site every 24 hours. d. Observing the insertion site frequently for signs of infiltration.

ANS: D The nursing responsibility for intravenous therapy is to calculate the amount to be infused in a given length of time, set the infusion rate, and monitor the apparatus frequently, at least every 1 to 2 hours, to make certain that the desired rate is maintained, the integrity of the system remains intact, the site remains intact (free of redness, edema, infiltration, or irritation), and the infusion does not stop. A minidropper (60 drops/mL) is the recommended intravenous tubing in pediatrics. The intravenous site should be protected. This may require soft restraints on the child. Insertion sites do not need to be changed every 24 hours unless a problem is found with the site. Frequent change exposes the child to significant trauma.

Nursing interventions to help the siblings of a child with a complex or chronic condition cope include: a. Explaining to the siblings that embarrassment is unhealthy. b. Encouraging the parents not to expect siblings to help them care for the child with special needs. c. Providing information to the siblings about the child's condition only as they request it. d. Suggesting to the parents ways of showing gratitude to the siblings who help care for the child with a disability or chronic condition.

ANS: D The presence of a child with special needs in a family will change the family dynamic. Siblings may be asked to take on additional responsibilities to help the parents to care for the child. The parents should show gratitude, such as an increase in allowance, special privileges, and verbal praise. Embarrassment may be associated with having a sibling with a chronic illness or disability. Parents must be able to respond in an appropriate manner without punishing the sibling. The parents may need assistance with the care of the child. Most siblings are positive about the extra responsibilities. The siblings need to be informed about the child's condition before a non-family member does so. The parents do not want the siblings to fantasize about what is wrong with the child.

The nurse is talking with the parent of a child newly diagnosed with a chronic illness. The parent is upset and tearful. The nurse asks, "With whom do you talk when something is worrying you?" This should be interpreted as: a. Inappropriate, because parent is so upset. b. A diversion of the present crisis to similar situations with which parent has dealt. c. An intervention to find someone to help parent. d. Part of assessing parent's available support system.

ANS: D This question will provide information about the marital relationship (does the parent speak to the spouse?), alternate support systems, and ability to communicate. These are very important data for the nurse to obtain and an appropriate part of an accurate assessment. By assessing these areas, the nurse can facilitate the identification and use of community resources as needed. The nurse is obtaining information to help support the parent through the diagnosis. The parent is not in need of additional parenting help at this time.

1. The nurse is preparing for the admission of an infant who will have several procedures performed. In which situation is informed consent required? (Select all that apply.) a. Catheterized urine collection b. Intravenous (IV) line insertion c. Oxygen administration d. Lumbar puncture e. Bone marrow aspiration

ANS: D, E Informed consent is required for invasive procedures that involve risk to a child, such as a lumbar puncture, chest tube insertion, and bone marrow aspirations. Catheterized urine collection, IV line insertion, and oxygen administration all fall under this category.

B (Traditional psychosocial theory holds that the developmental crises of adolescence lead to the formation of a sense of identity. Intimacy is the developmental stage for early adulthood. Initiative is the developmental stage for early childhood. Independence is not one of Erikson's developmental stages.)

According to Erikson, the psychosocial task of adolescence is developing: a. Intimacy. b. Identity. c. Initiative. d. Independence.

A (Cognitive thinking culminates with capacity for abstract thinking. This stage, the period of formal operations, is Piaget's fourth and last stage. The concrete operations stage usually develops between ages 7 and 11 years. Conventional and postconventional thought refer to Kohlberg's stages of moral development.)

According to Piaget, the adolescent is in the fourth stage of cognitive development, or period of: a. Formal operations. b. Concrete operations. c. Conventional thought. d. Postconventional thought.

A (Formal operations.)

According to Piaget, the adolescent is in the fourth stage of cognitive development, or period of: a. Formal operations. b. Concrete operations. c. Conventional thought. d. Postconventional thought.

C (Acute diarrhea is a sudden increase in frequency and change in consistency of stools and may be associated with antibiotic therapy. Hirschsprung's disease, hypothyroidism, and meconium ileus are usually manifested with constipation rather than diarrhea.)

Acute diarrhea is often caused by: a. Hirschsprung's disease. b. Hypothyroidism. c. Antibiotic therapy. d. Meconium ileus.

B (Most postpartum women often experience a "blue" period 2 days after childbirth, during which women may be emotional and cry for no explainable reason. This is called postpartum blues and is the natural hormonal reaction after giving birth. The nurse may instruct the patient to soak in a tub for 20 minutes on a regular basis to promote relaxation and help the patient cope with the postpartum blues. Postpartum women do not typically have trouble staying warm, so this is not the reason for the nurse's suggestion. Soaking in the tub does not facilitate eye contact, so the nurse would more likely encourage the mother to hold the baby en face position if this were the problem.)

After assessing a postpartum patient 2 days after childbirth, the nurse instructs the patient to soak in a warm water tub for 20 minutes every day. What is the most likely reason for this instruction? The patient: A. Is unable to stay warm. B. Cries easily for no apparent reason. C. Is joyful and has a feeling of well-being. D. Has trouble maintaining direct eye contact with the infant.

B, D, E (Detachment is the third stage of separation anxiety. It is also referred to as the denial stage. In this stage the child begins to take an interest in the surroundings. The child also forms new but superficial relationships with others and becomes more interested in interacting with strangers or familiar caregivers. The child's behavior indicates that the child has finally adjusted to the loss of the parents. This is a serious stage because reversal of the potential adverse effects is less likely to occur after detachment. Refusing to eat, drink, and get out of bed are characteristics of the despair stage of separation anxiety. Attempting to leave the hospital to find the parents is observed in protest stage of separation anxiety.)

After assessment, the nurse notices that a child is in the detachment stage of separation anxiety. Which behavioral changes would the nurse observe in the child? Select all that apply. A. Refuses to eat, drink, or get out of the bed B. Shows an increased interest in the surroundings C. Tries to leave the hospital to find the parents D. Begins to form new relationships with others E. Interacts with strangers or familiar caregivers

D (Biorhythmicity)

After birth, a crying infant may be soothed by being held in a position in which the newborn can hear the mother's heartbeat. This phenomenon is known as: A. Entrainment B. Reciprocity C. Synchrony D. Biorhythmicity

D (Provide time for the woman to bathe her infant after she views an infant bath demonstration)

After giving birth to a healthy infant boy, a primiparous woman, 16, is admitted to the postpartum unit. An appropriate nursing diagnosis for her at this time is "risk for impaired parenting related to deficient knowledge of newborn care." In planning for the woman's discharge, what should the nurse be certain to include in the plan of care? A. Tell the woman how to feed and bathe her infant B. Give the woman written information on bathing her infant C. Advise the woman that all mothers instinctively know how to care for their infants D. Provide time for the woman to bathe her infant after she views an infant bath demonstration

B (Siblings experience loneliness, fear, worry, anger, resentment, jealousy, and guilt. The siblings experience stress equal to that of the hospitalized child. These are not uncommon responses by normal siblings. There is no evidence that the family has maladaptive coping or that the siblings lack understanding.)

Amy, age 6 years, needs to be hospitalized again because of a chronic illness. The clinic nurse overhears her school-age siblings tell her, "We are sick of Mom always sitting with you in the hospital and playing with you. It isn't fair that you get everything and we have to stay with the neighbors." The nurse's best assessment of this situation is that: a. The siblings are immature and probably spoiled. b. Jealousy and resentment are common reactions to the illness or hospitalization of a sibling. c. The family has ineffective coping mechanisms to deal with chronic illness. d. The siblings need to better understand their sister's illness and needs.

A (Antibiotics should be given for their full course to prevent recurrence of infection with resistant bacteria. Symptoms may subside before the full course is given. Hearing loss is a complication of AOM. Antibiotics should continue to be given. Medication may take 24 to 48 hours to make symptoms subside. It should be continued.)

An 18-month-old child is seen in the clinic with AOM. Trimethoprim-sulfamethoxazole (Bactrim) is prescribed. Which statement made by the parent indicates a correct understanding of the instructions? a. "I should administer all the prescribed medication." b. "I should continue medication until the symptoms subside." c. "I will immediately stop giving medication if I notice a change in hearing." d. "I will stop giving medication if fever is still present in 24 hours."

D (These adolescents are at increased risk for health-damaging behaviors, not because of the sexual behavior itself, but because of society's reaction to the behavior. The nurse's first priority is to give the young man permission to discuss his feelings about this topic, knowing that the nurse will maintain confidentially, appreciate his feelings, and remain sensitive to his need to talk it. In recent studies among self-identified gay, lesbian, and bisexual adolescents, many of the adolescents report changing their self-labels one or more times during their adolescence.)

An adolescent boy tells the nurse that he has recently had homosexual feelings. The nurse's response should be based on knowledge that: a. This indicates that the adolescent is homosexual. b. This indicates that the adolescent will become homosexual as an adult. c. The adolescent should be referred for psychotherapy. d. The adolescent should be encouraged to share his feelings and experiences.

D (Notify parents that adolescent needs to see an ophthalmologist.)

An adolescent gets hit in the eye during a fight. The school nurse, using a flashlight, notes the presence of gross hyphema (hemorrhage into anterior chamber). The nurse should: a. Apply a Fox shield. b. Instruct the adolescent to apply ice for 24 hours. c. Have adolescent rest with eye closed and ice applied. d. Notify parents that adolescent needs to see an ophthalmologist.

A (Routine health assessments of adolescents should include questions that assess the presence of suicidal ideation or intent. Questions such as "Have you ever developed a plan to hurt yourself or kill yourself?" should be part of that assessment. Threats of suicide should always be taken seriously and evaluated. Suggesting that the adolescent needs a plan and encouraging her to devise this plan would be inappropriate statements by the nurse.)

An adolescent girl tells the nurse that she has suicidal thoughts. The nurse asks her if she has a specific plan. Asking this should be considered: a. An appropriate part of the assessment. b. Not a critical part of the assessment. c. Suggesting that the adolescent needs a plan. d. Encouraging the adolescent to devise a plan.

D (Myopia)

An adolescent male visits his primary care provider complaining of difficulty with his vision. When the nurse asks the adolescent to explain what visual deficits he is experiencing, the adolescent states, "I am having difficulty seeing distant objects; they are less clear than things that are close." What disorder does the nurse suspect the adolescent has? a. Hyphema b. Astigmatism c. Amblyopia d. Myopia

D (Some of the signs of bulimia include erosion of tooth enamel, increased dental caries from vomited gastric acid, throat complaints, fluid and electrolyte disturbances, and abdominal complaints from laxative abuse. Diarrhea is not a result of the vomiting. It may occur in patients with bulimia who also abuse laxatives. Amenorrhea and cold intolerance are characteristics of anorexia nervosa, which some bulimics have. These symptoms are related to the extreme low weight.)

An adolescent teen has bulimia. Which assessment finding should the nurse expect? a. Diarrhea b. Amenorrhea c. Cold intolerance d. Erosion of tooth enamel

B (A toddler experiences separation anxiety secondary to being separated from the parents. To avoid this, the parents should be encouraged to room in as much as possible. Maintaining routines and ensuring privacy are helpful interventions, but they would not substitute for the parents. Contact with same-aged children would not substitute for having the parents present.)

An appropriate nursing intervention to minimize separation anxiety in a hospitalized toddler is to: a. Provide for privacy. b. Encourage parents to room in. c. Explain procedures and routines. d. Encourage contact with children the same age.

A (Encouraging rest by clustering care and promoting a quiet environment is the best intervention for a child with pneumonia. Lying on the affected side may promote comfort by splinting the chest and reducing pleural rubbing. Analgesics are not indicated. Children should be placed in a semi-erect position or position of comfort.)

An appropriate nursing intervention when caring for a child with pneumonia is to: a. Encourage rest. b. Encourage the child to lie on the unaffected side. c. Administer analgesics. d. Place the child in the Trendelenburg position.

B (The infant needs to receive treatment immediately. Emergency help is called after attempting to remove the obstruction. This is the correct initial sequence of actions for an infant with an obstructed airway. Mouth-to-mouth resuscitation should not be used. This may push the object further into the child's respiratory system. If the child's airway is obstructed, the water will not be able to pass. This will increase the risk of aspiration.)

An immediate intervention when an infant chokes on a piece of food would be to: A. Have infant lie quietly while a call is placed for emergency help. B. Position the infant in a head-down, face-down position and administer five quick blows between the shoulder blades. C. Administer mouth-to-mouth resuscitation. D. Give water by cup to relieve the obstruction.

A (The main consideration is helping the child adhere to dietary management. Considerable time is spent in explaining to the child and parents the disease process, the specific role of gluten in aggravating the condition, and those foods that must be restricted. Referral to a nutritionist would help in this process. The most severe symptoms usually occur in early childhood and adult life. Dietary avoidance of gluten should be lifelong. Celiac disease is not transmissible or stress related.)

An important nursing consideration in the care of a child with celiac disease is to: a. Refer to a nutritionist for detailed dietary instructions and education. b. Help the child and family understand that diet restrictions are usually only temporary. c. Teach proper hand washing and Standard Precautions to prevent disease transmission. d. Suggest ways to cope more effectively with stress to minimize symptoms.

C (These clinical manifestations indicate dehydration. Symptoms of overhydration are edema and weight gain. Regardless of extracellular sodium levels, total body sodium is usually depleted in dehydration. Symptoms of hypocalcemia are a result of neuromuscular irritability and manifest as jitteriness, tetany, tremors, and muscle twitching.)

An infant is brought to the emergency department with poor skin turgor, weight loss, lethargy, and tachycardia. This is suggestive of: a. Overhydration. b. Sodium excess. c. Dehydration. d. Calcium excess.

A (Synagis is a monoclonal antibody specific for RSV. Monthly administration is expected to prevent infection with RSV. The goal of this drug is prevention of RSV. It will not affect the need to isolate the child if RSV develops. The antibody is specific to RSV, not bacterial infection. This will have no effect on antiviral agents.)

An infant with a congenital heart defect is receiving palivizumab (Synagis). The purpose of this is to: A. Prevent respiratory syncytial virus (RSV) infection. B. Make isolation of infant with RSV unnecessary. C. Prevent secondary bacterial infection. D. Decrease toxicity of antiviral agents.

D (Infants with excessive vomiting are prone to metabolic alkalosis from the loss of hydrogen ions. Chloride ions and sodium are lost with vomiting. Metabolic alkalosis, not acidosis, is likely.)

An infant with pyloric stenosis experiences excessive vomiting that can result in: a. Hyperchloremia. b. Metabolic acidosis. c. Hypernatremia. d. Metabolic alkalosis.

B (During TPN therapy, care must be taken to minimize the risk of complications related to the central venous access device, such as catheter infections, occlusions, or accidental removal. This is an important part of family teaching. The prognosis for patients with short bowel syndrome depends in part on the length of residual small intestine. It has improved with advances in TPN. Although parents need to be taught about nutritional needs, the caloric needs and prescribed TPN and rate are the responsibility of the health care team. The tubes should not be placed under the diapers because of risk of infection.)

An infant with short bowel syndrome will be discharged home on total parenteral nutrition (TPN) and gastrostomy feedings. Nursing care should include: a. Preparing the family for impending death. b. Teaching the family signs of central venous catheter infection. c. Teaching the family how to calculate caloric needs. d. Securing TPN and gastrostomy tubing under diaper to lessen risk of dislodgment.

A (Eliminating tobacco smoke from the child's environment is essential for preventing OM and other common childhood illnesses. Nasal decongestants are not useful in preventing OM. Children with uncomplicated OM are not contagious unless they show other upper respiratory infection symptoms. Children should be fed in an upright position to prevent OM.)

An infant's parents ask the nurse about preventing otitis media (OM). What should the nurse recommend? a. Avoid tobacco smoke. b. Use nasal decongestant. c. Avoid children with OM. d. Bottle-feed or breastfeed in supine position.

A, D, E (Language as used in social communication Symbolic or imaginative play Social interaction)

Autism is a complex developmental disorder. The diagnostic criteria for autism include delayed or abnormal functioning in which area(s) with onset before age 3 years (select all that apply)? a. Language as used in social communication b. Gross motor development c. Growth below the 5th percentile for height and weight d. Symbolic or imaginative play e. Social interaction

10. A ten-day postpartum breastfeeding client telephones the postpartum unit complaining of a reddened, painful breast and elevated temperature. Based on assessment of the client's complaints, the nurse tells the client to: A. "Stop breastfeeding because you probably have an infection." B. "Notify your physician because you may need medication." C. "Continue breastfeeding because this is a normal response in breastfeeding mothers." D. "Breastfeed only with the unaffected breast."

B. "Notify your physician because you may need medication." Rationale: Based on the signs and symptoms presented by the client (especially the elevated temperature), the physician should be notified because the client probably has mastitis, an infection in the breast. An antibiotic that is tolerated by the infant as well as the mother may be prescribed. The mother should continue to nurse on both breasts but should start the infant on the unaffected breast while the affected breast lets down.

What woman is at greatest risk for early postpartum hemorrhage? A. A primiparous woman (G 2 P 1 0 0 1) being prepared for an emergency cesarean birth for fetal distress B. A woman with severe preeclampsia on magnesium sulfate whose labor is being induced C. A multiparous woman (G 3 P 2 0 0 2) with an 8-hour labor D. A primigravida in spontaneous labor with preterm twins

B. A woman with severe preeclampsia on magnesium sulfate whose labor is being induced Magnesium sulfate administration during labor poses a risk for PPH. Magnesium acts as a smooth muscle relaxant, thereby contributing to uterine relaxation and atony.

4. The client has just given birth to a healthy, full-term infant. The client is Rho(D) negative and her baby is Rho(D) positive. Which intervention will take place to reduce the possibility of isoimmunization? A. Administering Rho(D) immune globulin to the baby, IM, within 72 hours B. Administering Rho(D) immune globulin to the mother, IM, within 72 hours C. Administering Rho(D) immune globulin to the mother, IM, at her 6-week visit D. Administering Rho(D) immune globulin to the mother, IM, within 3 months

B. Administering Rho(D) immune globulin to the mother, IM, within 72 hours Rho(D) immune globulin (RhoGam) is given to the Rho(D)-negative mother, within 72 hours after delivery of an Rho(D)-positive baby (if the Coombs is negative). RhoGam is never given to the baby.

9. A client gives birth to a stillborn infant at 36 weeks. When caring for this client, which strategy by the nurse would be most helpful? A. Be selective in providing the information that the client seeks B. Encourage the client to see, touch and hold the dead infant C. Provide information about the possible causes of the stillbirth only if the client requests it D. Let the child's father decide what information the mother receives.

B. Encourage the client to see, touch and hold the dead infant 8. Rationale: When caring for a client who has suffered perinatal loss, the nurse should provide opportunity for her to bond with the dead infant and for the infant to become part of the family unit. Parents not given that opportunity may have fantasies about the infant that are worse than reality. If the child has gross deformities, the nurse should prepare the client for these. If the client doesn't ask about her child, the nurse should encourage her to do so and provide any information she seems ready to hear. The client needs a full explanation of all factors related to the experience so she can grieve appropriately. Allowing the father to determine which information the client is given is inappropriate.

5. A client with cardiac disease delivers a baby. Afterwards, the nurse assesses the client for signs of cardiac decompensation. During the postpartum period, which condition can cause cardiac decompensation? A. Increased pain B. Increased cardiac output C. Decreased renal function D. Decreased hepatic blood flow

B. Increased cardiac output Rationale: Cardiac output increases immediately after delivery as blood that had been diverted to the uterus reenters the central circulation. A client who cannot tolerate these changes may experience cardiac decompensation and cardiac failure. After delivery, renal function increases. There is usually not an increase in pain after delivery except for small increments attributable to uterine cramps, perineal discomfort and breast tenderness. Although hepatic blood flow decreases to normal levels after delivery, this does not affect cardiac function.

The perinatal nurse assisting with establishing lactation is aware that acute mastitis can be minimized by: A. Washing the nipples and breasts with mild soap and water once a day B. Using proper breastfeeding techniques C. Wearing a nipple shield for the first few days of breastfeeding D. Wearing a supportive bra 24 hours a day

B. Using proper breastfeeding techniques Almost all instances of acute mastitis can be avoided by proper breastfeeding technique to prevent cracked nipples.

Nurses should first look for the most common cause of PPH, _____, by _____. A. Lacerations of the genital tract; checking for the source of blood B. Uterine atony; evaluating the contractility of the uterus C. Inversion of the uterus; feeling for a smooth mass through the dilated cervix D. Retained placenta; noting the type of bleeding

B. Uterine atony; evaluating the contractility of the uterus The leading cause of PPH is uterine atony, which complicates one in 20 births. The uterus is overstretched and contracts poorly after the birth.

D (When a child is hospitalized, the altered family role, physical disability, loss of peer acceptance, lack of productivity, and inability to cope with stress usurp individual power and identity. This is especially detrimental to school-age children, who are striving for independence and productivity and are now experiencing events that lessen their control and power. Infants, toddlers, and preschoolers, although affected by loss of power, are not as significantly affected as are school-age children.)

Because of their striving for independence and productivity, which age-group of children is particularly vulnerable to events that may lessen their feeling of control and power? a. Infants b. Toddlers c. Preschoolers d. School-age children

D (Concerns about pubertal delay should be considered for boys who exhibit no enlargement of the testes or scrotal changes by 13.5 to 14 years of age. Ages younger than 13.5 years are too young for initial concern.)

By what age should concerns about pubertal delay be considered in boys? a. 12 to 12.5 years b. 12.5 to 13 years c. 13 to 13.5 years d. 13.5 to 14 years

D (13.5 to 14 years)

By what age should concerns about pubertal delay be considered in boys? a. 12 to 12.5 years b. 12.5 to 13 years c. 13 to 13.5 years d. 13.5 to 14 years

Which nursing assessment indicates that a woman who is in second-stage labor is almost ready to give birth? a. The fetal head is felt at 0 station during vaginal examination. b. Bloody mucus discharge increases. c. The vulva bulges and encircles the fetal head. d. The membranes rupture during a contraction.

C (A bulging vulva that encircles the fetal head describes crowning, which occurs shortly before birth. Birth of the head occurs when the station is +4. A 0 station indicates engagement. Bloody show occurs throughout the labor process and is not an indication of an imminent birth. Rupture of membranes can occur at any time during the labor process and does not indicate an imminent birth.)

The nurse thoroughly dries the infant immediately after birth primarily to: a. Stimulate crying and lung expansion. b. Remove maternal blood from the skin surface. c. Reduce heat loss from evaporation. d. Increase blood supply to the hands and feet.

C (Infants are wet with amniotic fluid and blood at birth, and this accelerates evaporative heat loss. The primary purpose of drying the infant is to prevent heat loss. Rubbing the infant does stimulate crying; however, it is not the main reason for drying the infant. This process does not remove all the maternal blood.)

Despite popular belief, there is a rare type of hemophilia that affects women of childbearing age. Von Willebrand disease is the most common of the hereditary bleeding disorders and can affect males and females alike. It results from a factor VIII deficiency and platelet dysfunction. Although factor VIII levels increase naturally during pregnancy, there is an increased risk for postpartum hemorrhage from birth until 4 weeks postdelivery as levels of von Willebrand factor (vWf) and factor VIII decrease. The treatment that should be considered first for the client with von Willebrand disease who experiences a postpartum hemorrhage is: A. Cryoprecipitate B. Factor VIII and vWf C. Desmopressin D. Hemabate

C. Desmopressin Desmopressin is the primary treatment of choice. This hormone can be administered orally, nasally, and intravenously. This medication promotes the release of factor VIII and vWf from storage.

What infection is contracted mostly by first-time mothers who are breastfeeding? A. Endometritis B. Wound infections C. Mastitis D. Urinary tract infections

C. Mastitis Mastitis is infection in a breast, usually confined to a milk duct. Most women who suffer this are first-timers who are breastfeeding.

2. During the early postpartum period, the nurse is evaluating a client's attachment to her neonate. Which type of parent has the most difficulty attaching to her newborn? A. One who has little knowledge of parent-infant attachment B. One who recently lost a job C. One whose father recently died D. One who is an only child

C. One whose father recently died A parent that is grieving over a recent loss (in the process of detachment) will have the most difficulty bonding with the new baby. Knowledge of parent-infant attachment or being an only child are not related to successful bonding. A job loss does not have the impact that death of a family member does.

7. A woman is experiencing an early postpartum hemorrhage. Which of the following actions would be inappropriate? A. Insertion of an indwelling urinary catheter B. Fundal massage C. Pad count D. Administration of oxytocics

C. Pad count Rationale: Since the client is already hemorrhaging, it is inappropriate to initiate a pad count. Fundal massage and administration of oxytocics would be indicated if the hemorrhage is due to uterine atony. If a full bladder is displacing the uterus and preventing it from contracting, insertion of an indwelling catheter would be an appropriate response.

The first and most important nursing intervention when a nurse observes profuse postpartum bleeding is to: A. Call the woman's primary health care provider B. Administer the standing order for an oxytocic C. Palpate the uterus and massage it if it is boggy D. Assess maternal blood pressure and pulse for signs of hypovolemic shock

C. Palpate the uterus and massage it if it is boggy The initial management of excessive postpartum bleeding is firm massage of the uterine fundus.

The most effective and least expensive treatment of puerperal infection is prevention. What is important in this strategy? A. Large doses of vitamin C during pregnancy B. Prophylactic antibiotics C. Strict aseptic technique, including handwashing, by all health care personnel D. Limited protein and fat intake

C. Strict aseptic technique, including handwashing, by all health care personnel Strict adherence by all health care personnel to aseptic techniques during childbirth and the postpartum period is very important and the least expensive measure to prevent infection.

B (In a toddler, the carotid pulse is palpated. The radial pulse is not considered a central pulse. The femoral pulse is not the most central and accessible. The brachial pulse is felt in infants younger than 1 year.)

Cardiopulmonary resuscitation is begun on a toddler. Which pulse is usually palpated because it is the most central and accessible? a. Radial b. Carotid c. Femoral d. Brachial

D (Hepatitis A is spread person to person, by the fecal-oral route, and through contaminated food or water. Good hand washing is critical in preventing its spread. The virus can survive on contaminated objects for weeks. Irritable bowel syndrome is the result of increased intestinal motility and is not contagious. Ulcerative colitis is not infectious. Cirrhosis is not infectious.)

Careful hand washing before and after contact can prevent the spread of which condition in day care and school settings? a. Irritable bowel syndrome b. Hepatic cirrhosis c. Ulcerative colitis d. Hepatitis A

D (Using special or modified nipples for feeding techniques helps to meet the infant's sucking needs. Gastrostomy feedings are usually not indicated. Feeding is best accomplished with the infant's head in an upright position. The child requires both nutritive and nonnutritive sucking.)

Caring for the newborn with a cleft lip and palate before surgical repair includes: a. Gastrostomy feedings. b. Keeping the infant in near-horizontal position during feedings. c. Allowing little or no sucking. d. Providing satisfaction of sucking needs.

D (OME is characterized by an immobile or orange-discolored tympanic membrane and nonspecific complaints and does not cause severe pain. Fever and severe pain may be signs of AOM. Nausea and vomiting are associated with otitis media.)

Chronic otitis media with effusion (OME) is differentiated from acute otitis media (AOM) because it is usually characterized by: a. Fever as high as 40° C (104° F). b. Severe pain in the ear. c. Nausea and vomiting. d. A feeling of fullness in the ear.

B (Constipation may be associated with drugs such as antihistamines, antacids, diuretics, opioids, antiepileptics, and iron. Because this is the only known recent change in her habits, the addition of antihistamines is most likely the etiology of the diarrhea, rather than diet, allergies, or emotional factors. With a change in bowel habits, the presence and role of any recently prescribed medications should be assessed.)

Constipation has recently become a problem for a school-age girl. She is healthy except for seasonal allergies, which are now being successfully treated with antihistamines. The nurse should suspect that the constipation is most likely caused by: a. Diet. b. Antihistamines. c. Allergies. d. Emotional factors.

D (Thick mucous secretions are the probable cause of the multiple body system involvement. There is an identified autonomic nervous system anomaly, but it is not hypoactivity. The sweat glands are not hyperactive. The child loses a greater amount of salt because of abnormal chloride movement. Children with cystic fibrosis have thick mucous gland secretions. The viscous secretions obstruct small passages in organs such as the pancreas.)

Cystic fibrosis may affect singular or multiple systems of the body. The primary factor responsible for possible multiple clinical manifestations is: A. Atrophic changes in the mucosal wall of intestines. B. Hypoactivity of the autonomic nervous system. C. Hyperactivity of the sweat glands. D. Mechanical obstruction caused by increased viscosity of mucous gland secretions.

A primigravida at 39 weeks of gestation is observed for 2 hours in the intrapartum unit. The fetal heart rate has been normal. Contractions are 5 to 9 minutes apart, 20 to 30 seconds in duration, and of mild intensity. Cervical dilation is 1 to 2 cm and uneffaced (unchanged from admission). Membranes are intact. The nurse should expect the woman to be: a. Admitted and prepared for a cesarean birth. b. Admitted for extended observation. c. Discharged home with a sedative. d. Discharged home to await the onset of true labor.

D (This situation describes a woman with normal assessments who is probably in false labor and will likely not deliver rapidly once true labor begins. There is no indication that further assessments or observations are indicated; therefore, the patient will be discharged along with instructions to return when contractions increase in intensity and frequency. Neither a cesarean birth nor a sedative is required at this time.)

1. The clinic nurse is caring for a woman who is suspected of developing postpartum psychosis. Which of the following statements characterizes this disorder: A. Symptoms start within several days of delivery B. The disorder is common in postpartum women C. Suicide and infanticide are uncommon in this disorder D. Delusions and hallucinations accompany this disorder

D. Delusions and hallucinations accompany this disorder Rationale: A woman that develops postpartum psychosis usually does so within four weeks of delivery. Only 1% of women develop this disorder. Suicide and infanticide are common and the disorder is considered a medical emergency. Delusions and hallucinations accompany the disorder and the woman usually has a past history of a psychiatric disorder and treatment.

It is important for the perinatal nurse to be knowledgeable regarding conditions of abnormal adherence of the placenta. This occurs when the zygote implants in an area of defective endometrium and results in little to no zone separation between the placenta and decidua. Which classification of separation is not recognized as an abnormal adherence pattern? A. Placenta accreta B. Placenta increta C. Placenta percreta D. Placenta abruptio

D. Placenta abruptio Placenta abruptio is premature separation of the placenta as opposed to partial or complete adherence. This occurs between the 20th week of gestation and delivery in the area of the decidua basilis. Symptoms include localized pain and bleeding.

One of the first symptoms of puerperal infection to assess for in the postpartum woman is: A. Fatigue continuing for longer than 1 week B. Pain with voiding C. Profuse vaginal bleeding with ambulation D. Temperature of 38° C (100.4° F) or higher on 2 successive days starting 24 hours after birth

D. Temperature of 38° C (100.4° F) or higher on 2 successive days starting 24 hours after birth Postpartum or puerperal infection is any clinical infection of the genital canal that occurs within 28 days after miscarriage, induced abortion, or childbirth. The definition used in the United States continues to be the presence of a fever of 38° C (100.4° F) or higher on 2 successive days of the first 10 postpartum days, starting 24 hours after birth.

Nurses need to know the basic definitions and incidence data about postpartum hemorrhage. For instance: A. PPH is easy to recognize early; after all, the woman is bleeding. B. Traditionally, it takes more than 1000 ml of blood after vaginal birth and 2500 ml after cesarean birth to define the condition as PPH. C. If anything, nurses and doctors tend to overestimate the amount of blood loss. D. Traditionally, PPH has been classified as early or late with respect to birth.

D. Traditionally, PPH has been classified as early or late with respect to birth. Early PPH is also known as primary, or acute, PPH; late PPH is known as secondary PPH.

When caring for a postpartum woman experiencing hemorrhagic shock, the nurse recognizes that the most objective and least invasive assessment of adequate organ perfusion and oxygenation is: A. Absence of cyanosis in the buccal mucosa B. Cool, dry skin C. Diminished restlessness D. Urinary output of at least 30 ml/hr

D. Urinary output of at least 30 ml/hr Hemorrhage may result in hemorrhagic shock. Shock is an emergency situation in which the perfusion of body organs may become severely compromised, and death may occur. The presence of adequate urinary output indicates adequate tissue perfusion.

A (Vasoconstrictive nose drops such as Neo-Synephrine should not be used for more than 3 days to avoid rebound congestion. Drops should be discarded after one illness because they may become contaminated with bacteria. Vasoconstrictive nose drops can have a rebound effect after 3 days of use. Drops administered before feedings are more helpful.)

Decongestant nose drops are recommended for a 10-month-old infant with an upper respiratory tract infection. Instructions for nose drops should include: a. Avoiding use for more than 3 days. b. Keeping drops to use again for nasal congestion. c. Administering drops until nasal congestion subsides. d. Administering drops after feedings and at bedtime.

C (Postpartum blues)

During a phone follow-up conversation with a woman who is 4 days postpartum, the woman tells the nurse, "I don't know what's wrong. I love my son, but I feel so let down. I seem to cry for no reason!" The nurse would recognize that the woman is experiencing: A. Taking-in B. Postpartum depression (PPD) C. Postpartum blues D. Attachment difficulty

C (From this assessment, the nurse interprets that the child is in the despair stage. This is the second stage of separation anxiety. In the despair stage, the child appears less active, depressed, and uninterested in play or food. In this stage the child's physical condition may deteriorate from refusing to eat, drink, or get out of bed. The denial stage is the third stage of separation anxiety. In the denial stage, the child is more interested in the surroundings, plays with others, and forms new but superficial relationships with others. In the protest stage, the child reacts aggressively, cries, screams, and searches for the parents with the eyes. Detachment is the third stage of separation anxiety. It is also called denial.)

During the assessment of a child, the nurse finds that the child is inactive, depressed, sad, and uncommunicative; refuses to eat; and generally lacks interest in everything around her. What should the nurse interpret from this assessment? The child is in the: A. Denial stage. B. Protest stage. C. Despair stage. D. Detachment stage

C (He may be experiencing detachment, which is the third stage of separation anxiety.)

During the first 4 days of hospitalization, Eric, age 18 months, cried inconsolably when his parents left him, and he refused the staff's attention. Now the nurse observes that Eric appears to be "settled in" and unconcerned about seeing his parents. The nurse should interpret this as which of the following? a. He has successfully adjusted to the hospital environment. b. He has transferred his trust to the nursing staff. c. He may be experiencing detachment, which is the third stage of separation anxiety. d. Because he is "at home" in the hospital now, seeing his mother frequently will only start the cycle again.

B, D (After the baby's birth, the patient feels irritated and stressed due to lack of sleep. The nurse should suggest interventions that help the patient rest. For example, if the patient's family can be called upon to help with the household chores while the patient looks after the baby and rests, this can help the patient feel more relaxed in the early days of parenting. The patient should try to adjust her schedule to nap when the baby sleeps if possible. Installing bright lights is not an effective technique, as it does not promote sleep. Organizing a dinner party may increase the number of visitors and guests in the home, which, in turn, can increase the household work, causing the patient to become even more fatigued and tired. Coffee contains caffeine and may cause insomnia by stimulating the central nervous system (CNS). Therefore, drinking coffee 3 times a day does not support relaxation.)

During the postnatal visit, the patient tells the nurse, "I'm feeling irritated and tired. I haven't slept properly for days." The nurse suggests a few interventions to help the patient with relaxation and rest. Which techniques adopted by the patient indicate effective learning? Select all that apply. A. Installed bright lights in the room B. Asked a family member to stay overnight C. Organized a small dinner party at her home D. Takes a nap when the baby is sleeping E. Started drinking coffee 3 times a day

C (Parents should bring favorite items from home to be with the child. Young children associate inanimate objects with significant people; they gain comfort and reassurance from these items. New toys will not serve the purpose of familiar toys and objects from home. The parents may experience some guilt as a response to the hospitalization, but there is no evidence that it is maladaptive.)

Emma, age 3 years, is being admitted for about 1 week of hospitalization. Her parents tell the nurse that they are going to buy her "a lot of new toys because she will be in the hospital." The nurse's reply should be based on an understanding that: a. New toys make hospitalization easier. b. New toys are usually better than older ones for children of this age. c. At this age children often need the comfort and reassurance of familiar toys from home. d. Buying new toys for a hospitalized child is a maladaptive way to cope with parental guilt.

C (Atresia is suspected when a nasogastric tube fails to pass 10 to 11 cm beyond the gum line. Abdominal radiographs will confirm the diagnosis. Prenatal radiographs do not provide a definitive diagnosis. The defect is not externally visible. Bronchoscopy and endoscopy can be used to identify this defect. Infants with esophageal atresia may have been born prematurely and with a low birth weight, but neither is suggestive of the presence of an esophageal atresia.)

For what clinical manifestation should a nurse be alert when suspecting a diagnosis of esophageal atresia? a. A radiograph in the prenatal period indicates abnormal development. b. It is visually identified at the time of delivery. c. A nasogastric tube fails to pass at birth. d. The infant has a low birth weight.

D (Mothers need to reestablish their own well-being to effectively care for their baby. The woman should not be told what to do and needs to care for her own well-being. Because the woman needs to rest does not indicate ineffective maternal-newborn attachment. The behavior described is typical of this stage and is not a reflection of ineffective attachment unless it persists. Test-Taking Tip: Be alert for grammatical inconsistencies. If the response is intended to complete the stem (an incomplete sentence) but makes no grammatical sense to you, it might be a distractor rather than the correct response. Question writers typically try to eliminate these inconsistencies.)

Four hours after a difficult labor and birth, a primiparous woman refuses to feed her baby, stating that she is too tired and just wants to sleep. The nurse should: A. Tell the woman she can rest after she feeds her baby. B. Recognize this as a behavior of the taking-hold stage. C. Record the behavior as ineffective maternal-newborn attachment. D. Take the baby back to the nursery, reassuring the woman that her rest is a priority at this time.

D (In most girls, the initial indication of puberty is the appearance of breast buds, an event known as the larche. The usual sequence of secondary sexual characteristic development in girls is breast changes, rapid increase in height and weight, growth of pubic hair, appearance of axillary hair, menstruation, and abrupt deceleration of linear growth.)

In girls, the initial indication of puberty is: a. Menarche. b. Growth spurt. c. Growth of pubic hair. d. Breast development.

C (After children have taken antibiotics for 24 hours, even if the sore throat persists, they are no longer contagious to other children. Complications may take days to weeks to develop.)

It is generally recommended that a child with acute streptococcal pharyngitis can return to school: a. When the sore throat is better. b. If no complications develop. c. After taking antibiotics for 24 hours. d. After taking antibiotics for 3 days.

D (Otitis media and diabetes insipidus are not sequelae to GABHS. Otitis media and diabetes insipidus are not sequelae to GABHS. Children are at risk for glomerulonephritis, not nephritic syndrome. Children with Group A ß-hemolytic streptococci (GABHS) infection are at risk for acute rheumatic fever and acute glomerulonephritis.)

It is important that a child with Group A ß-hemolytic streptococci (GABHS) infection be treated with antibiotics to prevent: A. Otitis media. B. Diabetes insipidus. C. Nephrotic syndrome. D. Acute rheumatic fever.

A (School-age children need to have control of their environment. The nurse should offer explanations or prepare the child for experiences that are unavoidable. The nurse should refer to the child by the preferred name. Telling the child about all of the limitations of visiting does not help her adjust to the hospital. At the age of 8 years, the child and parents should be oriented to the environment.)

Latasha, age 8 years, is being admitted to the hospital from the emergency department with an injury from falling off her bicycle. What will help her most in her adjustment to the hospital? a. Explain hospital schedules such as mealtimes. b. Use terms such as "honey" and "dear" to show a caring attitude. c. Explain when parents can visit and why siblings cannot come to see her. d. Orient her parents, because she is young, to her room and hospital facility.

B ("Grandparents can help you with parenting skills and also help preserve family traditions.")

Many first-time parents do not plan on their parents' help immediately after the newborn arrives. What statement by the nurse is the most appropriate when counseling new parents about the involvement of grandparents? A. "You should tell your parents to leave you alone." B. "Grandparents can help you with parenting skills and also help preserve family traditions." C. "Grandparent involvement can be very disruptive to the family." D. "They are getting old. You should let them be involved while they can."

B (Children with Down syndrome have the same need for socialization as other children.)

Mark, a 9-year-old with Down syndrome, is mainstreamed into a regular third-grade class for part of the school day. His mother asks the school nurse about programs such as Cub Scouts that he might join. The nurse's recommendation should be based on knowing that: a. Programs such as Cub Scouts are inappropriate for children who are cognitively impaired. b. Children with Down syndrome have the same need for socialization as other children. c. Children with Down syndrome socialize better with children who have similar disabilities. d. Parents of children with Down syndrome encourage programs such as scouting because they deny that their children have disabilities.

A (Attachment, or bonding, is a process that occurs over time and does not require early contact.)

New parents express concern that because of the mother's emergency cesarean birth under general anesthesia, they did not have the opportunity to hold and bond with their daughter immediately after her birth. The nurse's response should convey to the parents that: A. Attachment, or bonding, is a process that occurs over time and does not require early contact. B. The time immediately after birth is a critical period for humans. C. Early contact is essential for optimum parent-infant relationships. D. They should just be happy that the infant is healthy.

A (Fever leads to great insensible fluid loss in young children because of increased body surface area relative to fluid volume. Respiratory rate influences insensible fluid loss and should be monitored in the mechanically ventilated child. Congestive heart failure is a case of fluid overload in children. ICP does not lead to increased fluid requirements in children.)

Nurses must be alert for increased fluid requirements when a child has: a. Fever. b. Congestive heart failure. c. Mechanical ventilation. d. Increased intracranial pressure (ICP).

D (An environment that fosters as much privacy as possible should be created)

Nursing activities that promote parent-infant attachment are many and varied. One activity that should not be overlooked is the management of the environment. While providing routine mother-baby care, the nurse should ensure that: A. The baby is able to return to the nursery at night so that the new mother can sleep B. Routine times for care are established to reassure the parents C. The father should be encouraged to go home at night to prepare for mother-baby discharge D. An environment that fosters as much privacy as possible should be created

C (Adolescent mothers have a higher documented incidence of child abuse.)

Of the many factors that influence parental responses, nurses should be aware that all of these statements regarding age are true except: A. An adolescent mother's egocentricity and unmet developmental needs interfere with her ability to parent effectively. B. An adolescent mother is likely to use less verbal instruction, be less responsive, and interact less positively than other mothers. C. Adolescent mothers have a higher documented incidence of child abuse. D. Mothers older than 35 often deal with more stress related to work and career issues, as well as decreasing libido.

A (Children in this age-group still fear that their insides may leak out at the injection site, even if the bleeding has stopped. Provide the Band-Aid. No explanation should be required.)

Olivia, age 5 years, tells the nurse that she "needs a Band-Aid" where she had an injection. The best nursing action is to: a. Apply a Band-Aid. b. Ask her why she wants a Band-Aid. c. Explain why a Band-Aid is not needed. d. Show her that the bleeding has already stopped.

D (The infection affects the asthma, not the medications. Exercise-induced asthma is caused by vigorous activity. Sensitivity to allergens is independent of respiratory infection. Respiratory infections can trigger an asthmatic attack. Annual influenza vaccine is recommended. All respiratory equipment should be kept clean.)

One of the goals for children with asthma is to prevent respiratory infection. This is because respiratory infection: A. Lessens effectiveness of medications. B. Encourages exercise-induced asthma. C. Increases sensitivity to allergens. D. Can trigger an episode or aggravate an asthmatic state.

B (Infants can learn to distinguish their mother's voice from others soon after birth.)

Other early sensual contacts between infant and mother involve sound and smell. Nurses should be aware that despite what folk wisdom might say: A. High-pitched voices irritate newborns. B. Infants can learn to distinguish their mother's voice from others soon after birth. C. All babies in the hospital smell alike. D. A mother's breast milk has no distinctive odor.

D (Enzymes may be administered in a small amount of cereal or fruit or swallowed whole at the beginning of a meal, not between meals. Pancreatic enzymes are not contraindicated with antibiotics. The dose of enzymes should be increased if the child is having frequent, bulky stools.)

Pancreatic enzymes are administered to the child with cystic fibrosis. Nursing considerations should include: a. Do not administer pancreatic enzymes if the child is receiving antibiotics. b. Decrease dose of pancreatic enzymes if the child is having frequent, bulky stools. c. Administer pancreatic enzymes between meals if at all possible. d. Pancreatic enzymes can be swallowed whole or sprinkled on a small amount of food taken at the beginning of a meal.

A (Having the sibling make or choose a gift for the new baby helps to make the child feel a part of the process. Special time should be set aside just for the other children without interruption from the newborn. Someone other than the mother should carry the baby into the home so she can give full attention to greeting her other children. Children should be actively involved in the care of the baby according to their ability without overwhelming them.)

Parents can facilitate the adjustment of their other children to a new baby by: A. Having the children choose or make a gift to give to the new baby on its arrival home. B. Emphasizing activities that keep the new baby and other children together. C. Having the mother carry the new baby into the home so she can show him or her to the other children. D. Reducing stress on other children by limiting their involvement in the care of the new baby.

D (Parents have understood the teaching about preventing childhood otitis media if they respond they will keep childhood immunizations up to date. The child should be maintained upright during feedings and after. Otitis media can be prevented by exclusively breastfeeding until at least 6 months of age. Propping bottles is discouraged to avoid pooling of milk while the child is in the supine position.)

Parents have understood teaching about prevention of childhood otitis media if they make which statement? a. "We will only prop the bottle during the daytime feedings." b. "Breastfeeding will be discontinued after 4 months of age." c. "We will place the child flat right after feedings." d. "We will be sure to keep immunizations up to date."

A, C, D, E (a. Peer-led education and support. c. Programs including the media. d. School-based programs. e. Information on the immediate effects of smoking.)

Research has shown that the most successful smoking cessation programs among teens include (select all that apply): a. Peer-led education and support. b. Information on the long-term effects of smoking. c. Programs including the media. d. School-based programs. e. Information on the immediate effects of smoking.

A, C, E (Unfamiliar environment Strange smells Inadequate knowledge of condition and routine)

Ryan has just been unexpectedly admitted to the intensive care unit after abdominal surgery. The nursing staff has completed the admission process, and Ryan's condition is beginning to stabilize. When speaking with the parents, the nurses should expect which stressors to be evident (select all that apply)? a. Unfamiliar environment b. Usual day-night routine c. Strange smells d. Provision of privacy e. Inadequate knowledge of condition and routine

D (Children who reside in high prevalence regions for tuberculosis should be tested every 2 to 3 years. Annual testing is not necessary. Testing is not necessary unless exposure is likely or an underlying medical risk factor is present.)

Skin testing for tuberculosis (the Mantoux test) is recommended: a. Every year for all children older than 2 years. b. Every year for all children older than 10 years. c. Every 2 years for all children starting at age 1 year. d. Periodically for children who reside in high-prevalence regions.

D (The apocrine sweat glands, nonfunctional in children, reach secretory capacity during puberty. They secrete a thick substance as a result of emotional stimulation that, when acted on by surface bacteria, becomes highly odoriferous. They are limited in distribution and grow in conjunction with hair follicles in the axillae, genital and anal areas, and other areas. Eccrine sweat glands are present almost everywhere on the skin and become fully functional and respond to emotional and thermal stimulation. Sebaceous glands become extremely active at this time, especially those on the genitals and the "flush" areas of the body, such as face, neck, shoulders, upper back, and chest. This increased activity is important in the development of acne. New deposits of fatty tissue insulate the body and cause increased sweat production, but this is not the etiology of apocrine sweat gland activity.)

Steve, 14 years old, mentions that he now has to use deodorant but never had to before. The nurse's response should be based on knowledge that: a. Eccrine sweat glands in the axillae become fully functional during puberty. b. Sebaceous glands become extremely active during puberty. c. New deposits of fatty tissue insulate the body and cause increased sweat production. d. Apocrine sweat glands reach secretory capacity during puberty.

A (The only successful treatment for end-stage liver disease and liver failure may be liver transplantation, which has improved the prognosis for many children with cirrhosis. Liver transplantation has revolutionized the approach to cirrhosis. Liver failure and cirrhosis are indications for transplantation. Liver transplantation reflects the failure of other medical and surgical measures, such as treatment with corticosteroids or immune globulin and nutritional support, to prevent or treat cirrhosis.)

The best chance of survival for a child with cirrhosis is: a. Liver transplantation. b. Treatment with immune globulin. c. Treatment with corticosteroids. d. Provision of nutritional support.

C (Atlantoaxial instability)

The child with Down syndrome should be evaluated for what characteristic before participating in some sports? a. Hyperflexibility b. Cutis marmorata c. Atlantoaxial instability d. Speckling of iris (Brushfield's spots)

D (The diagnosis of cognitive impairment includes subaverage intellectual functioning and deficits in adaptive skills, including an onset before age 18. IQ is only one component of the diagnosis of cognitive impairment. The onset of the deficit must be before age 18 to meet the diagnosis of cognitive impairment.)

The diagnosis of cognitive impairment is based on the presence of: A. intelligence quotient (IQ) of 75 or less. B. IQ of 70 or less. C. subaverage intellectual functioning, deficits in adaptive skills, and onset at any age. D. subaverage intellectual functioning, deficits in adaptive skills, and onset before 18 years of age.

A (Jaundice is the earliest and most striking manifestation of biliary atresia. It is first observed in the sclera and may be present at birth, but is usually not apparent until ages 2 to 3 weeks. Vomiting is not associated with biliary atresia. Hepatomegaly and abdominal distention are common but occur later. Stools are large and lighter in color than expected because of the lack of bile.)

The earliest clinical manifestation of biliary atresia is: a. Jaundice. b. Hepatomegaly. c. Vomiting. d. Absence of stooling.

A (The earliest clinical manifestation of CF is a meconium ileus, which is found in about 10% of children with CF. Clinical manifestations include abdominal distention, vomiting, failure to pass stools, and rapid development of dehydration. History of malabsorption is a later sign that manifests as failure to thrive. Foul-smelling stools and recurrent respiratory infections are later manifestations of CF.)

The earliest recognizable clinical manifestation of cystic fibrosis (CF) is: a. Meconium ileus. b. History of poor intestinal absorption. c. Foul-smelling, frothy, greasy stools. d. Recurrent pneumonia and lung infections.

C (PPD can easily go undetected)

The early postpartum period is a time of emotional and physical vulnerability. Many mothers can easily become psychologically overwhelmed by the reality of their new parental responsibilities. Fatigue compounds these issues. Although the baby blues are a common occurrence in the postpartum period, about one-half million women in America experience a more severe syndrome known as postpartum depression (PPD). Which statement regarding PPD is essential for the nurse to be aware of when attempting to formulate a nursing diagnosis? a) PPD symptoms are consistently severe b) This syndrome affects only new members c) PPD can easily go undetected d) Only mental health professionals should teach new parents about this condition

A (Safety)

The major consideration when selecting toys for a child who is cognitively impaired is: a. Safety. b. Age appropriateness. c. Ability to provide exercise. d. Ability to teach useful skills.

B (12.5 years)

The mean age of menarche in the United States is: a. 11.5 years b. 12.5 years c. 13.5 years d. 14 years

B (The average age of menarche is 12 years and 4 months in North American girls, with a normal range of 10.5 to 15 years.)

The mean age of menarche in the United States is: a. 11.5 years b. 12.5 years c. 13.5 years d. 14 years

D (The leading cause of all adolescent deaths in the United States is motor vehicle accidents. Drownings, firearms, and drug overdoses are major concerns in adolescence but do not cause the majority of deaths.)

The most common cause of death in the adolescent age-group involves: a. Drownings. b. Firearms. c. Drug overdoses. d. Motor vehicles.

A (Conductive.)

The most common type of hearing loss, which results from interference of transmission of sound to the middle ear, is called: a. Conductive. b. Sensorineural. c. Mixed conductive-sensorineural. d. Central auditory imperceptive.

B (The child does not have a temperature to manage. Because the child is not having difficulty breathing, the nurse should teach the parents the signs of respiratory distress and tell them to come to the emergency room if they develop. Cool mist is recommended to provide relief. Cough suppressants are not indicated. This is characteristic of laryngotracheobronchitis, not epiglottitis.)

The mother of a 20-month-old boy tells the nurse that he has a barking cough at night. His temperature is 37° C. The nurse suspects croup and should recommend: A. Controlling fever with acetaminophen and calling if the cough gets worse during the night. B. Trying a cool-mist vaporizer at night and watching for signs of difficulty breathing. C. Trying over-the-counter cough medicine and coming to the clinic in the morning if there is no improvement. D. Admitting to the hospital and observing for impending epiglottitis.

A (The most likely cause of the infant's vomiting is insufficient attachment between infant and parent. Therefore the nurse should advise the parent to maintain skin contact with the child while feeding. To accomplish this, the nurse should teach the mother kangaroo care, which promotes attachment between the infant and the mother. Breast milk provides optimal nutrition to the baby. If the patient is able to breastfeed the baby, the nurse should encourage the patient to continue breastfeeding rather than switching to formula or cow's milk. The nurse's primary goal should be to improve the attachment between the infant and mother so that the nurse does not need to involve other family members for feeding unless absolutely necessary.)

The mother of a newborn reports that the child vomits milk immediately after breastfeeding. The infant's laboratory report does not show any abnormalities. What is the most appropriate nursing intervention in this situation? A. Teach the mother kangaroo care. B. Ask the mother to give the child formula. C. Ask the mother to feed the child cow's milk. D. Teach other family members to feed the baby

B (Assessment of an infant's response to auditory stimulation is used to detect a conductive hearing impairment in the newborn. Because the nurse suspects hearing impairment, the infant might have failed to respond to auditory stimulus. Eye movement following a flashlight helps in assessing the infant's vision. Vocal expressions or sounds produced in response to communication may indicate whether the infant has a speech or hearing impairment. Assessment of infant's physical activity toward a large moving object is useful to assess the child's muscle coordination. It also tests the infant's visual abilities.)

The nurse assesses an infant at birth for height, weight, and other vital signs. What should the nurse include in the assessment to identify a conductive hearing disorder? The nurse assesses: A. To see whether the infant's eyes move toward a flashlight. B. The infant's response to an auditory stimulus. C. The infant's vocal expressions during vocal communication. D. The infant's physical activity toward a large moving object.

C (The child has fear of sleeping alone in the room. This information is usually noted under the self-perception—self-concept pattern of the nursing history. The nurse tries to comfort the child's fear of sleeping alone by being present until the child falls asleep. Nightmares can be managed by comforting the child and preventing specific fears. Nightmares and disturbed sleep may be brought on by hospitalization and may improve once the child adapts himself or herself to the new environment. Bed-wetting is common in younger children but needs further evaluation in older children.)

The nurse develops a plan of care based on the information documented in a child's admission assessment. The nurse instructs the health care team that they should not leave the room until the child falls asleep. What information documented under the self-perception-self-concept pattern would necessitate this nursing intervention? The child has: A. Nightmares. B. Disturbed sleep patterns. C. A fear of sleeping alone. D. The habit of bed-wetting.

B (The family's presence will decrease the child's distress. The mother may experience guilt, but this is not the best answer. Although separation from the mother is a developmental threat for toddlers, the main reason to keep parents at the child's bedside is to ease anxiety and therefore respiratory effort. The child should have constant cardiorespiratory monitoring and noninvasive oxygen saturation monitoring, but the parent should not play this role in the hospital.)

The nurse encourages the mother of a toddler with acute laryngotracheobronchitis to stay at the bedside as much as possible. The nurse's rationale for this action is primarily that: a. Mothers of hospitalized toddlers often experience guilt. b. The mother's presence will reduce anxiety and ease the child's respiratory efforts. c. Separation from the mother is a major developmental threat at this age. d. The mother can provide constant observations of the child's respiratory efforts.

C (The painting materials made from wallpaper paste may contain wheat. If the paint comes in contact with the child's skin, it can cause an allergic reaction. Therefore the nurse changes the activity. The child who is on a salt-restricted diet need not worry about salt being in the paint. A patient is placed on salt restriction to prevent water retention, not because of an allergy. The child who is lactose intolerant cannot ingest dairy products. Beetroot is used as natural dyes in paintings; however, the wallpaper paste does not contain beetroot extract.)

The nurse gets out finger painting materials made from wallpaper paste for a child. After reviewing the child's medical record, the nurse decides to get out crayons and a coloring book instead. What information did the nurse find in the nursing history? The child: A. Is on a salt-restricted diet. B. Has lactose intolerance. C. Has an allergy to wheat. D. Is allergic to beetroot.

B (It is impossible to evaluate the improvement in the respiratory function of the child without having any baseline data. The child's feedback provides only subjective assessment. Evaluation of respiratory system function requires formal knowledge of the respiratory system assessment. Therefore, parent opinion is not reliable for improvement in the child's respiratory function. Evaluation of the improvement in the respiratory function requires objective assessment. Obtaining the primary health care provider's opinion is a type of subjective assessment.)

The nurse has been assigned to the pediatric respiratory unit. What is the preliminary requirement for the nurse to evaluate improvement in the respiratory function of the child with treatment? A. The child's feedback B. The baseline data C. The parents' opinion D. The primary health care provider's opinion

C (Couples that become new parents may develop new challenges in their personal relationship because they will spend most of their time taking care of the baby. Therefore the couple needs to make time to spend together rather than spending all of their time focused on the baby. This can be accomplished by scheduling time for one another, apart from the time dedicated to caring for the infant. The couple has to share their expectations with each other and should appreciate each other for their assistance in the child care activities.)

The nurse instructs a group of nursing students about relationship issues in couples who are first-time parents. The nurse asks a student for suggestions to give new parents to prevent relationship problems. Which student response indicates the need for further teaching? A. "Appreciate each other for the assistance." B. "Assess your relationship on a regular basis." C. "You should both spend more time with the baby." D. "Share your personal expectations with each other.

A, B, C (Male sex carries a more favorable prognosis than female sex. Early recognition allows early intervention to help the child recover. Children with functional speech have a better prognosis than those who do not have functional speech. Children with higher intelligence have a more favorable prognosis than children with lesser intelligence. Children who do not have behavioral impairment have a better prognosis than children with behavioral impairment.)

The nurse is assessing a child with autism for prognostic factors. What findings in the child suggest a better prognosis? Select all that apply. A. Male sex B. Early recognition C. Functional speech D. Lower intelligence E. Behavioral impairment

A, B, E (Children with autism usually have verbal impairment caused by poor language development. Autistic children exhibit stereotyped behavioral patterns caused by impaired neuromuscular function. Such children show decreased interest in functional play activities. Autistic children do not usually have hearing and visual impairment. Autistic children exhibit repetitive behavioral patterns.)

The nurse is assessing a child with autism. What characteristic features of autism does the nurse expect to find in the child? Select all that apply. A. Verbal impairment B. Stereotyped behavior patterns C. Hearing and visual impairment D. Nonrepetitive behavioral patterns E. Decreased involvement in play

B (The self-care scale can be used for rating the functional self-care abilities of the child. The score ranges from 0 to IV. If the child is scored a II, this implies that the child requires assistance or supervision from another person. A child who is independent with activities of daily living would receive a 0. A score of I implies that the child requires equipment or a device for self-care. A score of III implies that the child requires assistance or supervision from another person and equipment or a device. A score of IV implies that the child is totally dependent and does not participate.)

The nurse is assessing a child's level of self-care. The nurse documents a rating of II for dressing and grooming. What can be inferred from this rating? The child: A. Is independent on all aspects of personal care. B. Depends on the supervision of another person. C. Needs to use equipment or another adaptive device. D. Requires direction from a person and uses equipment.

A (Newborns with Down syndrome have joint hyperflexibility and low muscle tone. This can make it difficult to hold the newborn because he or she can go limp like a rag doll. This makes it difficult for the parents to embrace and provide warmth to their newborn. This may make parents feel that the newborn is not bonding with them, but difficulty holding the child does not indicate impaired bonding between the child and parents. Inability to understand the child's needs and nonverbal communication indicates undeveloped bonding. Asking the parents whether they are more apprehensive does not answer their question. It is also a closed-ended question, which is not therapeutic communication. Telling the parents they need to see a counselor is not appropriate. They just need support and teaching.)

The nurse is assessing a newborn with Down syndrome. The newborn's parent tells the nurse, "We are having a hard time holding our baby. We didn't have this hard of a time with our other children." What would be the nurse's best response? A. "Children with Down syndrome have lower muscle tone." B. "This happens in some children because of undeveloped bonding." C. "Are you more apprehensive because your child has Down syndrome?" D. "You should see a counselor to help you cope with your child's condition."

A, C, D (Strabismus may result from poor vision and the resulting straining of eye muscles. Strabismus may result from a congenital defect as a developmental anomaly. Strabismus may also result from muscle imbalance caused by neuromuscular disorders. Short eyeball results in development of hyperopia, not strabismus. Unequal curvature of lens results in astigmatism, not strabismus.)

The nurse is assessing a patient with strabismus. Which finding would suggest the cause of strabismus? Select all that apply. A. Poor vision B. Short eyeball C. Congenital defect D. Muscle imbalance E. Unequal curvature in the lens

C (The nurse should rate the child as a II (two) because the child requires assistance of a caregiver for general hygiene and dressing. A grading of 0 (zero) is given to the child who is capable of taking full self-care. A grading of a I (one) is given to the child who requires the use of equipment or a device for self-care. A child who is totally dependent and does not participate in self-care would be rated a IV (four).)

The nurse is assessing the functional self-care level of a child and determines that the child requires the assistance of a caregiver for general hygiene and dressing. How would the nurse rate the child? A. 0 B. I C. II D. IV

C, D, E (Hydration is important in children with RSV bronchiolitis to loosen secretions and prevent shock. Clustering of care promotes periods of rest. The use of noninvasive oxygen monitoring is recommended.)

The nurse is caring for a 10-month-old infant with respiratory syncytial virus (RSV) bronchiolitis. Which interventions should be included in the child's care (Select all that apply)? a. Administer antibiotics. b. Administer cough syrup. c. Encourage infant to drink 8 ounces of formula every 4 hours. d. Institute cluster care to encourage adequate rest. e. Place on noninvasive oxygen monitoring.

A (The couple does not attempt to talk to the infant. This indicates that they lack interest in developing a bond with the infant. Calling the infant by name indicates the couple is giving importance to the individuality of the infant. In Chinese culture, the newborn is taken care of by the grandparents. This is done to promote rest and recovery in the mother after childbirth. Maintaining eye contact with the infant is a sign of attention by the parents toward the infant.)

The nurse is caring for a Chinese couple who have a newborn. What patient behavior indicates that the couple lacks interest in developing an attachment with the child? The couple: A. Never talks to the infant. B. Always calls the infant by name. C. Allows grandparents to take care of the child. D. Maintains good eye contact with the infant.

C (The infant wants to interact with the mother by looking at the mother's eyes and facial expressions. Visually impaired mothers may show impassive facial expressions, which makes the infant uninterested. The infant may abandon the mother and try to interact with other family members. The mother can interact efficiently by nodding and smiling frequently while talking. This conceals the impassive facial expressions of the mother. Because the father cannot improve the facial expressions of the mother by standing nearby, the father cannot improve the interaction between the mother and infant by standing nearby. As the infant looks at the face of the mother to interact, holding the newborn's hand while talking will not improve the interaction either. The mother should give spontaneous responses to the child's actions. Waiting for others to tell her about the infant's actions may not be helpful in this situation.)

The nurse is caring for a North American, postpartum patient who is visually impaired. The nurse finds that the mother is having a difficult time establishing attachment with the infant. What suggestion would the nurse give the mother in order to create an effective interaction with the infant? A. "Ask the infant's father to stand nearby." B. "Hold the newborn's hand while talking." C. "Nod and smile while talking to the infant." D. "Get informed of the actions of the infant."

B (Monitoring cardiopulmonary status is an important evaluation tool in the care of the child with ARDS. Maintenance of vascular volume and hydration is important and should be done parenterally. Seizures are not a side effect of ARDS. Adequate nutrition is necessary, but a high-protein diet is not helpful.)

The nurse is caring for a child with acute respiratory distress syndrome (ARDS) associated with sepsis. Nursing actions should include: a. Force fluids. b. Monitor pulse oximetry. c. Institute seizure precautions. d. Encourage a high-protein diet.

C (The nurse should use a telephone to maintain contact between the child and parents so that the child can feel comfortable. It helps relieve the child's anxiety. The nurse should maintain eye contact and gently touch the child to establish rapport. The nurse should talk with the child about the parents and family to prevent detachment of the child from the parents. The nurse should not use a laptop to contact the child and parents. The laptop may not be compatible with medical equipment, and use may be restricted in certain areas.)

The nurse is caring for a child with an influenza viral infection. The child is anxious because the parents are unable to stay with the child. What should the nurse do to relieve the child's anxiety? The nurse should: A. Not maintain any eye contact with the child. B. Not speak with the child about missing the parents. C. Use the phone to let the child talk with the parents. D. Use a laptop to allow the child and parents to talk

C (The nurse should ask the child's parents about how the child usually handles disappointment. This can help the nurse understand the coping-stress tolerance pattern of the child. It is also helpful for identifying stressors in the child. It is important to know how discipline problems are managed in the child. This helps to know about the child and parent role-relationship pattern. When the nurse asks about the child's friends, it is to assess the child's role and relationship patterns outside the home. The nurse can understand the role and relationship pattern between the parents and child after knowing who will stay in the hospital with the child.)

The nurse is caring for a child with cancer. What should the nurse ask the child's parents about in order to obtain information about the child's coping-stress tolerance pattern? A. "How do you both handle discipline problems at home?" B. "Have you ever noticed if your child has many friends?" C. "How does your child usually handle disappointment?" D. "Who will be staying with your child at the hospital?"

C (The statement "I do not know what is going on with this child's health" is inappropriate because it indicates that the nurse does not understand the child's needs. The statement "I need to know more about cognitive impairment" shows the nurse's desire to learn about the disorder and help treat the child better. The statement "I will ask the other staff to help with the child's care" shows the nurse's desire to increase his or her skill level. The statement "I will ask the health care provider to clarify my question" is an inappropriate statement made by the nurse. It implies that the nurse is not aware of the care that should be given to a cognitively impaired child.)

The nurse is caring for a child with cognitive impairment. Which statement made by the nurse to the parents is a reason for concern? A. "I need to know more about cognitive impairment." B. "I will ask the other staff to help with the child's care." C. "I do not know what is going on with this child's health." D. "I'll ask the health care provider to clarify my question."

A (When a newborn is suspected of having tracheoesophageal fistula, the most desirable position is supine with the head elevated on an inclined plane of at least 30 degrees. It is imperative that any source of aspiration be removed at once; oral feedings are withheld. Feedings of fluids should not be given to infants suspected of having tracheoesophageal fistulas. The oral pharynx should be kept clear of secretion by oral suctioning. This is to avoid the cyanosis that is usually the result of laryngospasm caused by overflow of saliva into the larynx.)

The nurse is caring for a neonate with a suspected tracheoesophageal fistula. Nursing care should include: a. Elevating the head but giving nothing by mouth. b. Elevating the head for feedings. c. Feeding glucose water only. d. Avoiding suctioning unless the infant is cyanotic.

A (Native American patients do not initiate breastfeeding until their breast milk comes in. They avoid feeding the first milk (colostrum) to the child. In this situation, the nurse should inform the patient about the health benefits of feeding colostrum to the child. Mother's milk is the best source of nutrition for the baby; the formula milk is not as nutritious. The mother is also unlikely to have problems ejecting milk. Native American mothers tend to avoid feeding the baby unless the colostrum stops ejecting and the mother starts ejecting milk. Because the patient has not yet started breastfeeding, it is unlikely that the patient has sore nipples, so the nurse would not need to apply anti-inflammatory ointment.)

The nurse is caring for a postpartum Native American patient who is unwilling to breastfeed the baby until after the breast milk has come in. What will the nurse do in this situation? A. Explain the importance of first milk to the patient. B. Ask the patient to give formula milk to the infant. C. Use additional measures for milk ejection in the patient. D. Apply anti-inflammatory ointment around the patient's nipples.

D (While interacting with the parents of an infant who are hearing-impaired, the nurse should suggest that the parents show video recordings to the infant to improve vocalization, as this helps the newborn become familiar with human voices. Interacting with the child using sign language can enhance communication, but may not help to improve vocalization. Playing rhymes and music at a very high volume may damage the eardrum of the infant and can lead to hearing impairment. Asking a family member or friend to talk to the baby is a good solution, but it may not be a practical solution if the parents care for the child independently.)

The nurse is caring for a postpartum patient with a hearing impairment. The nurse finds that the patient's partner is also hearing-impaired. What does the nurse suggest to both parents to improve vocalization in the newborn? A. Interact with the infant using sign language. B. Play rhymes and music at a very high volume. C. Ask a family member or friend to talk to the baby. D. Show the infant recordings of television programs.

A (When the drug is questionable or unknown, every effort must be made to determine the type, amount of drug taken, the mode and time of administration, and factors relating to the onset of presenting symptoms. Because the actual content of most street drugs is highly questionable, this information would be difficult to obtain. It is helpful to know the pattern of use but not essential during this emergency. This is an inappropriate time for an evaluation about the level of interest in rehabilitation.)

The nurse is caring for an adolescent brought to the hospital with acute drug toxicity. Cocaine is believed to be the drug involved. Data collection should include the: a. Mode of administration. b. Actual content of the drug. c. Function the drug plays in the adolescent's life. d. Adolescent's level of interest in rehabilitation.

C (Loss of peer-group contact may pose a severe emotional threat to an adolescent because of loss of group status; friends visiting is an important aspect of hospitalization for an adolescent and would be very reassuring. Adolescents may welcome the opportunity to be away from their parents. The separation from siblings may produce reactions from difficulty coping to a welcome relief.)

The nurse is caring for an adolescent who had an external fixator placed after suffering a fracture of the wrist during a bicycle accident. Which statement by the adolescent would be expected about separation anxiety? a. "I wish my parents could spend the night with me while I am in the hospital." b. "I think I would like for my siblings to visit me but not my friends." c. "I hope my friends don't forget about visiting me." d. "I will be embarrassed if my friends come to the hospital to visit."

B (The suture line should be cleansed gently after feeding. The child should be positioned on back or side or in an infant seat. Elbows are restrained to prevent the child from accessing the operative site. Postural drainage is not indicated. This would increase the pressure on the operative site when the child is placed in different positions. Mouth irrigations would not be indicated.)

The nurse is caring for an infant whose cleft lip was repaired. Important aspects of this infant's postoperative care include: a. Arm restraints, postural drainage, and mouth irrigations. b. Cleansing of suture line, supine and side-lying positions, and arm restraints. c. Mouth irrigations, prone position, and cleansing of suture line. d. Supine and side-lying positions, postural drainage, and arm restraints.

C (Visible gastric peristaltic waves that move from left to right across the epigastrium are observed in pyloric stenosis, as is weight loss. Abdominal rigidity and pain on palpation, and rounded abdomen and hypoactive bowel sounds, are usually not present. The upper abdomen is distended, not the lower abdomen.)

The nurse is caring for an infant with suspected pyloric stenosis. Which clinical manifestation would indicate pyloric stenosis? a. Abdominal rigidity and pain on palpation b. Rounded abdomen and hypoactive bowel sounds c. Visible peristalsis and weight loss d. Distention of lower abdomen and constipation

B (Smokeless tobacco is a popular substitute for cigarettes and poses serious health hazards to children and adolescents. Smokeless tobacco is associated with cancer of the mouth and jaw. Smokeless tobacco is just as addictive as cigarettes. Although teens believe that it is easy to stop using smokeless tobacco, this is not the case. A popular belief is that smokeless tobacco is a safe alternative to cigarettes; this has been proven incorrect. Half of all teens who use smokeless tobacco agree that it poses significant health risks.)

The nurse is completing a health history with a 16-year-old male. He informs the nurse that he has started using smokeless tobacco after he plays baseball. Which information regarding smokeless tobacco would be most correct for the nurse to provide to this teen? a. Not addicting. b. Proven to be carcinogenic. c. Easy to stop using. d. A safe alternative to cigarette smoking.

A, C, E (Young children's posthospital behaviors include: They show initial aloofness toward parents; this may last from a few minutes (most common) to a few days. This is frequently followed by dependency behaviors: tendency to cling to parents; demands for parents' attention; vigorous opposition to any separation (e.g., staying at preschool or with a babysitter). Other negative behaviors include: new fears (e.g., nightmares); resistance to going to bed, night waking; withdrawal and shyness; hyperactivity; temper tantrums; food peculiarities; attachment to blanket or toy; regression in newly learned skills (e.g., self-toileting). Posthospital behaviors for older children include negative behaviors: emotional coldness followed by intense, demanding dependence on parents; anger toward parents; jealousy toward others (e.g., siblings).)

The nurse is discharging a young child from the hospital. The nurse should instruct the parents to look for which posthospital child behaviors? Select all the apply. A. Tendency to cling to parents B. Jealousy toward others C. Demands for parents' attention D. Anger toward parents E. New fears such as nightmares

C (Individuals with cognitive impairment need a well-defined, concrete code of sexual conduct.)

The nurse is discussing sexuality with the parents of an adolescent girl with moderate cognitive impairment. Which should the nurse consider when dealing with this issue? a. Sterilization is recommended for any adolescent with cognitive impairment. b. Sexual drive and interest are limited in individuals with cognitive impairment. c. Individuals with cognitive impairment need a well-defined, concrete code of sexual conduct. d. Sexual intercourse rarely occurs unless the individual with cognitive impairment is sexually abused.

B, C, E (Ambulatory care is associated with an increased cost-saving as compared to hospital admissions, since there are no admission-related costs. Ambulatory care is associated with lesser chances of acquiring infections due to limited exposure to health care facilities. Ambulatory care is devoid of the stressors of hospitalization. There is deficient care due to the absence of qualified medical person for supervision. Ambulatory care is more challenging when compared to hospitalization as the child and the parents need to rely mostly on themselves for providing care to the child.)

The nurse is educating a group of parents and children in the pediatric ward about the benefits of ambulatory care. What benefits does the nurse discuss with the group? Select all that apply. A. Improved care B. Increased cost-saving C. Reduced chances of infection D. Ambulatory care is lesser challenging E. Minimum stressors of hospitalization

C (Tanner sexual-maturing ratings are based on the development of stages of primary and secondary sexual characteristics. Tanner stages are not based on chronologic age. The age at which an adolescent enters puberty is variable. The puberty stage in girls begins with breast development. The puberty stage in boys begins with genital enlargement. Primary sexual characteristics are not the sole basis of Tanner staging.)

The nurse is explaining Tanner staging to an adolescent and her mother. Which statement best describes Tanner staging? a. Predictable stages of puberty that are based on chronologic age b. Staging of puberty based on the initiation of menarche and nocturnal emissions c. Predictable stages of puberty that are based on primary and secondary sexual characteristics d. Staging of puberty based on the initiation of primary sexual characteristics

B, C, D (The bill of rights emphasizes the quality of health care to the children and explains that children and teens should be treated with respect and dignity. The bill also states that children have the right to make choices and decisions in their health care. Children have right to get emotional support from the health care professionals. Children cannot always expect economic assistance from the hospital. The nurse need not explain complicated information such as pathologic process of the disease to the child since the child will not be able to understand. Such information should be provided to the caregivers of the child.)

The nurse is explaining the health care bill of rights for children to parents and children in a pediatric ward. What key information does the nurse discuss with the group? Select all that apply. A. Quality health care B. Economic assistance C. Respect and personal dignity D. Making choices and decisions E. Complex information

C (It is important to emphasize the need for the child to brush the teeth or rinse the mouth after vomiting to dilute the hydrochloric acid that comes in contact with the teeth. Administration of a glucose-electrolyte solution to an alert child will help restore water and electrolytes satisfactorily. It is important to include carbohydrates to spare body protein and avoid ketosis.)

The nurse is explaining to a parent how to care for a child with vomiting associated with a viral illness. The nurse should include: a. Avoiding carbohydrate-containing liquids. b. Giving nothing by mouth for 24 hours. c. Brushing teeth or rinsing mouth after vomiting. d. Giving plain water until vomiting ceases for at least 24 hours.

A (The nurse records the admission history of the patient in terms of different functional health patterns. This helps in documenting all the required information about the patient. The patient requires help inserting his or her contact lenses. This implies that the patient needs support to perform an activity. The nurse should record this information under the activity-exercise pattern. The cognitive-perceptual pattern recognizes the cognitive development in the child and includes information such as defects in vision, hearing, or grading in the school. The nutrition-metabolic pattern is used in the assessment of nutrition in the patient, food allergies, and food intake habits. The health perception-health management pattern reports the medication and the health history of the child.)

The nurse is obtaining the admission history of a recently admitted adolescent. The nurse notes the patient requires help inserting contact lenses. Under which functional health pattern should the nurse record this observation? A. Activity-exercise pattern B. Cognitive-perceptual pattern C. Nutrition-metabolic pattern D. Health perception-health management pattern

A (The child is refusing to take vaccination because of fear of bleeding and pain. The nurse should ask the child to select the color of the bandage to be used. This reassures the child and will make him or her feel better. Even if the nurse tells the child that the bleeding will stop when the needle is removed, it does little to help relieve the child's fear. The nurse should not scold the child in a firm tone because the child may get frightened. Giving a favorite toy to the child for playing is not helpful for relieving the fear. A favorite toy may help the child sleep at night. Requesting a staff member sit beside the child may not be helpful for relieving the child's fear. It may be needed to help hold the child still during a procedure.)

The nurse is preparing to administer a vaccine to a child. The child is refusing to take the vaccination because of fear of bleeding. What should the nurse do in this situation? A. Tell the child he or she can pick the bandage color. B. Tell the child bleeding will stop in a few seconds. C. Request a staff member sit beside the child. D. Give a favorite toy to the child for distraction.

B, D, E (Feedings are usually instituted soon after a pyloromyotomy surgery, beginning with clear liquids and advancing to formula or breast milk as tolerated. IV fluids are administered until the infant is taking and retaining adequate amounts by mouth. Appropriate analgesics should be given around the clock because pain is continuous. Ice should not be applied to the incisional area as it vasoconstricts and would reduce circulation to the incisional area and impair healing.)

The nurse is preparing to care for an infant returning from pyloromyotomy surgery. Which prescribed orders should the nurse anticipate implementing (Select all that apply)? a. Nothing by mouth for 24 hours b. Administration of analgesics for pain c. Ice bag to the incisional area d. Intravenous (IV) fluids continued until tolerating fluids by mouth e. Clear liquids as the first feeding

B (Play is one of the most important aspects of a child's life and one of the most effective tools for managing stress. It is helpful for the child to relieve stress. It is also essential for the child's mental, emotional, and social well-being. Play does not make the child anxious in an unfamiliar environment. It helps the child feel more secure in a strange environment. Play lessens the stress of separation from the family because the child is busy. During play, the child communicates with others, which helps develop a positive attitude toward others. It also stimulates thinking in the child by allowing the child to express creative ideas.)

The nurse is teaching the nursing students about functions of play in the hospital. Which statement made by the nursing student indicates the need for further teaching? "Play: A. Can lessen the stress of separation from the family." B. Makes the child nervous in a strange environment." C. Helps the child develop a positive attitude for others." D. Provides an expressive outlet for the child's creative ideas."

B (Intravenous antibiotics are a priority action for the nurse. A short delay may be possible to allow the child some choice, but a prolonged delay only serves to increase the anxiety. The nurse should start the IV line, recognizing that the child is attempting to gain control. If the timing of the IV line start was not essential for the start of IV antibiotics, postponing might be acceptable. The child may never be ready. The anxiety is likely to increase with prolonged delay.)

The nurse needs to start an intravenous (IV) line on an 8-year-old child to begin administering intravenous antibiotics. The child starts to cry and tells the nurse, "Do it later, O.K.?" The nurse should: A. start the IV line because allowing the child to manipulate the nurse is bad. B. start the IV line because unlimited procrastination results in heightened anxiety. C. postpone starting the IV line until the child is ready so that the child experiences a sense of control. D. postpone starting the IV line until the child is ready so that the child's anxiety is reduced.

A (What appears to be a lack of interest in the newborn is in fact the Vietnamese way of demonstrating intense love by attempting to ward off evil spirits.)

The nurse notes that a Vietnamese woman does not cuddle or interact with her newborn other than to feed him, change his diapers or soiled clothes, and put him to bed. In evaluating the woman's behavior with her infant, the nurse realizes that: A. What appears to be a lack of interest in the newborn is in fact the Vietnamese way of demonstrating intense love by attempting to ward off evil spirits. B. The woman is inexperienced in caring for newborns. C. The woman needs a referral to a social worker for further evaluation of her parenting behaviors once she goes home with the newborn. D. Extra time needs to be planned for assisting the woman in bonding with her newborn.

B (The woman should be encouraged to hold her infant in the en face position and make eye contact with him. Talking and cooing to her son is a normal infant-parent interaction. Cuddling is a normal infant-parent interaction. Sharing her son's success at feeding is a normal infant-parent interaction.)

The nurse observes several interactions between a postpartum woman and her new son. What behavior (if exhibited by this woman) does the nurse identify as a possible maladaptive behavior regarding parent-infant attachment? A. Talks and coos to her son B. Seldom makes eye contact with her son C. Cuddles her son close to her D. Tells visitors how well her son is feeding

B (Show the mother how the infant initiates interaction and attends to her.)

The nurse observes that a 15-year-old mother seems to ignore her newborn. A strategy that the nurse can use to facilitate mother-infant attachment in this mother is: A. Tell the mother she must pay attention to her infant B. Show the mother how the infant initiates interaction and attends to her C. Demonstrate for the mother different positions for holding her infant while feeding D. Arrange for the mother to watch a video on parent-infant interaction

A (Mexican mothers may believe that excessive admiration may result in evil eye (mal de ojo). They feel that children are more susceptible to evil eye. This behavior does not indicate that the mother does not have a good attachment with the baby. Similarly, this evasive behavior does not indicate the mother is worried about infection or the baby becoming fussy after exposure to new faces.)

The nurse observes that a Spanish-speaking patient of Mexican descent does not like the nurses to lean in and admire her newborn. What is the most likely reason for this behavior? The mother: A. Wants to protect the infant from evil eye. B. Does not have good attachment with the baby. C. Feels that the baby may acquire infection. D. Feels that the baby may become fussy seeing new faces.

A (Patients of Southeast Asian descent believe that minimal touching and cuddling of the newborn after birth protects the child from evil spirits. If the patient had postpartum blues, the patient may show different symptoms such as crying episodically. If the patient does not hold or feed the child, it would indicate that the patient is afraid of handling the child. Avoiding cuddling and touching the newborn does not indicate that the patient is improving the baby's strength and immunity. Southeast Asians believe that massaging the newborn would be helpful in improving the baby's strength and immunity.)

The nurse observes that a patient of Southeast Asian descent avoids cuddling and touching her newborn. What can the nurse infer about the patient from these observations? The patient: A. May be trying to protect the child from evil spirits. B. Is depressed because of postpartum blues. C. Is afraid of handling the child all by herself. D. Is improving the baby's strength and immunity

D (While learning to breastfeed, women may refrain from talking or smiling excessively so that the infant will not be distracted and will continue to feed. The patient does not avoid talking to soothe and quiet the infant; instead, she may gently stroke the infant to provide comfort after feeding. Avoiding speaking with the infant is not helpful for coping with depression and is not a typical behavior of postpartum blues. The mother's quiet behavior during breastfeeding does not indicate that she has not properly bonded with the baby.)

The nurse observes that a postpartum patient does not talk or smile during breastfeeding but instead quietly maintains her gaze on the infant. What does the nurse infer from this observation? The patient: A. Aims to soothe and quiet the infant. B. Is experiencing postpartum blues. C. Is not properly bonding with the infant. D. Wants the infant to suck the milk without interruption.

A, B, E (For assessing the role-relationship pattern in the child, the nurse should ask the parents about any security objects the child may have at home that provide comfort, discipline problems of the child, and family changes. From this information, the nurse can understand the relationship between the parents and the child. Information about sweating gives an idea about the elimination pattern in the child. Information about the disappointment handling potential of the child gives an idea about the child's coping-stress tolerance pattern.)

The nurse plans to assess the role-relationship pattern in a child. Which questions should the nurse ask the parents? Select all that apply. A. "Does the child have any security objects at home?" B. "How do you handle discipline problems at home?" C. "Have you ever noticed that your child sweats a lot?" D. "How does your child usually handle disappointments?" E. "Have any major changes in the family occurred lately?"

B (Hospitalization is a stressor in children and so they may react differently to it. Certain children are more susceptible to the stressful effects of hospitalization than others. Children who have difficult temperament may not readily adjust with the unfamiliar environment of the hospital. These children may experience adverse effects of hospitalization. Female children are able to withhold stress more when compared to male children and thus are less likely to experience stressors. Children with average intelligence may be able to understand their condition and the importance of hospitalization and thus may be more adaptable. Children with lower IQ would not understand the purpose of hospital admission and thus would be extremely stressed due to hospitalization. Children who are older than 6 years of age have developed the maturity to understand their condition and the purpose of hospitalization. Thus, they would be more adaptable to their condition, and experience less stress related to hospitalization.)

The nurse works in a pediatric unit. Which child would have an increased vulnerability to the stresses of hospitalization? A. A female child B. A child with a difficult temperament C. A child with an average intelligence D. A child older than 6 years of age

B, C, E (The nurse should assess the child's usual health habits at home to promote a more normal environment in the hospital. This includes the child's sleep-rest, nutritional-metabolic, and activity-exercise patterns. The nurse would assess the sleep-rest pattern by asking when the child goes to sleep at night. Assessing the nutritional-metabolic pattern would include asking about food preferences. The nurse should also ask what toy the child plays with at home as part of the activity-exercise pattern. These will help the nurse plan individualized care for the child. History about herbal and complementary therapy helps in preventing drug-drug interaction and severe adverse effects.)

The nursing student is caring for a child admitted to the hospital. The nursing student asks the nurse instructor, "How can we keep the child's routine habits while he is in the hospital?" What would be the best response by the nurse instructor? Select all that apply. "Ask the parents: A. "About the use of any herbal therapies." B. "When the child goes to sleep at night." C. "What foods the child prefers to eat." D. "How the child's grades are in school." E. "Which toy the child plays with at home."

B (Children 2 to 3 years old understand the common language used at home, and they try to communicate with family members in the same language. If the child has difficulty understanding and responding after the parent repeats a statement several times, this may indicate the child has a hearing problem. The child does not have lack of orientation, so the nurse should not infer that the child has cognitive impairment. Children stop using gestures and start communicating verbally around the age of 15 months. Therefore the child does not have normal development. The child is not bullying or being aggressive, so the nurse should not infer that the child has a chronic mental illness.)

The parent of a 2-year-old child tell the nurse that the child likes to play alone and asks people to repeat questions several times. The parent also says that the child uses gestures to communicate. What should the nurse infer from this? The child has: A. Cognitive impairment. B. Difficulty hearing. C. Normal development. D. Chronic mental illness.

A (The child is exhibiting signs of increasing respiratory distress suggestive of a pneumothorax. He or she needs to be seen as soon as possible. Bronchodilation and carbon dioxide retention would not produce the symptoms listed. Bronchodilation and carbon dioxide retention would not produce the symptoms listed. The increased viscosity of sputum is characteristic of cystic fibrosis. The change in respiratory status is potentially caused by a pneumothorax.)

The parent of a child with cystic fibrosis calls the clinic nurse to report that the child has developed tachypnea, tachycardia, dyspnea, pallor, and cyanosis. The nurse should tell the parent to bring the child to the clinic because these symptoms are suggestive of: A. Pneumothorax. B. Bronchodilation. C. Carbon dioxide retention. D. Increased viscosity of sputum.

B (Spasmodic croup is viral in origin, is usually preceded by several days of symptoms of upper respiratory tract infection, and often begins at night. It is marked by a harsh, metallic, barky cough; sore throat; inspiratory stridor; and hoarseness. Wheezing is not a distinguishing manifestation of croup. It can accompany conditions such as asthma or bronchiolitis. A high fever is not usually present.)

The parent of a toddler calls the nurse, asking about croup. What is a distinguishing manifestation of spasmodic croup? a. Wheezing is heard audibly. b. It has a harsh, barky cough. c. It is bacterial in nature. d. The child has a high fever.

D (Changes in respiratory patterns and increasing activity reflect the child's excitement about being close to the parent. The child is able to hear the parent's voice but is not able to smile when hearing the parent's voice at this age. This happens after about 2 years of age. Decreased attention span could be caused by lack of interest in the communication. When a child makes sounds in response to the parent's communication, it is sign of intimate bonding between the child and parent.)

The parent of a visually impaired infant says to the nurse, "I am afraid that my child may not be able to bond with me because my child cannot maintain eye contact with me." The nurse teaches the parent about other signs that indicate that the child is responding. What should the nurse include in the explanation? A. "The child compensates by increasing listening to your voice and smiling." B. "The child's attention span decreases when a parent is trying to communicate." C. "The child does not make throaty sounds when a parent is trying to communicate." D. "The child's breathing or activity increases when the child is in contact with or near a parent."

D (If an infant with nasopharyngitis has a fever over 101° F, there is early evidence of respiratory complications. Irritability and a slight fever are common in an infant with a viral illness. Cough can be a sign of nasopharyngitis.)

The parent of an infant with nasopharyngitis should be instructed to notify the health care professional if the infant: a. Becomes fussy. b. Has a cough. c. Has a fever over 99° F. d. Shows signs of an earache.

D (If a child has a penetrating eye injury of any kind, the nurse should examine the eye to determine whether any aqueous humor has leaked from the penetration site. The nurse should observe the presence of hyphema, or bleeding from the eye. The nurse should also assess for pupillary reaction to light because it helps assess the functioning of the pupil. The nurse does not need to examine the eye for foreign bodies because there is already a foreign body in the eye. If the child is experiencing a penetrating eye injury, the nurse does not irrigate the eye to remove the object because this can further damage the cornea. In the case of chemical burns, the nurse rinses the eye by everting the upper eyelid.)

The parents brought their child to the emergency department after a needle penetrated the child's eye. Which action should the nurse perform while caring for the child? A. Examine the eye to look for foreign bodies. B. Irrigate the eye to remove the needle from the eye. C. Evert the upper eyelid to wash the eye thoroughly. D. Observe for hyphema and reaction of the pupil to light.

C (The peer group serves as a strong support to teenagers, providing them with a sense of belonging and strength and power. During adolescence, the parent-child relationship changes from one of protection-dependency to one of mutual affection and quality. Parents continue to play an important role in personal and health-related decisions. The peer group forms the transitional world between dependence and autonomy.)

The parents of a 14-year-old girl express concerns about the number of hours their daughter spends with her friends. The nurse explains that peer relationships become more important during adolescence because: a. Adolescents dislike their parents. b. Adolescents no longer need parental control. c. They provide adolescents with a feeling of belonging. d. They promote a sense of individuality in adolescents.

C (Egocentric and narcissistic behavior is normal during this period of development. The teenager is seeking a personal identity. Stricter limits are not an appropriate response for a behavior that is part of normal development. More responsibility at home is not an appropriate response for this situation. The behavior is normal and needs no further investigation.)

The parents of a 15-year-old girl are concerned that their adolescent spends too much time looking in the mirror. Which statement is the most appropriate for the nurse to make? a. "Your teenager needs clearer and stricter limits about her behavior." b. "Your teenager needs more responsibility at home." c. "During adolescence this behavior is not unusual." d. "The behavior is abnormal and needs further investigation."

C (A strange light in the eyes indicates that the child may have retinoblastoma. It is diagnosed by ophthalmoscopic examination, which involves dilation of the pupil. During this procedure, the eyes become sensitive, and the child may not be able to see clearly for some time. Informing the parents about it will reduce anxiety. Hematologic assessment is not used to diagnose retinoblastoma. A retinoblastoma can be diagnosed by ophthalmoscopic examination under general anesthesia and with imaging studies, including ultrasonography and computed tomography. A child with visual impairment may have difficulty distinguishing between colors, but it is not helpful for the parents to learn about it before the child's illness is diagnosed. The child may need immediate hospitalization after the examination depending on the severity of tumor, but the nurse should not tell the parents about hospitalization because it could make them panic.)

The parents of a 3-year-old child report seeing a whitish glow in the child's eyes. The nurse begins to examine the child. What information should the nurse give to the parents before assessment? "The child: A. Needs hematologic assessment for confirmation of diagnosis." B. May not be able to distinguish between colors." C. May not see clearly for some time after the examination." D. Needs immediate hospitalization after the examination."

C (Behavior modification with positive reinforcement is effective in children with cognitive impairment. Discipline is essential in assisting the child in developing boundaries. Positive behaviors and desirable actions should be reinforced. Most children with cognitive impairment will not be able to understand the reason for the physical punishment; consequently behavior will not change as a result of the punishment.)

The parents of a cognitively impaired child ask the nurse for guidance with discipline. The nurse's best response is: A. "Discipline is ineffective with cognitively impaired children." B. "Discipline is not necessary for cognitively impaired children." C. "Behavior modification is an excellent form of discipline." D. "Physical punishment is the most appropriate form of discipline."

C (The second most common genetic cause of cognitive impairment.)

The pediatric nurse understands that fragile X syndrome is: a. A chromosome defect affecting only females. b. A chromosome defect that follows the pattern of X-linked recessive disorders. c. The second most common genetic cause of cognitive impairment. d. The most common cause of noninherited cognitive impairment.

B (Common emotional consequences of obesity include poor body image, low self-esteem, social isolation, and feelings of depression and isolation. Sexual promiscuity is an unlikely effect of obesity. The obese adolescent often substitutes food for affection. Eating behaviors are closely related to memory. Memory and appetite are closely linked and can be modified over time with treatment. Obese adolescents most often have a very poor self-image.)

The psychologic effects of being obese during adolescence include: a. Sexual promiscuity. b. Poor body image. c. Memory having no effect on eating behavior. d. Accurate body image but self-deprecating attitude.

C (Health professionals who work with adolescents should consider the adolescents' increasing independence and responsibility while maintaining privacy and ensuring confidentiality. However, circumstances may occur in which they are not able to maintain confidentiality, such as self-destructive behavior or maltreatment by others. Confidentiality and privacy are necessary to facilitate trust with this age group. The nurse must be aware of the limits placed on confidentiality by local jurisdiction.)

The school nurse tells adolescents in the clinic that confidentiality and privacy will be maintained unless a life-threatening situation arises. This practice is: a. Not appropriate in a school setting. b. Never appropriate because adolescents are minors. c. Important in establishing trusting relationships. d. Suggestive that the nurse is meeting his or her own needs.

D (Cover the "good" eye completely with a patch.)

The teaching plan for the parents of a 3-year-old child with amblyopia ("lazy eye") should include what instruction? a. Apply a patch to the child's eyeglass lenses. b. Apply a patch only during waking hours. c. Apply a patch over the "bad" eye to strengthen it. d. Cover the "good" eye completely with a patch.

C (Rotavirus is the most frequent viral pathogen that causes diarrhea in young children. Giardia and Salmonella are bacterial pathogens that cause diarrhea. Shigella is a bacterial pathogen that is uncommon in the United States.)

The viral pathogen that frequently causes acute diarrhea in young children is: a. Giardia organisms. b. Shigella organisms. c. Rotavirus. d. Salmonella organisms.

D (Weight loss may be triggered by a typical adolescent crisis such as the onset of menstruation or a traumatic interpersonal incident; situations of severe family stress such as parental separation or divorce; or circumstances in which the young person lacks personal control, such as being teased, changing schools, or entering college. There may in fact be a history of sexual abuse; however, this is not the trigger. These adolescents are often overachievers who are successful in school, not failures in school. The adolescent is most often enmeshed with his or her family.)

The weight loss of anorexia nervosa is often triggered by: a. Sexual abuse. b. School failure. c. Independence from family. d. Traumatic interpersonal conflict.

D (Most children with Hirschsprung's disease require surgical rather than medical management. Surgery is done to remove the aganglionic portion of the bowel, relieve obstruction, and restore normal bowel motility and function of the internal anal sphincter. Preoperative management may include enemas and low-fiber, high-calorie, high-protein diet until the child is physically ready for surgery. The colostomy that is created in Hirschsprung's disease is usually temporary.)

Therapeutic management of most children with Hirschsprung's disease is primarily: a. Daily enemas. b. Low-fiber diet. c. Permanent colostomy. d. Surgical removal of affected section of bowel.

C (ORS is the first treatment for acute diarrhea. Clear liquids are not recommended because they contain too much sugar, which may contribute to diarrhea. Adsorbents are not recommended and neither are antidiarrheals because they do not get rid of pathogens.)

Therapeutic management of the child with acute diarrhea and dehydration usually begins with: a. Clear liquids. b. Adsorbents such as kaolin and pectin. c. Oral rehydration solution (ORS). d. Antidiarrheal medications such as paregoric.

A, B, E (Encourage parents to bring in homework and schedule study times. Allow the adolescent to wear street clothes. Encourage parents to bring in favorite foods.)

What is an age-appropriate nursing intervention to facilitate psychologic adjustment for an adolescent expected to have a prolonged hospitalization (select all that apply)? a. Encourage parents to bring in homework and schedule study times. b. Allow the adolescent to wear street clothes. c. Involve the parents in care. d. Follow home routines. e. Encourage parents to bring in favorite foods.

A (The chronic inflammatory process of Crohn's disease involves any part of the GI tract from the mouth to the anus but most often affects the terminal ileum. Ulcerative colitis, Meckel's diverticulum, and irritable bowel syndrome do not affect the entire GI tract.)

What is characterized by a chronic inflammatory process that may involve any part of the gastrointestinal (GI) tract from mouth to anus? a. Crohn's disease b. Meckel's diverticulum c. Ulcerative colitis d. Irritable bowel syndrome

C (Let the child decide which color arm board to use with the IV.)

What is the best action for the nurse to take when a 5-year-old child who requires another 2 days of intravenous (IV) antibiotics cries, screams, and resists having the IV restarted? a. Exit the room and leave the child alone until he stops crying. b. Tell the child big boys and girls "don't cry." c. Let the child decide which color arm board to use with the IV. d. Administer a narcotic analgesic for pain to quiet the child.

C (Simple dietary modifications are effective in the management of lactose intolerance. Symptoms of lactose intolerance are usually relieved after instituting a lactose-free diet. Medications are not typically ordered in the management of lactose intolerance. Providing emotional support to family members is not specific to this medical condition. Diarrhea is a manifestation of lactose intolerance. Enemas are contraindicated for this alteration in bowel elimination.)

What is the major focus of the therapeutic management for a child with lactose intolerance? a. Compliance with the medication regimen b. Providing emotional support to family members c. Teaching dietary modifications d. Administration of daily normal saline enemas

C (Risk of aspiration is a priority nursing diagnosis for the infant with gastroesophageal reflux. The parents must be taught infant CPR. Correct positioning minimizes the risk for aspiration. The correct position for the infant is on the right side after feeding and supine for sleeping. Knowledge of developmental needs should be included in discharge planning for all hospitalized infants, but it is not the most important in this case. Keeping a record of intake and output is not a priority and may not be necessary.)

What is the most important information to be included in the discharge planning for an infant with gastroesophageal reflux? a. Teach parents to position the infant on the left side. b. Reinforce the parents' knowledge of the infant's developmental needs. c. Teach the parents how to do infant cardiopulmonary resuscitation (CPR). d. Have the parents keep an accurate record of intake and output.

D (The nurse should refer the child with cognitive impairment for speech therapy. It helps improve communication and promotes social behavior of the child. Periodic testing of the thyroid function is done if the child has Down syndrome. Thyroid function is not altered in all children with cognitive impairment. Sexual information is given to adolescents with cognitive impairment. Younger school-age children with cognitive impairment may not understand information regarding sexuality. Self-care skills should be taught by the parents to children with cognitive impairment.)

What nursing care should be provided to a school-aged child with cognitive impairment? A. Periodic testing of thyroid function B. Education on sexuality C. Education on self-care skills for the child D. Speech therapy referral for the child

A (The major stress for children from infancy through the preschool years is separation anxiety, also called anaclitic depression. This is a major stressor of hospitalization. Loss of control, fear of bodily injury, and fear of pain are all stressors associated with hospitalization. However, separation from family is a primary stressor in this age-group.)

What represents the major stressor of hospitalization for children from middle infancy throughout the preschool years? a. Separation anxiety b. Loss of control c. Fear of bodily injury d. Fear of pain

C (A language translator should be used when any patient speaks a foreign language, no matter what his or her cognitive level is. Inserting the patient's hearing aids will help the patient hear, but it will not break the language barrier. Verbal expressions can be helpful, but they are not as effective as having a translator speaking the patient's language. Visual aids and drawings may be helpful, but not everything is easily communicated with these methods.)

What should the nurse do to communicate with a patient who is cognitively impaired and speaks a foreign language? A. Insert the patient's hearing aids. B. Use verbal expressions. C. Use a language translator. D. Use visual aids and drawings.

D (Coping with the stress of chronic illness and the clinical manifestations associated with ulcerative colitis (diarrhea, pain) are important teaching foci. Avoidance of triggers can help minimize the impact of the disease and its effect on the child. Ulcerative colitis is not infectious. Although nutritional guidance is a priority teaching focus, diarrhea is a problem with ulcerative colitis, not constipation. Daily enemas are not part of the therapeutic plan of care.)

What should the nurse stress in a teaching plan for the mother of an 11-year-old boy with ulcerative colitis? a. Preventing the spread of illness to others b. Nutritional guidance and preventing constipation c. Teaching daily use of enemas d. Coping with stress and avoiding triggers

B (Achieves a mental age of 8 to 12 years.)

When a child with mild cognitive impairment reaches the end of adolescence, what characteristic would be expected? a. Achieves a mental age of 5 to 6 years b. Achieves a mental age of 8 to 12 years c. Is unable to progress in functional reading or arithmetic d. Acquires practical skills and useful reading and arithmetic to an eighth-grade level

A (If a toddler is not prepared for hospitalization, a typical preschooler fantasy is to attribute the hospitalization to punishment for real or imagined misdeeds. Threat to child's self-image and loss of companionship with friends are reactions typical of school-age children. Regression is a response characteristic of toddlers when threatened with loss of control.)

When a preschool child is hospitalized without adequate preparation, the nurse should recognize that the child may likely see hospitalization as: a. Punishment. b. Threat to child's self-image. c. An opportunity for regression. d. Loss of companionship with friends.

B, C, D, E (Risk factors for increased stress level of a child to illness or hospitalization: "Difficult" temperament; Lack of fit between child and parent; Age (especially between 6 months and 5 years old); Male gender; Below-average intelligence; Multiple and continuing stresses (e.g., frequent hospitalizations).)

When admitting a child to the inpatient pediatric unit, the nurse should assess for which risk factors that can increase the child's stress level associated with hospitalization? Select all that apply. A. Mild temperament B. Lack of fit between parent and child C. Below-average intelligence D. Age E. Gender

A (This is the most obvious early sign of bleeding from the operative site. Gargling should be avoided because of potential trauma to the suture line. The child should be positioned on the side or abdomen to facilitate drainage. Cold is preferred. Ice collars and cold liquids are encouraged.)

When caring for a child after a tonsillectomy, the nurse should: A. Watch for continuous swallowing. B. Encourage gargling to reduce discomfort. C. Position the child on the back for sleeping. D. Apply warm compresses to the throat

C (Signs of peritonitis, in addition to fever, include sudden relief from pain after perforation. Tachycardia, not bradycardia, is a manifestation of peritonitis. Anorexia is already a clinical manifestation of appendicitis. Abdominal distention usually increases in addition to an increase in pain (usually diffuse and accompanied by rigid guarding of the abdomen).)

When caring for a child with probable appendicitis, the nurse should be alert to recognize that a sign of perforation is: a. Bradycardia. b. Anorexia. c. Sudden relief from pain. d. Decreased abdominal distention.

B, C, E (Suggestions for coping with postpartum blues include: (1) Remember that the "blues" are normal and that both the mother and the father or partner may experience them. (2) Get plenty of rest; nap when the baby does if possible. Go to bed early, and let friends and family know when to visit and how they can help. (Remember, you are not "Supermom.") (3) Use relaxation techniques learned in childbirth classes (or ask the nurse to teach you and your partner some techniques). (4) Do something for yourself. Take advantage of the time your partner or family members care for the baby—soak in the tub (a 20-minute soak can be the equivalent of a 2-hour nap), or go for a walk. (5) Plan a day out of the house—go to the mall with the baby, being sure to take a stroller or carriage, or go out to eat with friends without the baby. Many communities have churches or other agencies that provide child care programs such as Mothers' Morning Out. (6) Talk to your partner about the way you feel—for example, about feeling tied down, how the birth met your expectations, and things that will help you (do not be afraid to ask for specifics). (7) If you are breastfeeding, give yourself and your baby time to learn. (8) Seek out and use community resources such as La Leche League or community mental health centers.)

When helping a woman cope with postpartum blues, what suggestions should the nurse offer? Select all that apply. A. Have the father take over care of the baby, because postpartum blues are exclusively a female problem. B. Get plenty of rest. C. Plan to get out of the house occasionally. D. Do not ask for help because this will not foster independence. e. Use La Leche League or community mental health centers.

C (Vacillating between the desire to have her own nurturing needs met and the need to take charge of her own care and that of her newborn is characteristic of the taking-in stage, which lasts for the first few days after birth. Expressing a strong need to review events and her behavior during the process of labor and birth is characteristic of the taking-in stage, which lasts for the first few days after birth. One week after birth, the woman should exhibit behaviors characteristic of the taking-hold stage. This stage lasts for as long as 4 to 5 weeks after birth. Reestablishing her role as a spouse/partner reflects the letting-go stage, which indicates that psychosocial recovery is complete. Test-Taking Tip: Read the question carefully before looking at the answers: (1) Determine what the question is really asking; look for key words; (2) Read each answer thoroughly and see if it completely covers the material asked by the question; (3) Narrow the choices by immediately eliminating answers you know are incorrect.)

When making a visit to the home of a postpartum woman 1 week after birth, the nurse should recognize that characteristically the woman would: A. Express a strong need to review events and her behavior during the process of labor and birth. B. Exhibit a reduced attention span, limiting readiness to learn. C. Facilitate between the desire to have her own nurturing needs met and the need to take charge of her own care and that of her newborn. D. Have reestablished her role as a spouse/partner.

D (Problems that teenagers identify and are interested in are typically the problems that they are the most willing to address. Confidentiality is important to adolescents. Adolescents prefer to confer privately (without parents) with the nurse and health care provider. Teenagers are socially and cognitively at the developmental stage where the health care provider can teach them. The nurse must keep in mind that teenagers are more interested in immediate health care needs than in long-term needs)

When planning care for adolescents, the nurse should: a. Teach parents first, and they, in turn, will teach the teenager. b. Provide information for their long-term health needs because teenagers respond best to long-range planning. c. Maintain the parents' role by providing explanations for treatment and procedures to the parents only. d. Give information privately to adolescents about how they can manage the specific problems that they identify.

A, B, C, D (The skepticism, open or hidden, of health care professionals presents an additional and unneeded hurdle for the parents. After the parents' capabilities have been assessed (including some the nurse may not have expected), the nurse can help find ways to assist the parents that focus on their strengths. The Internet affords an extra teaching tool for the deaf, as do videos with subtitles or nurses signing. A number of electronic devices can turn sound into light flashes to help pick up a child's cry. Sign language is acquired readily by young children. Childbirth education and other materials are available in Braille. Other sensory output can be provided by the parent, other people can participate, and other coping devices can be used.)

When working with parents who have some form of sensory impairment, what information should nurses consider when writing a plan of care? Select all that apply. A. One of the major difficulties visually impaired parents experience is the skepticism of health care professionals. B. The best approach for the nurse is to assess the parents' capabilities rather than focusing on their disabilities. C. Technologic advances, including the Internet, can provide deaf parents with a full range of parenting activities and information D. Childbirth education and other materials are available in Braille.

C (Encourage parents to leave the child alone for extended periods of time.)

Which action is contraindicated when a child with Down syndrome is hospitalized? a. Determine the child's vocabulary for specific body functions. b. Assess the child's hearing and visual capabilities. c. Encourage parents to leave the child alone for extended periods of time. d. Have meals served at the child's usual mealtimes.

A (Adolescents are no longer restricted to the real and actual. They also are concerned with the possible; they think beyond the present. During concrete operations (between ages 7 and 11 years), children exhibit the ability to place things in a sensible and logical order, the ability to see things from another's point of view, and the ability to make judgments based on what they reason rather than just what they see.)

Which aspect of cognition develops during adolescence? a. Capability to use a future time perspective b. Ability to place things in a sensible and logical order c. Ability to see things from the point of view of another d. Progress from making judgments based on what they see to making judgments based on what they reason

B (During adolescence, energy is focused within. Adolescents concentrate on themselves in an effort to determine who they are or who they will be. Adolescents are likely to be impulsive and impatient. Parents often describe their teenager as being "self-centered" or "lazy." The peer group validates acceptable behavior during adolescence. Adolescents move from family and enjoy spending time with peers. Adolescents also spend time alone; they need this time to think and concentrate on themselves. Conformity becomes less important in late adolescence.)

Which behavior suggests appropriate psychosocial development in the adolescent? a. The adolescent seeks validation for socially acceptable behavior from older adults. b. The adolescent is self-absorbed and self-centered and has sudden mood swings. c. Adolescents move from peers and enjoy spending time with family members. d. Conformity with the peer group increases in late adolescence.

B (In the protest phase, the child aggressively responds to separation from parents (such as clinging to a parent). Inactivity is characteristic of despair. Depression and sadness are characteristics of despair. Regression to earlier behavior is characteristic of despair.)

Which behavior would most likely be manifested in a young child experiencing the protest phase of separation anxiety? A. Inactivity B. Clings to parent C. Depressed, sad D. Regression to earlier behavior

D (Pain is the cardinal feature. It is initially generalized and usually periumbilical. The pain localizes to the right lower quadrant at McBurney's point. Rebound tenderness is not a reliable sign and is extremely painful to the child. Abdominal pain that is relieved by eating and bright or dark red rectal bleeding are not signs of acute appendicitis.)

Which clinical manifestation would most suggest acute appendicitis? a. Rebound tenderness b. Bright red or dark red rectal bleeding c. Abdominal pain that is relieved by eating d. Abdominal pain that is most intense at McBurney's point

B (Drug therapy for TB includes isoniazid, rifampin, and pyrazinamide daily for 2 months and 2 or 3 times a week for the remaining 4 months. Although skin testing and adequate nutrition and hydration are important, pharmacotherapy is the most important intervention for TB.)

Which consideration is the most important in managing tuberculosis (TB) in children? a. Skin testing annually b. Pharmacotherapy c. Adequate nutrition d. Adequate hydration

B, C (Jordanian mothers have a 40-day lying-in after birth, during which their mothers or sisters care for the baby. Japanese mothers rest for the first 2 months after childbirth. Asian mothers must remain at home with the baby up to 30 days after birth and are not supposed to engage in household chores, including care of the baby. Hispanic practice involves many food restrictions after childbirth, such as avoiding fish, pork, and citrus foods. Vietnamese mothers may give minimal care to their babies and refuse to cuddle or further interact with the baby to ward off "evil" spirits.)

Which culturally appropriate beliefs should the maternity nurse use to incorporate parental-infant attachment into the plan of care? Select all that apply. A. Asian mothers are encouraged to return to work as soon as possible. B. Jordanian mothers have a 40-day lying-in after birth. C. Japanese mothers rest for the first 2 months after childbirth. D. Encourage Hispanics to eat plenty of fish and pork to increase vitamin intake. E. Encourage Vietnamese mothers to cuddle with the newborn.

B (Pressure on the bowel from obstruction leads to passage of "currant jelly" stools. Ribbon-like stools are characteristic of Hirschsprung's disease. With intussusception, passage of bloody mucus-coated stools occurs. Stools will not be hard. Loose, foul-smelling stools may indicate infectious gastroenteritis.)

Which description of a stool is characteristic of intussusception? A. Ribbon-like stools B. "Currant jelly" stools C. Hard stools positive for guaiac D. Loose, foul-smelling stools

A, B, C (Carpets retain dust. To reduce exposure to dust, carpeting should be replaced with wood, tile, slate, or vinyl. These floors can be cleaned easily. For anyone with pollen allergies, it is best to keep the windows closed and to run the air conditioner. Covering mattresses and pillows with dust-proof covers will reduce exposure to dust. A humidity level above 60% promotes dust mites. It is recommended that household humidity be kept between 40% and 50% to reduce dust mites inside the house. Keeping pets outside will help to decrease exposure to dander, but will not affect exposure to pollen and dust.)

Which information should the nurse teach families about reducing exposure to pollens and dust (Select all that apply)? a. Replace wall-to-wall carpeting with wood and tile floors. b. Use an air conditioner. c. Put dust-proof covers on pillows and mattresses. d. Keep humidity in the house above 60%. e. Keep pets outside.

D (Meticulous hand washing can decrease the spread of organisms. RSV infection is not airborne. It is acquired mainly through contact with contaminated surfaces. RSV can live on skin or paper for up to 1 hour and on cribs and other nonporous surfaces for up to 6 hours.)

Which information should the nurse teach workers at a day care center about respiratory syncytial virus (RSV)? a. RSV is transmitted through particles in the air. b. RSV can live on skin or paper for up to a few seconds after contact. c. RSV can survive on nonporous surfaces for about 60 minutes. d. Frequent hand washing can decrease the spread of the virus.

C (Taking the child into the cool, humid, night air may relieve mucosal swelling and improve symptoms. Decongestants are inappropriate for croup, which affects the middle airway level. A dry environment may contribute to symptoms. Croup is caused by a virus. Antibiotic treatment is not indicated.)

Which intervention for treating croup at home should be taught to parents? a. Have a decongestant available to give the child when an attack occurs. b. Have the child sleep in a dry room. c. Take the child outside. d. Give the child an antibiotic at bedtime.

D (Offering realistic choices is helpful in meeting the school-age child's sense of control. To facilitate bowel elimination, the child should sit on the toilet for 5 to 10 minutes after breakfast and dinner. Decreasing the amount of sugar in the diet will help keep stools soft. Daily Fleet enemas can result in hypernatremia and hyperphosphatemia, and are used only during periods of fecal impaction.)

Which intervention should be included in the nurse's plan of care for a 7-year-old child with encopresis who has cleared the initial impaction? a. Have the child sit on the toilet for 30 minutes when he gets up in the morning and at bedtime. b. Increase sugar in the child's diet to promote bowel elimination. c. Use a Fleet enema daily. d. Give the child a choice of beverage to mix with a laxative.

A, C, E (The child with hepatitis should be placed on a well-balanced, low-fat diet. Parents should be taught to not give over-the-counter medications because of impaired liver function. Hand hygiene is the most important preventive measure for the spread of hepatitis. The child will be in contact isolation in the hospital, so playtime with other hospitalized children is not scheduled. The child will be on contact isolation for a minimum of 1 week after the onset of jaundice. After that period, the child will be allowed to return to school.)

Which interventions should a nurse implement when caring for a child with hepatitis (Select all that apply)? a. Provide a well-balanced, low-fat diet. b. Schedule playtime in the playroom with other children c. Teach parents not to administer any over-the-counter medications. d. Arrange for home schooling because the child will not be able to return to school. e. Instruct parents on the importance of good hand washing.

B, C, E (Keep items in the room in the same location. Describe the placement of the eating utensils on the meal tray. Identify noises for the child.)

Which interventions should the nurse plan when caring for a child with a visual impairment (select all that apply)? a. Touch the child upon entering the room before speaking. b. Keep items in the room in the same location. c. Describe the placement of the eating utensils on the meal tray. d. Use color examples to describe something to a child who has been blind since birth. e. Identify noises for the child.

B (During growth spurts, the need for sleep is increased. Rapid physical growth, the tendency toward overexertion, and the overall increased activity of this age contribute to fatigue.)

Which predisposes the adolescent to feel an increased need for sleep? a. An inadequate diet b. Rapid physical growth c. Decreased activity that contributes to a feeling of fatigue d. The lack of ambition typical of this age group

B, C, E (b. Vision c. Hearing e. Scoliosis)

Which screening tests should the school nurse perform for the adolescent (select all that apply)? a. Glucose b. Vision c. Hearing d. Cholesterol e. Scoliosis

B (Emergency hospitalization)

Which situation poses the greatest challenge to the nurse working with a child and family? a. Twenty-four-hour observation b. Emergency hospitalization c. Outpatient admission d. Rehabilitation admission

C (Hepatitis A is the most common form of acute hepatitis in most parts of the world. It is characterized by a rapid acute onset. The incubation period is approximately 3 weeks for hepatitis A. The principal mode of transmission for hepatitis A is the fecal-oral route. Hepatitis A does not have a carrier state.)

Which statement best characterizes hepatitis A? a. The incubation period is 6 weeks to 6 months. b. The principal mode of transmission is through the parenteral route. c. Onset is usually rapid and acute. d. There is a persistent carrier state.

B (Children have increased immunity after exposure to a virus. The amount of lymphoid tissue increases as children grow older. Viral organisms are not less prevalent, but older children have the ability to resist invading organisms. Secondary infections after viral illnesses include Mycoplasma pneumoniae and groups A and B streptococcal infections.)

Which statement best describes why children have fewer respiratory tract infections as they grow older? a. The amount of lymphoid tissue decreases. b. Repeated exposure to organisms causes increased immunity. c. Viral organisms are less prevalent in the population. d. Secondary infections rarely occur after viral illnesses.

C (Suicide is the third most common cause of death among American adolescents. A suicide threat from an adolescent serves as a dramatic message to others and should be taken seriously. Adolescents at risk should be targeted for supportive guidance and counseling before a crisis occurs. Instructing a 15-year-old to tell a friend to come to the clinic immediately provides the teen with limited information and does not address the concern. It is important to determine whether a person threatening suicide has a plan of action; however, the best information for the 15-year-old to have is that all threats of suicide should be taken seriously and immediate help is important. Taking time to gather details or waiting until the teen discusses it a second time may be too late.)

Which statement by the nurse is most appropriate to a 15-year-old whose friend has mentioned suicide? a. "Tell your friend to come to the clinic immediately." b. "You need to gather details about your friend's suicide plan." c. "Your friend's threat needs to be taken seriously and immediate help for your friend is important." d. "If your friend mentions suicide a second time, you will want to get your friend some help."

A (CF is an autosomal recessive gene inherited from both parents and is found primarily in Caucasian populations. An autosomal recessive inheritance pattern means that there is a 25% chance that a sibling will be infected but a 50% chance a sibling will be a carrier.)

Which statement expresses accurately the genetic implications of cystic fibrosis (CF)? a. If it is present in a child, both parents are carriers of this defective gene. b. It is inherited as an autosomal dominant trait. c. It is a genetic defect found primarily in non-Caucasian population groups. d. There is a 50% chance that siblings of an affected child also will be affected.

D (Historically AOM has been treated with a range of antibiotics, and it is the most common disorder treated with antibiotics in the ambulatory setting. The etiology of AOM may be bacterial, such as Streptococcus pneumoniae, Haemophilus influenzae, and Moraxella catarrhalis, or a viral agent. Recent concerns about drug-resistant organisms have caused authorities to recommend judicious use of antibiotics and that antibiotics are not required for initial treatment. Permanent hearing loss is not a frequent cause of properly treated AOM. Intramuscular antibiotics are not necessary. Oral amoxicillin is the treatment of choice.)

Which statement is characteristic of acute otitis media (AOM)? a. The etiology is unknown. b. Permanent hearing loss often results. c. It can be treated by intramuscular antibiotics. d. It is treated with a broad range of antibiotics.

A (Heredity is an important fact that contributes to obesity. Identical twins reared apart tend to resemble their biologic parents to a greater extent than their adoptive parents. It is difficult to distinguish between hereditary and environmental factors. The rate of childhood obesity has increased so dramatically that it has now reached epidemic proportions. Inactivity is an important contributing factor; however, obesity is the result of a combination of a number of other factors. Fewer than 5% of all cases of obesity can be linked to underlying disease.)

Which statement is correct about childhood obesity? a. Heredity is an important factor in the development of obesity. b. Childhood obesity in the United States is decreasing. c. Childhood obesity is the result of inactivity. d. Childhood obesity can be attributed to an underlying disease in most cases.

D (Individuals with bulimia are of normal weight or more commonly slightly above normal weight. Those who also restrict their intake can become severely underweight. Behavior related to this eating disorder is secretive, frenzied, and out of control. These cycles are followed by self-deprecating thoughts and a depressed mood. These young women are keenly aware that this eating pattern is abnormal.)

Which statement is most descriptive of bulimia during adolescence? a. Strong sense of control over eating behavior b. Feelings of elation after the binge-purge cycle c. Profound lack of awareness that the eating pattern is abnormal d. Weight that can be normal, slightly above normal, or below normal

B (Central nervous system stimulants such as amphetamines and cocaine produce a strong psychologic dependence. This class of drugs does not produce strong physical dependence and can be withdrawn without much danger. Acute intoxication leads to violent, aggressive behavior or psychotic episodes characterized by paranoia, uncontrollable agitation, and restlessness.)

Which statement is most descriptive of central nervous system stimulants? a. They produce strong physical dependence. b. They can result in strong psychologic dependence. c. Withdrawal symptoms are life threatening. d. Acute intoxication can lead to coma.

D (Symptoms of anorexia nervosa include lower body temperature, severe weight loss, decreased blood pressure, dry skin, brittle nails, altered metabolic activity, and presence of lanugo hair. Amenorrhea, rather than dysmenorrhea, and cold intolerance are manifestations of anorexia nervosa. Bradycardia, rather than tachycardia, may be present.)

Which symptoms should the nurse expect to observe during the physical assessment of an adolescent girl with severe weight loss and disrupted metabolism associated with anorexia nervosa? a. Dysmenorrhea and oliguria b. Tachycardia and tachypnea c. Heat intolerance and increased blood pressure d. Lowered body temperature and brittle nails

C (Wear eye protection when participating in high-risk sports such as paintball.)

Which teaching guideline helps prevent eye injuries during sports and play activities? a. Restrict helmet use to those who wear eyeglasses or contact lenses. b. Discourage the use of goggles with helmets. c. Wear eye protection when participating in high-risk sports such as paintball. d. Wear a face mask when playing any sport or playing roughly.

C (Cataract)

Which term refers to opacity of the crystalline lens that prevents light rays from entering the eye and refracting on the retina? a. Myopia b. Amblyopia c. Cataract d. Glaucoma

B (Epiglottitis is always a medical emergency needing antibiotics and airway support for treatment. Laryngitis is a common viral illness in older children and adolescents, with hoarseness and upper respiratory infection symptoms. Spasmodic croup is treated with humidity. LTB may progress to a medical emergency in some children.)

Which type of croup is always considered a medical emergency? a. Laryngitis b. Epiglottitis c. Spasmodic croup d. Laryngotracheobronchitis (LTB)

D (Hypertonic dehydration results from water loss in excess of electrolyte loss. This is the most dangerous type of dehydration. It is caused by feeding children fluids with high amounts of solute. Isotonic dehydration occurs in conditions in which electrolyte and water deficits are present in balanced proportion. Isosmotic dehydration is another term for isotonic dehydration. Hypotonic dehydration occurs when the electrolyte deficit exceeds the water deficit, leaving the serum hypotonic.)

Which type of dehydration results from water loss in excess of electrolyte loss? a. Isotonic dehydration b. Hypotonic dehydration c. Isosmotic dehydration d. Hypertonic dehydration

D (A strangulated hernia is one in which the blood supply to the herniated organ is impaired. A hiatal hernia is the intrusion of an abdominal structure, usually the stomach, through the esophageal hiatus. An incarcerated hernia is a hernia that cannot be reduced easily. Omphalocele is the protrusion of intraabdominal viscera into the base of the umbilical cord. The sac is covered with peritoneum and not skin.)

Which type of hernia has an impaired blood supply to the herniated organ? a. Hiatal hernia b. Omphalocele c. Incarcerated hernia d. Strangulated hernia

C (Universal vaccination for hepatitis B is now recommended for all newborns. A vaccine is available for hepatitis A, but it is not yet universally recommended. No vaccine is currently available for hepatitis C. Only hepatitis B vaccine is recommended for newborns.)

Which vaccine is now recommended for the immunization of all newborns? a. Hepatitis A vaccine b. Hepatitis C vaccine c. Hepatitis B vaccine d. Hepatitis A, B, and C vaccines

D (Fat-soluble vitamins are poorly absorbed because of deficient pancreatic enzymes in children with cystic fibrosis; therefore, supplements are necessary. Vitamin C and calcium are not fat soluble. Vitamins B6 and B12are not fat-soluble vitamins.)

Which vitamin supplements are necessary for children with cystic fibrosis? a. Vitamin C and calcium b. Vitamins B6 and B12 c. Magnesium d. Vitamins A, D, E, and K

D (According to traditional Hispanic customs, fathers do not see their wives or infants until they are cleaned and dressed. It is believed that this custom incorporates the individuals into the family and integrates the family. It does not imply that the father is too scared to hold the baby. In this situation, the father is following the traditional Hispanic custom by not seeing the baby after birth. It does not implicate that the father has no time to see the baby. The father would want to see the baby as soon as it is cleaned and dressed.)

While caring for a Hispanic patient, the nurse asks the infant's father to hold the baby after its birth. The father is unwilling to see the baby. What reason would the nurse attribute to this unwillingness? The father: A. Is too scared to hold the baby. B. Has no time to visit the baby. C. Is not following the traditional Hispanic custom. D. Is doing it to integrate the individuals into the family

D (While no family follows just one pattern, in American culture the paternal grandparents are typically considered secondary to the maternal grandparents after the baby is born. This is because a woman may be closer to her own parents and feel more comfortable accepting their help than that of her partner's family. As a result, paternal grandparents may be naturally less involved with the newborn. There are no cultural rules in the American culture dictating that paternal grandparents care for the second-born child or bond with the baby only after 6 months. Grandparents are not typically responsible for the financial expenses of the baby in the United States.)

While caring for a patient, the nurse notices that the newborn's paternal grandparents visit the newborn and leave after an hour. The maternal grandparents, on the other hand, stay all day with the couple to look after the newborn. What could be the reason for this in terms of traditional paternal grandparents' cultural behaviors? A. They care for the second born child. B. Typically, they are called to care for baby after 6 months. C. They feel that they are responsible only for the financial expenses of the child. D. They do not feel responsible for the care of the newborn or new mother.

B (Children may feel insecure when parents pay more attention to a newborn sibling. The older child may develop feelings of sibling rivalry and exhibit negative behaviors, such as increased attention seeking and aggression toward the baby. Therefore trying to grab the attention of the parents indicates that the child has not adapted well to the newborn. Positive behaviors, such as increased interest and concern for the baby, indicate effective adaptation. Therefore expressing a desire to stay with the mother and newborn in hospital, watching the baby silently, and being willing to feed the baby does not indicate a lack of adaptation.)

While caring for a postpartum patient, the nurse observes that the patient's older child is not adapting well to the newborn and may have feelings of sibling rivalry. What behavior of the older child would make the nurse conclude this? A. Watches the baby silently in the hospital B. Always tries to grab the parents' attention C. Always wants to be with the mother and newborn D. Asks to feed the newborn with a milk bottle

C (Algerian mothers tightly wrap the infant in swaddling clothes to protect them physically and physiologically, a custom followed by Algerians as a part of the acquaintance process. Algerians follow this custom to give physical and psychological protection but not to protect the infant from evil spirits. Vietnamese mothers interact minimally with the infants to protect them from evil spirits. The mother has to accept the assistance from others, as she alone cannot take care of the baby. Therefore she cannot avoid others touching the infant. Asian and Jordanian mothers hand over the baby to the grandparents immediately after birth. They do this to get some rest after childbirth.)

While caring for an Algerian patient, the nurse suggests that the patient give kangaroo care to the infant. The mother is unwilling to follow the suggestion of the nurse. What is the reason for this unwillingness? The patient is attempting to: A. Protect the infant from evil spirits. B. Prevent others from touching the baby. C. Protect the infant physically and psychologically. D. Take adequate rest after the birth of the child.

B (Postpartum women spend most of their time caring for the infant, especially those mothers who breastfeed and may not easily be able to share child care duties with their partners. As a result, the infant's father may feel depressed, bored, and jealous of the infant. The infant's father should express these feelings so that the mother can make more time for her partner. It is not appropriate to tell the father to spend more time with his friends, as this does not solve the problem in the home. Even if the father comes home early from work, the mother may not be able to spend time with the father, so this suggestion is not necessarily helpful. Engaging in other activities may relieve the infant's father from boredom but not from depression and jealousy. The father would get relieved of these feelings by having the mother's attention and care.)

While speaking to the newborn's father in a well child visit, the nurse finds out that the newborn's father is feeling depressed and bored, as the mother spends most of her time with the infant. What instruction should the nurse give to the father in this situation? A. "Spend more time with your friends." B. "Share your feelings with your partner." C. "Work fewer hours and come home early." D. "Try to distract yourself with other activities."

C (Patting or gently rubbing the infant's back provides comfort to the infant after feeding. Gentle stroking on the infant's back helps to soothe and quiet the infant. Making eye contact with the infant helps to develop a bond and enhances the trust between the mother and infant. Touching the infant with the mother's fingertips and caressing the infant's trunk help to develop attachment with the infant.)

While teaching the patient about breastfeeding, the nurse instructs the patient to rub the infant's back after feeding. What is the reason for this instruction? A. To develop trust in the infant B. To quiet and soothe the infant C. To provide comfort to the infant D. To promote attachment with the infant

D (The airway in infants and young children is narrower, not wider, and respiratory distress can occur quickly because mucus and edema can cause obstruction to their small airways. Sucking is not necessarily related to problems with the airway. The gag reflex is necessary to prevent aspiration. It does not produce mucus.)

Why do infants and young children quickly have respiratory distress in acute and chronic alterations of the respiratory system? a. They have a widened, shorter airway. b. There is a defect in their sucking ability. c. The gag reflex increases mucus production. d. Mucus and edema obstruct small airways.

C (If the parents cannot stay with the child in the hospital, the nurse may ask the parents to leave an article such as a blanket or toy from home. This is because young children associate such inanimate objects with significant people, and they gain comfort and reassurance from these possessions. When a child is frightened, the nurse should provide physical contact to ease the child. If the child is allergic to the linens at the hospital, it would be the hospital's responsibility to find alternative bedding. There are plenty of blankets available in the hospital, so the parents would not bring the blanket to keep the child warm.)

Why does the nurse ask the parents of a hospitalized child to bring the child's blanket from home? A. To alleviate any fears in the child B. To decrease any allergic reactions C. To provide comfort for the child D. To keep the child warm at night

C (Extended contact is especially important for adolescents and low-income women because they are at risk for parenting inadequacies.)

With regard to parents' early and extended contact with their infant and the relationships built, nurses should be aware that: A. Immediate contact is essential for the parent-child relationship. B. Skin-to-skin contact is preferable to contact with the body totally wrapped in a blanket. C. Extended contact is especially important for adolescents and low-income women because they are at risk for parenting inadequacies. D. Mothers need to take precedence over their partners and other family matters.

B (Participation in preparation classes helps both siblings and grandparents.)

With regard to the adaptation of other family members, mainly siblings and grandparents, to the newborn, nurses should be aware that: A. Sibling rivalry cannot be dismissed as overblown psychobabble; negative feelings and behaviors can take a long time to blow over. B. Participation in preparation classes helps both siblings and grandparents. C. In the United States, paternal and maternal grandparents consider themselves of equal importance and status. D. Since 1990, the number of grandparents providing permanent care to their grandchildren has been declining.

A thrombosis results from the formation of a blood clot or clots inside a blood vessel and is caused by inflammation or partial obstruction of the vessel. Three thromboembolic conditions are of concern during the postpartum period and include all except: a) Amniotic fluid embolism (AFE) b) Superficial venous thrombosis c) Deep vein thrombosis d) Pulmonary embolism

a) Amniotic fluid embolism (AFE)

Early postpartum hemorrhage is defined as a blood loss greater than: a. 500 mL in the first 24 hours after vaginal delivery. b. 750 mL in the first 24 hours after vaginal delivery. c. 1000 mL in the first 48 hours after cesarean delivery. d. 1500 mL in the first 48 hours after cesarean delivery.

a. 500 mL in the first 24 hours after vaginal delivery.

Which measure may prevent mastitis in the breastfeeding mother? a. Initiating early and frequent feedings b. Nursing the infant for 5 minutes on each breast c. Wearing a tight-fitting bra d. Applying ice packs before feeding

a. Initiating early and frequent feedings Early and frequent feedings prevent stasis of milk, which contributes to engorgement and mastitis.

The nurse should expect medical intervention for subinvolution to include: a. oral methylergonovine maleate (Methergine) for 48 hours. b. oxytocin intravenous infusion for 8 hours. c. oral fluids to 3000 mL/day. d. intravenous fluid and blood replacement.

a. oral methylergonovine maleate (Methergine) for 48 hours. Methergine provides long-sustained contraction of the uterus.

A white blood cell (WBC) count of 28,000 cells/mm3 on the morning of the first postpartum day indicates: a. possible infection. b. normal WBC limit. c. serious infection. d. suspicion of sexually transmitted disease.

a. possible infection. A WBC count in the upper ranges of normal (20,000 to 30,000) may indicate an infection.

B (These medications work to dilate the bronchioles in acute exacerbations. These medications do not liquefy secretions or reduce infection. Corticosteroids and mast cell stabilizers reduce inflammation in the lungs.)

b-Adrenergic agonists and methylxanthines are often prescribed for a child with an asthma attack. What is their action? a. Liquefy secretions b. Dilate the bronchioles c. Reduce inflammation of the lungs d. Reduce infection

A postpartum client would be at increased risk for postpartum hemorrhage if she delivered a: a. 5-lb, 2-oz infant with outlet forceps. b. 6.5-lb infant after a 2-hour labor. c. 7-lb infant after an 8-hour labor. d. 8-lb infant after a 12-hour labor.

b. 6.5-lb infant after a 2-hour labor. A rapid labor and delivery may cause exhaustion of the uterine muscle and prevent contraction.

A woman delivered a 9-lb, 10-oz baby 1 hour ago. When you arrive to perform her 15-minute assessment, she tells you that she "feels all wet underneath." You discover that both pads are completely saturated and that she is lying in a 6-inch-diameter puddle of blood. What is your first action? a. Call for help. b. Assess the fundus for firmness. c. Take her blood pressure. d. Check the perineum for lacerations.

b. Assess the fundus for firmness. Firmness of the uterus is necessary to control bleeding from the placental site. The nurse should first assess for firmness and massage the fundus as indicated.

What instructions should be included in the discharge teaching plan to assist the client in recognizing early signs of complications? a. Palpate the fundus daily to ensure that it is soft. b. Notify the physician of any increase in the amount of lochia or a return to bright red bleeding. c. Report any decrease in the amount of brownish red lochia. d. The passage of clots as large as an orange can be expected.

b. Notify the physician of any increase in the amount of lochia or a return to bright red bleeding.

The visiting nurse must be aware that women who have had a postpartum hemorrhage are subject to a variety of complications after discharge from the hospital. These include: (Choose those that apply.) a. dehydration. b. anemia. c. exhaustion. d. failure to attach to her infant. e. postpartum infection.

b. anemia. c. exhaustion. d. failure to attach to her infant. e. postpartum infection. Postpartum hemorrhage often results in anemia, and iron therapy may need to be initiated. Exhaustion is common after hemorrhage. It may take the new mother weeks to feel like herself again. Fatigue may interfere with normal parent-infant bonding and attachment processes. The mother is likely to require assistance with housework and infant care. Excessive blood loss increases the risk for infection.

The client who is being treated for endometritis is placed in Fowler's position because it: a. promotes comfort and rest. b. facilitates drainage of lochia. c. prevents spread of infection to the urinary tract. d. decreases tension on the reproductive organs.

b. facilitates drainage of lochia. Lochia and infectious material are eliminated by gravity drainage.

The nurse knows that a measure for preventing late postpartum hemorrhage is to: a. administer broad-spectrum antibiotics. b. inspect the placenta after delivery. c. manually remove the placenta. d. pull on the umbilical cord to hasten the delivery of the placenta.

b. inspect the placenta after delivery. If a portion of the placenta is missing, the clinician can explore the uterus, locate the missing fragments, and remove the potential cause of late postpartum hemorrhage.

A steady trickle of bright red blood from the vagina in the presence of a firm fundus suggests: a. uterine atony. b. lacerations of the genital tract. c. perineal hematoma. d. infection of the uterus.

b. lacerations of the genital tract. Undetected lacerations will bleed slowly and continuously. Bleeding from lacerations is uncontrolled by uterine contraction.

A multiparous woman is admitted to the postpartum unit after a rapid labor and birth of a 4000 g infant. Her fundus is boggy, lochia is heavy, and vital signs are unchanged. The nurse has the woman void and massages her fundus, but her fundus remains difficult to find, and the rubra lochia remains heavy. The nurse should: a. continue to massage the fundus. b. notify the physician. c. recheck vital signs. d. insert a Foley catheter.

b. notify the physician. Treatment of excessive bleeding requires the collaboration of the physician and the nurses. Do not leave the client alone.

If the nurse suspects a uterine infection in the postpartum client, she should assess the: a. pulse and blood pressure. b. odor of the lochia. c. episiotomy site. d. abdomen for distention.

b. odor of the lochia. An abnormal odor of the lochia indicates infection in the uterus.

Herbal remedies have been used with some success to control PPH after initial management. Some herbs have homeostatic actions, whereas others work as oxytocic agents to contract the uterus. ________________ is a commonly used oxytocic herbal remedy. a) Witch hazel b) Lady's mantel c) Blue cohosh d) Yarrow

c) Blue cohosh

Which nursing measure would be appropriate to prevent thrombophlebitis in the recovery period after a cesarean birth? a. Roll a bath blanket and place it firmly behind the knees. b. Limit oral intake of fluids for the first 24 hours. c. Assist the client in performing leg exercises every 2 hours. d. Ambulate the client as soon as her vital signs are stable.

c. Assist the client in performing leg exercises every 2 hours. Leg exercises promote venous blood flow and prevent venous stasis while the client is still on bed rest.

A mother with mastitis is concerned about breastfeeding while she has an active infection. The nurse should explain that: a. the infant is protected from infection by immunoglobulins in the breast milk. b. the infant is not susceptible to the organisms that cause mastitis. c. the organisms that cause mastitis are not passed to the milk. d. the organisms will be inactivated by gastric acid.

c. the organisms that cause mastitis are not passed to the milk. The organisms are localized in the breast tissue and are not excreted in the breast milk.

Which statement by a postpartal woman indicates that further teaching is not needed regarding thrombus formation? a. "I'll stay in bed for the first 3 days after my baby is born." b. "I'll keep my legs elevated with pillows." c. "I'll sit in my rocking chair most of the time." d. "I'll put my support stockings on every morning before rising."

d. "I'll put my support stockings on every morning before rising." Venous congestion begins as soon as the woman stands up. The stockings should be applied before she rises from the bed in the morning.

Which temperature indicates the presence of postpartum infection? a. 99.6° F in the first 48 hours b. 100° F for 2 days postpartum c. 100.4° F in the first 24 hours d. 100.8° F on the second and third postpartum days

d. 100.8° F on the second and third postpartum days A temperature elevation to greater than 100.4° F on 2 postpartum days not including the first 24 hours indicates infection.

18. Maternity nurses often have to answer questions about the many, sometimes unusual ways people have tried to make the birthing experience more comfortable. For instance, nurses should be aware that: a. music supplied by the support person has to be discouraged because it could disturb others or upset the hospital routine. b. women in labor can benefit from sitting in a bathtub, but they must limit immersion to no longer than 15 minutes at a time. c. effleurage is permissible, but counterpressure is almost always counterproductive. d. electrodes attached to either side of the spine to provide high-intensity electrical impulses facilitate the release of endorphins.

ANS: D Transcutaneous electrical nerve stimulation does help. Music may be very helpful for reducing tension and certainly can be accommodated by the hospital. Women can stay in a bath as long as they want, although repeated baths with breaks may be more effective than a long soak. Counterpressure can help the woman cope with lower back pain.

A woman who is 39 weeks pregnant expresses fear about her impending labor and how she will manage. The nurse's best response is: a. "Don't worry about it. You'll do fine." b. "It's normal to be anxious about labor. Let's discuss what makes you afraid." c. "Labor is scary to think about, but the actual experience isn't." d. "You can have an epidural. You won't feel anything."

B ("It's normal to be anxious about labor. Let's discuss what makes you afraid" allows the woman to share her concerns with the nurse and is a therapeutic communication tool. "Don't worry about it. You'll do fine" negates the woman's fears and is not therapeutic. "Labor is scary to think about, but the actual experience isn't" negates the woman's fears and offers a false sense of security. It is not true that every woman may have an epidural. A number of criteria must be met for use of an epidural. Furthermore, many women still experience the feeling of pressure with an epidural.)

The nurse restricts the visitors of a pregnant patient and gives a specific time for the patient to rest and sleep after the labor. What maternal patient experience could be the probable reason for this nursing action? A. Severe pain during labor B. Severe fatigue during labor C. Ineffective birth process D. Problem of irregular urination

B (The patient may have severe fatigue after labor due to depletion of energy. In order to restore the energy levels, the nurse gives a specific time for the patient to rest and sleep by restricting the visitors. Severe pain, inefficiency in the birth process, and a problem of irregular urination are not the reason for the nurse to limit visitors. The nurse would administer analgesics or anesthesia on an order if the patient experienced acute pain. The nurse would provide comfort measures if the patient was ineffective in the birth process. The nurse would palpate the patient's bladder if irregular urination were a concern.)

While caring for a multiparous patient in the second stage of labor, the patient reports the urge to defecate. What is the best nursing intervention? A. Provide a bedpan to the patient to defecate. B. Place an enema in the rectum of the patient. C. Assess cervical dilation and station of the patient. D. Use running water to stimulate defection for the patient.

C (A multiparous patient feels an urge to defecate in the second stage of labor due to rectal pressure by the deeply descending presenting part in the pelvis. Rectal pressure may occur even in the absence of stool in the anorectal area. This often means that the patient is about to give birth to the child. Therefore the nurse has to perform vaginal examination of the patient to assess cervical dilation and station. The patient does not really defecate, so providing a bedpan is not necessary. Placing an enema in the rectum of the patient is not a suitable intervention, as it is done to increase peristalsis. Running water is used to stimulate voiding for the patient if there is a risk of urinary elimination. However, it is unrelated to the patient's urge of defecation.)

The nurse teaches a pregnant woman about the characteristics of true labor contractions. The nurse evaluates the woman's understanding of the instructions when she states, "True labor contractions will: a. Subside when I walk around." b. Cause discomfort over the top of my uterus." c. Continue and get stronger even if I relax and take a shower." d. Remain irregular but become stronger."

C (True labor contractions occur regularly, becoming stronger, lasting longer, and occurring closer together. They may become intense during walking and continue despite comfort measures. Typically true labor contractions are felt in the lower back, radiating to the lower portion of the abdomen. During false labor, contractions tend to be irregular and felt in the abdomen above the navel. Typically the contractions often stop with walking or a change of position.)

Which collection of risk factors most likely would result in damaging lacerations (including episiotomies)? a. A dark-skinned woman who has had more than one pregnancy, who is going through prolonged second-stage labor, and who is attended by a midwife b. A reddish-haired mother of two who is going through a breech birth c. A dark-skinned, first-time mother who is going through a long labor d. A first-time mother with reddish hair whose rapid labor was overseen by an obstetrician

D (Reddish-haired women have tissue that is less distensible than that of darker-skinned women and therefore may have less efficient healing. First time mothers are also more at risk, especially with breech births, long second-stage labors, or rapid labors in which there is insufficient time for the perineum to stretch. The rate of episiotomies is higher when obstetricians rather than midwives attend births.)

The nurse assesses a pregnant patient and finds that the patient has reduced strength of uterine contractions (UCs). Upon further assessment, the nurse suspects that the patient may have slow progress in labor. Which statement made by the patient indicates the reason for slow progress in labor? A. "I have a family history of diabetes and hypertension." B. "I stopped taking folic acid supplements a week ago." C. "I have been on a diet with high amounts of protein for 15 days." D. "I am worried a lot this time; I had a lot of problems in my last labor."

D (The nurse suspects that the patient may have slow progress in labor after knowing that the patient is worried and stressed, because she had complications in the previous labor. Stress may reduce the progress in the labor by decreasing the levels of catecholamines. This, in turn, reduces the UCs. Family history of diabetes does not affect the labor progression or UCs. Folic acid supplements are necessary for fetal growth and are given early in pregnancy to prevent neural tube defects. They do not affect the birth process. Taking a diet with a high amount of protein may not affect the onset of labor. Moreover, it helps in the fetal growth and development.)

In documenting labor experiences, nurses should know that a uterine contraction is described according to all these characteristics except: a. Frequency (how often contractions occur). b. Intensity (the strength of the contraction at its peak). c. Resting tone (the tension in the uterine muscle). d. Appearance (shape and height).

D (Uterine contractions are described in terms of frequency, intensity, duration, and resting tone.)

2. Nursing care measures are commonly offered to women in labor. Which nursing measure reflects application of the gate-control theory? a. Massaging the woman's back b. Changing the woman's position c. Giving the prescribed medication d. Encouraging the woman to rest between contractions

ANS: A According to the gate-control theory, pain sensations travel along sensory nerve pathways to the brain, but only a limited number of sensations, or messages, can travel through these nerve pathways at one time. Distraction techniques such as massage or stroking, music, focal points, and imagery reduce or completely block the capacity of nerve pathways to transmit pain. These distractions are thought to work by closing down a hypothetic gate in the spinal cord and thus preventing pain signals from reaching the brain. The perception of pain is thereby diminished. Changing the woman's position, giving prescribed medication, and encouraging rest do not reduce or block the capacity of nerve pathways to transmit pain using the gate-control theory.

20. With regard to nerve block analgesia and anesthesia, nurses should be aware that: a. most local agents are related chemically to cocaine and end in the suffix -caine. b. local perineal infiltration anesthesia is effective when epinephrine is added, but it can be injected only once. c. a pudendal nerve block is designed to relieve the pain from uterine contractions. d. a pudendal nerve block, if done correctly, does not significantly lessen the bearing-down reflex.

ANS: A Common agents include lidocaine and chloroprocaine. Injections can be repeated to prolong the anesthesia. A pudendal nerve block relieves pain in the vagina, vulva, and perineum but not the pain from uterine contractions, and it lessens or shuts down the bearing-down reflex.

7. A woman is experiencing back labor and complains of intense pain in her lower back. An effective relief measure would be to use: a. counterpressure against the sacrum. b. pant-blow (breaths and puffs) breathing techniques. c. effleurage. d. conscious relaxation or guided imagery.

ANS: A Counterpressure is a steady pressure applied by a support person to the sacral area with the fist or heel of the hand. This technique helps the woman cope with the sensations of internal pressure and pain in the lower back. The pain-management techniques of pant-blow, effleurage, and conscious relaxation or guided imagery are usually helpful for contractions per the gate-control theory.

17. With regard to breathing techniques during labor, maternity nurses should understand that: a. breathing techniques in the first stage of labor are designed to increase the size of the abdominal cavity to reduce friction. b. by the time labor has begun, it is too late for instruction in breathing and relaxation. c. controlled breathing techniques are most difficult near the end of the second stage of labor. d. the patterned-paced breathing technique can help prevent hyperventilation.

ANS: A First-stage techniques promote relaxation of abdominal muscles, thereby increasing the size of the abdominal cavity. Instruction in simple breathing and relaxation techniques early in labor is possible and effective. Controlled breathing techniques are most difficult in the transition phase at the end of the first stage of labor when the cervix is dilated 8 to 10 cm. Patterned-paced breathing sometimes can lead to hyperventilation.

15. Nurses should be aware of the differences experience can make in labor pain such as: a. sensory pain for nulliparous women often is greater than for multiparous women during early labor. b. affective pain for nulliparous women usually is less than for multiparous women throughout the first stage of labor. c. women with a history of substance abuse experience more pain during labor. d. multiparous women have more fatigue from labor and therefore experience more pain.

ANS: A Sensory pain is greater for nulliparous women because their reproductive tract structures are less supple. Affective pain is higher for nulliparous women during the first stage but decreases for both nulliparous and multiparous during the second stage. Women with a history of substance abuse experience the same amount of pain as those without such a history. Nulliparous women have longer labors and therefore experience more fatigue.

11. To help patients manage discomfort and pain during labor, nurses should be aware that: a. the predominant pain of the first stage of labor is the visceral pain located in the lower portion of the abdomen. b. referred pain is the extreme discomfort between contractions. c. the somatic pain of the second stage of labor is more generalized and related to fatigue. d. pain during the third stage is a somewhat milder version of the second stage.

ANS: A This pain comes from cervical changes, distention of the lower uterine segment, and uterine ischemia. Referred pain occurs when the pain that originates in the uterus radiates to the abdominal wall, lumbosacral area of the back, iliac crests, and gluteal area. Second-stage labor pain is intense, sharp, burning, and localized. Third-stage labor pain is similar to that of the first stage.

3. While developing an intrapartum care plan for the patient in early labor, it is important that the nurse recognize that psychosocial factors may influence a woman's experience of pain. These include: (Select all that apply.) a. culture. b. anxiety and fear. c. previous experiences with pain. d. intervention of caregivers. e. support systems.

ANS: A, B, C, E Culture: A woman's sociocultural roots influence how she perceives, interprets, and responds to pain during childbirth. Some cultures encourage loud and vigorous expressions of pain, whereas others value self-control. The nurse should avoid praising some behaviors (stoicism) while belittling others (noisy expression). Anxiety and fear: Extreme anxiety and fear magnify sensitivity to pain and impair a woman's ability to tolerate it. Anxiety and fear increase muscle tension in the pelvic area, which counters the expulsive forces of uterine contractions and pushing efforts. Previous experiences with pain: Fear and withdrawal are a natural response to pain during labor. Learning about these normal sensations ahead of time helps a woman suppress her natural reactions of fear regarding the impending birth. If a woman previously had a long and difficult labor, she is likely to be anxious. She may also have learned ways to cope and may use these skills to adapt to the present labor experience. Support systems: An anxious partner is less able to provide help and support to a woman during labor. A woman's family and friends can be an important source of support if they convey realistic and positive information about labor and delivery. Although the intervention of caregivers may be necessary for the well-being of the woman and her fetus, some interventions add discomfort to the natural pain of labor (i.e., fetal monitor straps, intravenous lines).

14. With regard to a pregnant woman's anxiety and pain experience, nurses should be aware that: a. even mild anxiety must be treated. b. severe anxiety increases tension, which increases pain, which in turn increases fear and anxiety, and so on. c. anxiety may increase the perception of pain, but it does not affect the mechanism of labor. d. women who have had a painful labor will have learned from the experience and have less anxiety the second time because of increased familiarity.

ANS: B Anxiety and pain reinforce each other in a negative cycle. Mild anxiety is normal for a woman in labor and likely needs no special treatment other than the standard reassurances. Anxiety increases muscle tension and ultimately can build sufficiently to slow the progress of labor. Unfortunately, an anxious, painful first labor is likely to carry over, through expectations and memories, into an anxious and painful experience in the second pregnancy.

23. After change-of-shift report the nurse assumes care of a multiparous patient in labor. The woman is complaining of pain that radiates to her abdominal wall, lower back, and buttocks and down her thighs. Before implementing a plan of care, the nurse should understand that this type of pain is: a. visceral. b. referred. c. somatic. d. afterpain.

ANS: B As labor progresses the woman often experiences referred pain. This occurs when pain that originates in the uterus radiates to the abdominal wall, the lumbosacral area of the back, the gluteal area, and thighs. The woman usually has pain only during a contraction and is free from pain between contractions. Visceral pain is that which predominates in the first stage of labor. This pain originates from cervical changes, distention of the lower uterine segment, and uterine ischemia. Visceral pain is located over the lower portion of the abdomen. Somatic pain is described as intense, sharp, burning, and well localized. This results from stretching of the perineal tissues and the pelvic floor. This occurs during the second stage of labor. Pain experienced during the third stage of labor or afterward during the early after birth period is uterine. This pain is very similar to that experienced in the first stage of labor.

24. It is important for the nurse to develop a realistic birth plan with the pregnant woman in her care. The nurse can explain that a major advantage of nonpharmacologic pain management is: a. greater and more complete pain relief is possible. b. no side effects or risks to the fetus are involved. c. the woman remains fully alert at all times. d. a more rapid labor is likely.

ANS: B Because nonpharmacologic pain management does not include analgesics, adjunct drugs, or anesthesia, it is harmless to the mother and the fetus. There is less pain relief with nonpharmacologic pain management during childbirth. The woman's alertness is not altered by medication; however, the increase in pain will decrease alertness. Pain management may or may not alter the length of labor. At times when pain is decreased, the mother relaxes and labor progresses at a quicker pace.

19. With regard to systemic analgesics administered during labor, nurses should be aware that: a. systemic analgesics cross the maternal blood-brain barrier as easily as they do the fetal blood-brain barrier. b. effects on the fetus and newborn can include decreased alertness and delayed sucking. c. intramuscular (IM) administration is preferred over intravenous (IV) administration. d. IV patient-controlled analgesia (PCA) results in increased use of an analgesic.

ANS: B Effects depend on the specific drug given, the dosage, and the timing. Systemic analgesics cross the fetal blood-brain barrier more readily than the maternal blood-brain barrier. IV administration is preferred over IM administration because the drug acts faster and more predictably. PCA results in decreased use of an analgesic.

21. With regard to spinal and epidural (block) anesthesia, nurses should know that: a. this type of anesthesia is commonly used for cesarean births but is not suitable for vaginal births. b. a high incidence of after-birth headache is seen with spinal blocks. c. epidural blocks allow the woman to move freely. d. spinal and epidural blocks are never used together.

ANS: B Headaches may be prevented or mitigated to some degree by a number of methods. Spinal blocks may be used for vaginal births, but the woman must be assisted through labor. Epidural blocks limit the woman's ability to move freely. Combined use of spinal and epidural blocks is becoming increasingly popular.

9. The role of the nurse with regard to informed consent is to: a. inform the patient about the procedure and have her sign the consent form. b. act as a patient advocate and help clarify the procedure and the options. c. call the physician to see the patient. d. witness the signing of the consent form.

ANS: B Nurses play a part in the informed consent process by clarifying and describing procedures or by acting as the woman's advocate and asking the primary health care provider for further explanations. The physician is responsible for informing the woman of her options, explaining the procedure, and advising the patient about potential risk factors. The physician must be present to explain the procedure to the patient. However, the nurse's responsibilities go further than simply asking the physician to see the patient. The nurse may witness the signing of the consent form. However, depending on the state's guidelines, the woman's husband or another hospital health care employee may sign as witness.

10. A first-time mother is concerned about the type of medications she will receive during labor. She is in a fair amount of pain and is nauseous. In addition, she appears to be very anxious. You explain that opioid analgesics are often used with sedatives because: a. "The two together work the best for you and your baby." b. "Sedatives help the opioid work better, and they also will assist you to relax and relieve your nausea." c. "They work better together so you can sleep until you have the baby." d. "This is what the doctor has ordered for you."

ANS: B Sedatives can be used to reduce the nausea and vomiting that often accompany opioid use. In addition, some ataractics reduce anxiety and apprehension and potentiate the opioid analgesic affects. A potentiator may cause the two drugs to work together more effectively, but it does not ensure maternal or fetal complications will not occur. Sedation may be a related effect of some ataractics, but it is not the goal. Furthermore, a woman is unlikely to be able to sleep through transitional labor and birth. "This is what the doctor has ordered for you" may be true, but it is not an acceptable comment for the nurse to make.

5. The nurse should be aware that an effective plan to achieve adequate pain relief without maternal risk is most effective if: a. the mother gives birth without any analgesic or anesthetic. b. the mother and family's priorities and preferences are incorporated into the plan. c. the primary health care provider decides the best pain relief for the mother and family. d. the nurse informs the family of all alternative methods of pain relief available in the hospital setting.

ANS: B The assessment of the woman, her fetus, and her labor is a joint effort of the nurse and the primary health care providers, who consult with the woman about their findings and recommendations. The needs of each woman are different and many factors must be considered before a decision is made whether pharmacologic methods, nonpharmacologic methods, or a combination of the two will be used to manage labor pain.

30. To assist the woman after delivery of the infant, the nurse knows that the blood patch is used after spinal anesthesia to relieve: a. hypotension. b. headache. c. neonatal respiratory depression. d. loss of movement.

ANS: B The subarachnoid block may cause a postspinal headache resulting from loss of cerebrospinal fluid from the puncture in the dura. When blood is injected into the epidural space in the area of the dural puncture, it forms a seal over the hole to stop leaking of cerebrospinal fluid. Hypotension is prevented by increasing fluid volume before the procedure. Neonatal respiratory depression is not an expected outcome with spinal anesthesia. Loss of movement is an expected outcome of spinal anesthesia.

8. If an opioid antagonist is administered to a laboring woman, she should be told that: a. her pain will decrease. b. her pain will return. c. she will feel less anxious. d. she will no longer feel the urge to push.

ANS: B The woman should be told that the pain that was relieved by the opioid analgesic will return with administration of the opioid antagonist. Opioid antagonists, such as Narcan, promptly reverse the central nervous system (CNS) depressant effects of opioids. In addition, the antagonist counters the effect of the stress-induced levels of endorphins. An opioid antagonist is especially valuable if labor is more rapid than expected and birth is anticipated when the opioid is at its peak effect.

28. The laboring woman who imagines her body opening to let the baby out is using a mental technique called: a. dissociation. b. effleurage. c. imagery. d. distraction.

ANS: C Imagery is a technique of visualizing images that will assist the woman in coping with labor. Dissociation helps the woman learn to relax all muscles except those that are working. Effleurage is self-massage. Distraction can be used in the early latent phase by having the woman engage in another activity.

4. A woman in labor has just received an epidural block. The most important nursing intervention is to: a. limit parenteral fluids. b. monitor the fetus for possible tachycardia. c. monitor the maternal blood pressure for possible hypotension. d. monitor the maternal pulse for possible bradycardia.

ANS: C The most important nursing intervention for a woman who has received an epidural block is to monitor the maternal blood pressure frequently for signs of hypotension. Intravenous fluids are increased for a woman receiving an epidural, to prevent hypotension. The nurse observes for signs of fetal bradycardia. The nurse monitors for signs of maternal tachycardia secondary to hypotension.

22. A woman in labor is breathing into a mouthpiece just before the start of her regular contractions. As she inhales, a valve opens, and gas is released. She continues to inhale the gas slowly and deeply until the contraction starts to subside. When the inhalation stops, the valve closes. This procedure is: a. not used much anymore. b. likely to be used in the second stage of labor but not in the first stage. c. an application of nitrous oxide. d. a prelude to cesarean birth.

ANS: C This is an application of nitrous oxide, which could be used in either the first or second stage of labor (or both) as part of the preparation for a vaginal birth. Nitrous oxide is self-administered and found to be very helpful.

29. The obstetric nurse is preparing the patient for an emergency cesarean birth, with no time to administer spinal anesthesia. The nurse is aware and prepared for the greatest risk of administering general anesthesia to the patient. This risk is: a. respiratory depression. b. uterine relaxation. c. inadequate muscle relaxation. d. aspiration of stomach contents.

ANS: D Aspiration of acidic gastric contents with possible airway obstruction is a potentially fatal complication of general anesthesia. Respirations can be altered during general anesthesia, and the anesthesiologist will take precautions to maintain proper oxygenation. Uterine relaxation can occur with some anesthesia; however, this can be monitored and prevented. Inadequate muscle relaxation can be improved with medication.

1. An 18-year-old pregnant woman, gravida 1, is admitted to the labor and birth unit with moderate contractions every 5 minutes that last 40 seconds. The woman states, "My contractions are so strong that I don't know what to do with myself." The nurse should: a. assess for fetal well-being. b. encourage the woman to lie on her side. c. disturb the woman as little as possible. d. recognize that pain is personalized for each individual.

ANS: D Each woman's pain during childbirth is unique and is influenced by a variety of physiologic, psychosocial, and environmental factors. A critical issue for the nurse is how support can make a difference in the pain of the woman during labor and birth. Assessing for fetal well-being includes no information that would indicate fetal distress or a logical reason to be overly concerned about the well-being of the fetus. The left lateral position is used to alleviate fetal distress, not maternal stress. The nurse has an obligation to provide physical, emotional, and psychosocial care and support to the laboring woman. This patient clearly needs support.

12. Which statement correctly describes the effects of various pain factors? a. Higher prostaglandin levels arising from dysmenorrhea can blunt the pain of childbirth. b. Upright positions in labor increase the pain factor because they cause greater fatigue. c. Women who move around trying different positions are experiencing more pain. d. Levels of pain-mitigating -endorphins are higher during a spontaneous, natural childbirth.

ANS: D Higher endorphin levels help women tolerate pain and reduce anxiety and irritability. Higher prostaglandin levels correspond to more severe labor pains. Upright positions in labor usually result in improved comfort and less pain. Moving freely to find more comfortable positions is important for reducing pain and muscle tension.

27. Which method of pain management is safest for a gravida 3 para 2 admitted at 8 cm cervical dilation? a. Epidural anesthesia b. Narcotics c. Spinal block d. Breathing and relaxation techniques

ANS: D Nonpharmacologic methods of pain management may be the best option for a woman in advanced labor. It is unlikely that enough time remains to administer epidural or spinal anesthesia. A narcotic given at this time may reach its peak about the time of birth and result in respiratory depression in the newborn.

6. A woman in the active phase of the first stage of labor is using a shallow pattern of breathing, which is about twice the normal adult breathing rate. She starts to complain about feeling light-headed and dizzy and states that her fingers are tingling. The nurse should: a. notify the woman's physician. b. tell the woman to slow the pace of her breathing. c. administer oxygen via a mask or nasal cannula. d. help her breathe into a paper bag.

ANS: D This woman is experiencing the side effects of hyperventilation, which include the symptoms of lightheadedness, dizziness, tingling of the fingers, or circumoral numbness. Having the woman breathe into a paper bag held tightly around her mouth and nose may eliminate respiratory alkalosis. This enables her to rebreathe carbon dioxide and replace the bicarbonate ion.

Nurses alert to signs of the onset of the second stage of labor can be certain that this stage has begun when: a. The woman has a sudden episode of vomiting. b. The nurse is unable to feel the cervix during a vaginal examination. c. Bloody show increases. d. The woman involuntarily bears down.

B (The only certain objective sign that the second stage has begun is the inability to feel the cervix because it is fully dilated and effaced. Vomiting, an increase in bloody show, and involuntary bearing down are only suggestions of second-stage labor.)

Concerning the third stage of labor, nurses should be aware that: a. The placenta eventually detaches itself from a flaccid uterus. b. An expectant or active approach to managing this stage of labor reduces the risk of complications. c. It is important that the dark, roughened maternal surface of the placenta appear before the shiny fetal surface. d. The major risk for women during the third stage is a rapid heart rate.

B (Active management facilitates placental separation and expulsion, thus reducing the risk of complications. The placenta cannot detach itself from a flaccid (relaxed) uterus. Which surface of the placenta comes out first is not clinically important. The major risk for women during the third stage of labor is postpartum hemorrhage.)

The nurse knows that the second stage of labor, the descent phase, has begun when: a. The amniotic membranes rupture. b. The cervix cannot be felt during a vaginal examination. c. The woman experiences a strong urge to bear down. d. The presenting part is below the ischial spines.

C (During the descent phase of the second stage of labor, the woman may experience an increase in the urge to bear down. Rupture of membranes has no significance in determining the stage of labor. The second stage of labor begins with full cervical dilation. Many women may have an urge to bear down when the presenting part is below the level of the ischial spines. This can occur during the first stage of labor, as early as 5-cm dilation.)

he nurse is examining a newly admitted patient who is 39 weeks pregnant and notes that the patient is in the transition phase of labor. Which symptoms does the nurse note to reach this conclusion? A. No evidence of uterine contractions (UCs) B. Mild uterine contractions (UCs) C. Strong uterine contractions (UCs) D. Moderate uterine contractions (UCs)

C (Regular and strong UCs may occur in the transition phase of labor. Absence of uterine contractions means that the labor has not started. Mild UCs can be observed during early labor. Mild to moderate UCs can be observed during the latent and active phases of labor.)

If a woman complains of back labor pain, the nurse could best suggest that she: a. Lie on her back for a while with her knees bent. b. Do less walking around. c. Take some deep, cleansing breaths. d. Lean over a birth ball with her knees on the floor.

D (The hands-and-knees position, with or without the aid of a birth ball, should help with the back pain. The supine position should be discouraged. Walking generally is encouraged.)

Leopold maneuvers would be an inappropriate method of assessment to determine: a. Gender of the fetus. b. Number of fetuses. c. Fetal lie and attitude. d. Degree of the presenting part's descent into the pelvis.

A (Leopold maneuvers help identify the number of fetuses, the fetal lie and attitude, and the degree of descent of the presenting part into the pelvis. The gender of the fetus is not a goal of the examination at this time.)

When assessing a multiparous woman who has just given birth to an 8-pound boy, the nurse notes that the woman's fundus is firm and has become globular in shape. A gush of dark red blood comes from her vagina. The nurse concludes that: a. The placenta has separated. b. A cervical tear occurred during the birth. c. The woman is beginning to hemorrhage. d. Clots have formed in the upper uterine segment.

A (Placental separation is indicated by a firmly contracting uterus, a change in the uterus from a discoid to a globular ovoid shape, a sudden gush of dark red blood from the introitus, an apparent lengthening of the umbilical cord, and a finding of vaginal fullness. Cervical tears that do not extend to the vagina result in minimal blood loss. Signs of hemorrhage are a boggy uterus, bright red vaginal bleeding, alterations in vital signs, pallor, lightheadedness, restlessness, decreased urinary output, and alteration in the level of consciousness. If clots have formed in the upper uterine segment, the nurse would expect to find the uterus boggy and displaced to the side.)

A means of controlling the birth of the fetal head with a vertex presentation is: a. The Ritgen maneuver. b. Fundal pressure. c. The lithotomy position. d. The De Lee apparatus.

A (The Ritgen maneuver extends the head during the actual birth and protects the perineum. Gentle, steady pressure against the fundus of the uterus facilitates vaginal birth. The lithotomy position has been commonly used in Western cultures, partly because it is convenient for the health care provider. The De Lee apparatus is used to suction fluid from the infant's mouth.)

The most critical nursing action in caring for the newborn immediately after birth is: a. Keeping the newborn's airway clear. b. Fostering parent-newborn attachment. c. Drying the newborn and wrapping the infant in a blanket. d. Administering eye drops and vitamin K.

A (The care given immediately after the birth focuses on assessing and stabilizing the newborn. Although fostering parent-infant attachment is an important task for the nurse, it is not the most critical nursing action in caring for the newborn immediately after birth. The nursing activities would be (in order of importance) to maintain a patent airway, support respiratory effort, and prevent cold stress by drying the newborn and covering the infant with a warmed blanket or placing the newborn under a radiant warmer. After the newborn has been stabilized, the nurse assesses the newborn's physical condition, weighs and measures the newborn, administers prophylactic eye ointment and a vitamin K injection, affixes an identification bracelet, wraps the newborn in warm blankets, and then gives the infant to the partner or mother when he or she is ready.)

As the United States and Canada continue to become more culturally diverse, it is increasingly important for the nursing staff to recognize a wide range of varying cultural beliefs and practices. Nurses need to develop respect for these culturally diverse practices and learn to incorporate these into a mutually agreed on plan of care. Although it is common practice in the United States for the father of the baby to be present at the birth, in many societies this is not the case. When implementing care, the nurse would anticipate that a woman from which country would have the father of the baby in attendance? a. Mexico b. China c. Iran d. India

A (A woman from Mexico may be stoic about discomfort until the second stage, at which time she will request pain relief. Fathers and female relatives are usually in attendance during the second stage of labor. The father of the baby is expected to provide encouragement, support, and reassurance that all will be well. Fathers are usually not present in China. The Iranian father will not be present. Female support persons and female care providers are preferred. For many, a male caregiver is unacceptable. The father is usually not present in India, but female relatives are usually present. Natural childbirth methods are preferred.)

What is an essential part of nursing care for the laboring woman? a. Helping the woman manage the pain b. Eliminating the pain associated with labor c. Sharing personal experiences regarding labor and delivery to decrease her anxiety d. Feeling comfortable with the predictable nature of intrapartum care

A (Helping a woman manage the pain is an essential part of nursing care because pain is an expected part of normal labor and cannot be fully relieved. Decreasing anxiety is important; however, managing pain is a top priority. The labor nurse should consistently deliver care based on the standard of care related to the maternity patient. Because of the unpredictable nature of labor, the nurse should always be alert for unanticipated events.)

After reviewing the laboratory reports of a pregnant patient at term, the primary health care provider (PHP) advised the nurse to administer intravenous (I.V.) fluids to the patient. What is the reason for giving such advice? A. Dehydration B. Hypertension C. Maternal hyperglycemic D. Preterm labor

A (I.V. fluids are administered to increase the amount of fluids and restore the electrolyte balance. As the patient is dehydrated, the PHP advises the nurse to administer I.V. fluids. Administration of I.V. fluids as a medical treatment for the prevention of preterm labor is not indicated unless medical management involves use of therapeutic protocols such as magnesium sulfate. As the patient is at term, preterm labor would not be a factor. Administering fluids may increase the venous pressure, thereby enhancing the blood pressure. Therefore I.V. fluids must not be administered if the patient has hypertension. Other prospective medical management should be initiated if maternal hypertension is noted. I.V. fluids should not be administered to hyperglycemic patients, but rather other prospective medical management should be initiated if maternal hyperglycemia is noted and deemed to be significant.)

The nurse is caring for a non-English-speaking pregnant patient. What nursing interventions would help explain the procedure of vaginal examination to the patient? A. Call a service for an interpreter. B. Try to communicate nonverbally. C. Limit the use of medical terminologies. D. Ask for the assistance of the hospital staff

A (It is important that the nurse explain the procedure to the patient. Because the patient does not speak English, it is advisable to call an interpreter. This helps the patient understand the test procedures without any confusion. Nonverbal communication is not useful in this case, because it may cause the patient to become confused. Explaining the medical examination procedure may include complex terms and words. Limiting those words may not help clarify to the patient who does not speak English. Finally, the patient may not feel comfortable in the presence of additional hospital staff.)

The nurse is caring for a pregnant patient. What interventions should the nurse follow to ensure proper hygiene in the patient? A. Clean the perineum of the patient frequently. B. Clean the patient's teeth with a warm wet cloth. C. Offer a warm washcloth to the patient for a face wash. D. Allow cool water to flow on the patient's back for 5 minutes.

A (The patient's perineum should be cleaned frequently to prevent the risk for infection. This helps maintain proper hygiene and provides comfort to the patient. The nurse can clean the patient's teeth with an ice-cold wet washcloth, which helps prevent a feeling of thirst and dryness of the mouth. Using a warm cloth may not be helpful. The patient is offered a cool cloth for wiping her face, which helps prevent diaphoresis. Warm water should be poured on the patient's back to provide relaxation and accelerate labor. Using a warm washcloth for a face wash and placing cool water on the patient's back will not help in providing comfort.)

When assessing a woman in the first stage of labor, the nurse recognizes that the most conclusive sign that uterine contractions are effective would be: a. Dilation of the cervix. b. Descent of the fetus. c. Rupture of the amniotic membranes. d. Increase in bloody show.

A (The vaginal examination reveals whether the woman is in true labor. Cervical change, especially dilation, in the presence of adequate labor indicates that the woman is in true labor. Descent of the fetus, or engagement, may occur before labor. Rupture of membranes may occur with or without the presence of labor. Bloody show may indicate slow, progressive cervical change (e.g., effacement) in both true and false labor)

A pregnant woman is in her third trimester. She asks the nurse to explain how she can tell true labor from false labor. The nurse would explain that "true" labor contractions: a. Increase with activity such as ambulation. b. Decrease with activity. c. Are always accompanied by the rupture of the bag of waters. d. Alternate between a regular and an irregular pattern.

A (True labor contractions become more intense with walking. False labor contractions often stop with walking or position changes. Rupture of membranes may occur before or during labor. True labor contractions are regular.)

When managing the care of a woman in the second stage of labor, the nurse uses various measures to enhance the progress of fetal descent. These measures include: a. Encouraging the woman to try various upright positions, including squatting and standing. b. Telling the woman to start pushing as soon as her cervix is fully dilated. c. Continuing an epidural anesthetic so pain is reduced and the woman can relax. d. Coaching the woman to use sustained, 10- to 15-second, closed-glottis bearing-down efforts with each contraction.

A (Upright positions and squatting both may enhance the progress of fetal descent. Many factors dictate when a woman will begin pushing. Complete cervical dilation is necessary, but it is only one factor. If the fetal head is still in a higher pelvic station, the physician or midwife may allow the woman to "labor down" (allowing more time for fetal descent, thereby reducing the amount of pushing needed) if the woman is able. The epidural may mask the sensations and muscle control needed for the woman to push effectively. Closed glottic breathing may trigger the Valsalva maneuver, which increases intrathoracic and cardiovascular pressures, reducing cardiac output and inhibiting perfusion of the uterus and placenta. In addition, holding the breath for longer than 5 to 7 seconds diminishes the perfusion of oxygen across the placenta and results in fetal hypoxia.)

With regard to a woman's intake and output during labor, nurses should be aware that: a. The tradition of restricting the laboring woman to clear liquids and ice chips is being challenged because regional anesthesia is used more often than general anesthesia. b. Intravenous (IV) fluids usually are necessary to ensure that the laboring woman stays hydrated. c. Routine use of an enema empties the rectum and is very helpful for producing a clean, clear delivery. d. When a nulliparous woman experiences the urge to defecate, it often means birth will follow quickly.

A (Women are awake with regional anesthesia and are able to protect their own airway, which reduces the worry over aspiration. Routine IV fluids during labor are unlikely to be beneficial and may be harmful. Routine use of an enema is at best ineffective and may be harmful. A multiparous woman may feel the urge to defecate and it may mean birth will follow quickly, but not for a first-timer.)

For the labor nurse, care of the expectant mother begins with which situations? Select all that apply. A. The onset of progressive, regular contractions B. The bloody, or pink, show C. The spontaneous rupture of membranes D. Formulation of the woman's plan of care for labor E. Moderately painful contractions

A, B, C (Labor care begins with the onset of progressive, regular contractions. The woman and the nurse can formulate their plan of care before labor or during treatment. Labor care begins when the blood-tinged mucoid vaginal discharge appears. The woman and the nurse can formulate their plan of care before labor or during treatment. Labor care begins when amniotic fluid is discharged from the vagina. The woman and the nurse can formulate their plan of care before labor or during treatment. Labor care begins when progressive, regular contractions begin, the blood-tinged mucoid vaginal discharge appears, or fluid is discharged from the vagina. The woman and the nurse can formulate their plan of care before labor or during treatment. Pain is subjective. The onset of progressive, regular contractions signals the beginning of labor, not the intensity of the pain.)

The nurse finds that the pregnant patient has impaired urinary elimination. Which interventions should be performed by the nurse to relieve the patient's problem? Select all that apply. A. Encourage the patient to urinate every 2 hours. B. Catheterize the patient immediately for voiding. C. Palpate patient's bladder superior to symphysis. D. Ask the patient to place the hand in running water. E. Provide effleurage massage to the patient frequently.

A, C, D (Impaired urinary elimination occurs as a result of sensory impairment caused by the labor process. Therefore the nurse has to perform interventions that help in emptying the patient's bladder every 2 hours. The nurse should encourage the patient to void every 2 hours to avoid bladder distention. The nurse can use running water to stimulate voiding by asking the patient to keep her hands in the running water. The nurse should palpate the patient's bladder on a frequent basis to detect the inability to void. The nurse should not catheterize the patient immediately for voiding, because it may result in trauma to the bladder. Effleurage helps in reducing pain but does not help stimulate voiding in the patient.)

Under which circumstances should a vaginal examination be performed by the nurse? Select all that apply. A. An admission to the hospital at the start of labor B. When accelerations of the fetal heart rate (FHR) are noted C. On maternal perception of perineal pressure or the urge to bear down D. When membranes rupture E. When bright, red bleeding is observed

A, C, D (Vaginal examinations should be performed when the woman is admitted to the hospital or birthing center at the start of labor. When the woman perceives perineal pressure or the urge to bear down is an appropriate time to perform a vaginal examination. After rupture of membranes (ROM), a vaginal examination should be performed. The nurse must be aware that there is an increased risk of prolapsed cord immediately after ROM. An accelerated FHR is a positive sign; variable decelerations, however, merit a vaginal examination. Examinations are never done by the nurse if vaginal bleeding is present because the bleeding could be a sign of placenta previa and a vaginal examination could result in further separation of the low-lying placenta.)

The nurse assesses a pregnant patient and reports to the primary health care provider (PHP) that the patient is in the second stage of labor. Which of the patient's signs enabled the nurse to give such a report to the PHP? Select all that apply. A. Urge to defecate B. Cheeks appear to be flushing C. Cervical dilation of 10 cm D. Brownish discharge of mucus from the vagina E. Premature urge to bear down

A, C, E (After an assessment, the nurse reports to the PHP that a pregnant patient is in the second stage of labor because the patient has a cervical dilation of 10 cm (fully dilated). The patient has a premature urge to bear down and an urge to defecate. The patient may have flushed cheeks in the active phase of first stage of labor, but it is not a sign of second stage of labor. Brownish discharge of mucus is a sign of latent phase of first stage of labor, but does not appear in the second stage of labor.)

A patient has been admitted to the labor room. What are the measures to be taken by the nurse to support the partner of the patient? Select all that apply. A. Offer snacks and fluids to the partner as required. B. Do not discuss the psychological change in the patient. C. Demonstrate the performance of the comfort measures. D. Guide the partner to make decisions about his involvement. E. Relieve the person occasionally from the job of supporting the patient.

A, C, E (Any comfort measures useful for the patient should be demonstrated to the patient's partner. The patient's partner may be reminded to take food. The nurse can also offer snacks and fluids to the partner. The nurse can offer to relieve him of the duty of supporting and encouraging the patient in order to get proper rest. The decision regarding the involvement of the partner in the process of labor should be left to the couple. The nurse should respect their decision. The nurse should tell the partner about the changes that may take place in the patient's behavior during labor and childbirth.)

1. Maternal hypotension is a potential side effect of regional anesthesia and analgesia. What nursing interventions could you use to raise the patient's blood pressure? (Select all that apply.) a. Place the woman in a supine position. b. Place the woman in a lateral position. c. Increase intravenous (IV) fluids. d. Administer oxygen. e. Perform a vaginal examination.

ANS: B, C, D Nursing interventions for maternal hypotension arising from analgesia or anesthesia include turning the woman to a lateral position, increasing IV fluids, administering oxygen via face mask, elevating the woman's legs, notifying the physician, administering an IV vasopressor, and monitoring the maternal and fetal status at least every 5 minutes until these are stable. Placing the patient in a supine position would cause venous compression, thereby limiting blood flow to and oxygenation of the placenta and fetus. A sterile vaginal examination has no bearing on maternal blood pressure.

A woman who is gravida 3 para 2 enters the intrapartum unit. The most important nursing assessments are: a. Contraction pattern, amount of discomfort, and pregnancy history. b. Fetal heart rate, maternal vital signs, and the woman's nearness to birth. c. Identification of ruptured membranes, the woman's gravida and para, and her support person. d. Last food intake, when labor began, and cultural practices the couple desires.

B (All options describe relevant intrapartum nursing assessments; however, this focused assessment has priority. If the maternal and fetal conditions are normal and birth is not imminent, other assessments can be performed in an unhurried manner. This includes: gravida, para, support person, pregnancy history, pain assessment, last food intake, and cultural practices.)

What is an expected characteristic of amniotic fluid? a. Deep yellow color b. Pale, straw color with small white particles c. Acidic result on a Nitrazine test d. Absence of ferning

B (Amniotic fluid normally is a pale, straw-colored fluid that may contain white flecks of vernix. Yellow-stained fluid may indicate fetal hypoxia up to 36 hours before rupture of membranes, fetal hemolytic disease, or intrauterine infection. Amniotic fluid produces an alkaline result on a Nitrazine test. The presence of ferning is a positive indication of amniotic fluid.)

Under which circumstance would it be unnecessary for the nurse to perform a vaginal examination? a. An admission to the hospital at the start of labor b. When accelerations of the fetal heart rate (FHR) are noted c. On maternal perception of perineal pressure or the urge to bear down d. When membranes rupture

B (An accelerated FHR is a positive sign; however, variable decelerations merit a vaginal examination. Vaginal examinations should be performed when the woman is admitted, when she perceives perineal pressure or the urge to bear down, when her membranes rupture, when a significant change in her uterine activity has occurred, or when variable decelerations of the FHR are noted.)

A woman who is 39 weeks pregnant expresses fear about her impending labor and how she will manage. The nurse's best response is: A. "Don't worry about it. You'll do fine." B. "It's normal to be anxious about labor. Let's discuss what makes you afraid." C. "Labor is scary to think about, but the actual experience isn't." D. "You may have an epidural. You won't feel anything."

B (Discussing the woman's fears allows her to share her concerns with the nurse and is a therapeutic communication tool. Telling the woman not to worry negates her fears and is not therapeutic. Telling the woman that labor is not scary negates her fears and offers a false sense of security. A number of criteria must be met for use of an epidural. Furthermore, many women still experience the feeling of pressure with an epidural. Test-Taking Tip: Practicing a few relaxation techniques may prove helpful on the day of an examination. Relaxation techniques such as deep breathing, imagery, head rolling, shoulder shrugging, rotating and stretching of the neck, leg lifts, and heel lifts with feet flat on the floor can effectively reduce tension while causing little or no distraction to those around you. It is recommended that you practice one or two of these techniques intermittently to avoid becoming tense. The more anxious and tense you become, the longer it will take you to relax.)

Which test is performed to determine if membranes are ruptured? A. Urine analysis B. Fern test C. Leopold maneuvers D. Artificial rupture of membranes (AROM

B (In many instances, a sterile speculum examination and a Nitrazine (pH) and fern test are performed to confirm that fluid seepage is indeed amniotic fluid. A urine analysis should be performed on admission to labor and delivery. This test is used to identify the presence of glucose and protein. The nurse performs Leopold maneuvers to identify fetal lie, presenting part, and attitude. AROM is the procedure of artificially rupturing membranes, usually with a device known as an amnihook. Test-Taking Tip: Be aware that information from previously asked questions may help you respond to other examination questions.)

The primary difference between the labor of a nullipara and that of a multipara is the: a. Amount of cervical dilation. b. Total duration of labor. c. Level of pain experienced. d. Sequence of labor mechanisms.

B (Multiparas usually labor more quickly than nulliparas, thus making the total duration of their labor shorter. Cervical dilation is the same for all labors. The level of pain is individual to the woman, not to the number of labors she has experienced. The sequence of labor mechanisms remains the same with all labors.)

The nurse expects to administer an oxytocic (e.g., Pitocin, Methergine) to a woman after expulsion of her placenta to: a. Relieve pain. b. Stimulate uterine contraction. c. Prevent infection. d. Facilitate rest and relaxation.

B (Oxytocics stimulate uterine contractions, which reduce blood loss after the third stage of labor. Oxytocics are not used to treat pain or prevent infection. They cause the uterus to contract, which reduces blood loss. Oxytocics do not facilitate rest and relaxation.)

The nurse is caring for a Hispanic patient who has given birth to a baby. When does the nurse expect the patient to start breastfeeding? A. First hour after birth B. When the milk comes C. When the infant cries D. After the patient has rested

B (Some patients often wish to breastfeed after the ejection of the milk. The patient cannot be given instruction to breastfeed 1 hour after birth. The patient may require rest, but breastfeeding should be encouraged only after the milk ejection. Some patients prefer to breastfeed during the infant's reactive state, but patients of a Hispanic background may not choose to do this, as it may not fall within their cultural belief system. The nurse should always respect the cultural beliefs of the patient.)

After an emergency birth, the nurse encourages the woman to breastfeed her newborn. The primary purpose of this activity is to: a. Facilitate maternal-newborn interaction. b. Stimulate the uterus to contract. c. Prevent neonatal hypoglycemia. d. Initiate the lactation cycle.

B (Stimulation of the nipples through breastfeeding or manual stimulation causes the release of oxytocin and prevents maternal hemorrhage. Breastfeeding facilitates maternal-newborn interaction, but it is not the primary reason a woman is encouraged to breastfeed after an emergency birth. The primary intervention for preventing neonatal hypoglycemia is thermoregulation. Cold stress can result in hypoglycemia. The woman is encouraged to breastfeed after an emergency birth to stimulate the release of oxytocin, which prevents hemorrhage. Breastfeeding is encouraged to initiate the lactation cycle, but it is not the primary reason for this activity after an emergency birth.)

Nurses can help their clients by keeping them informed about the distinctive stages of labor. What description of the phases of the first stage of labor is accurate? a. Latent: Mild, regular contractions; no dilation; bloody show; duration of 2 to 4 hours b. Active: Moderate, regular contractions; 4- to 7-cm dilation; duration of 3 to 6 hours c. Lull: No contractions; dilation stable; duration of 20 to 60 minutes d. Transition: Very strong but irregular contractions; 8- to 10-cm dilation; duration of 1 to 2 hours

B (The active phase is characterized by moderate, regular contractions; 4- to 7-cm dilation; and a duration of 3 to 6 hours. The latent phase is characterized by mild-to-moderate, irregular contractions; dilation up to 3 cm; brownish-to-pale pink mucus, and a duration of 6 to 8 hours. No official "lull" phase exists in the first stage. The transition phase is characterized by strong to very strong, regular contractions; 8- to 10-cm dilation; and a duration of 20 to 40 minutes.)

The nurse observes that a pregnant patient has a blood glucose level of 180 mg/dL in early labor. Which medication order does the nurse expect to receive from the primary health care provider (PHP)? A. Lidocaine (Nervocaine) to the patient B. Ringer's lactate solution to the patient C. Hydromorphone (Dilaudid) to the patient D. Intravenous (IV) solution containing a small amount of dextrose

B (The blood glucose level of 180 mg/dL indicates that the patient has high blood glucose levels. Therefore the patient has to be administered an electrolyte solution without glucose to prevent the risk of fetal hyperglycemia and hyperinsulinism. Hence, the nurse would expect the PHP to prescribe Ringer's lactate solution to the patient, as it does not increase blood sugar levels. Lidocaine (Nervocaine) is an anesthetic preparation, which may be given during emergency. Hydromorphone (Dilaudid) is an opioid preparation and is not used in treating blood glucose levels in the body. IV solution containing a small amount of dextrose is administered to increase the fatty acid metabolism when the patient has ketosis. It is not useful to treat hyperglycemia.)

Through vaginal examination the nurse determines that a woman is 4 cm dilated, and the external fetal monitor shows uterine contractions every 3.5 to 4 minutes. The nurse would report this as: a. First stage, latent phase. b. First stage, active phase. c. First stage, transition phase. d. Second stage, latent phase.

B (The first stage, active phase of maternal progress indicates that the woman is in the active phase of the first stage of labor. During the latent phase of the first stage of labor, the expected maternal progress would be 0 to 3 cm dilation with contractions every 5 to 30 minutes. During the transition phase of the first stage of labor, the expected maternal progress is 8 to 10 cm dilation with contractions every 2 to 3 minutes. During the latent phase of the second stage of labor, the woman is completely dilated and experiences a restful period of "laboring down.")

The nurse is caring for a pregnant patient during labor. What should the nurse do immediately after the child's birth? A. Ask the mother to hold the infant. B. Dry the infant and place in warm blanket. C. Record the Apgar scores after 30 minutes. D. Cut the umbilical cord 3.5 cm above the clamp.

B (The infant should be dried to prevent cold stress due to rapid loss of heat and then covered with a warm blanket. The Apgar score is to be recorded at 1 and 5 minutes after the birth of the infant. Recording it after 30 minutes may lead to failure in assessing the fetal signs. The cord should be cut at 2.5 cm above the placement of the clamp. A newborn may be very slippery to hold, and the mother may not be able to hold the baby due to fatigue. The infant can be given to the mother only after complete drying.)

A patient whose cervix is dilated to 5 cm is considered to be in which phase of labor? a. Latent phase b. Active phase c. Second stage d. Third stage

B (The latent phase is from the beginning of true labor until 3 cm of cervical dilation. The active phase of labor is characterized by cervical dilation of 4 to 7 cm. The second stage of labor begins when the cervix is completely dilated until the birth of the baby. The third stage of labor is from the birth of the baby until the expulsion of the placenta. This patient is in the active phase of labor.)

For women who have a history of sexual abuse, a number of traumatic memories may be triggered during labor. The woman may fight the labor process and react with pain or anger. Alternately, she may become a passive player and emotionally absent herself from the process. The nurse is in a unique position of being able to assist the client to associate the sensations of labor with the process of childbirth and not the past abuse. The nurse can implement a number of care measures to help the client view the childbirth experience in a positive manner. Which intervention would be key for the nurse to use while providing care? a. Telling the client to relax and that it won't hurt much b. Limiting the number of procedures that invade her body c. Reassuring the client that as the nurse you know what is best d. Allowing unlimited care providers to be with the client

B (The number of invasive procedures such as vaginal examinations, internal monitoring, and intravenous therapy should be limited as much as possible. The nurse should always avoid words and phrases that may result in the client's recalling the phrases of her abuser (e.g., "Relax, this won't hurt" or "Just open your legs.") The woman's sense of control should be maintained at all times. The nurse should explain procedures at the client's pace and wait for permission to proceed. Protecting the client's environment by providing privacy and limiting the number of staff who observe the client will help to make her feel safe.)

After performing Leopold maneuvers, the nurse finds that the fetus of a pregnant patient is in occiput posterior position. Which suitable action should the nurse employ while caring for the patient? A. Help the patient to lie in supine position on the bed. B. Encourage the patient to sit in hands-and-knees position. C. Place a pillow under the patient's hip when lying in supine position. D. Ask the patient to lie in lateral position on the opposite side of the fetal spine.

B (The nurse should place the patient in a position that helps the rotation of the fetal occiput from a posterior to an anterior position. Therefore the nurse should encourage the patient to sit in hands-and-knees position, as it increases the pelvic diameter, allowing the head to rotate toward anterior position. The patient should not lie in supine position, as it may cause postural hypotension. Placing a pillow under the patient's hip when lying in supine position helps prevent supine hypotensive syndrome, but does not help in delivering the baby. The nurse should not ask the patient to lie in lateral position on the opposite side of the fetal spine, as it increases counter pressure on the back. Instead, lying in lateral position on the same side of the fetal spine will help the fetus rotate toward the posterior, as the gravity pulls the fetal back forward.)

The nurse is assessing a pregnant patient in the last week of gestation. The nurse observes that the patient has flushed cheeks, uterine contractions (UCs) of 65 seconds with a frequency of 4 minutes, and pink to bloody mucus. What stage of labor should the nurse infer that the patient is in based on these observations? A. Latent phase B. Active phase C. Transition phase D. Active pushing phase

B (The patient has flushed cheeks, UCs of 65 seconds with a frequency of 4 minutes, and pink to bloody mucus. These symptoms are observed during the active phase of labor. The symptoms of the patient do not correlate with the latent, transition, or active pushing phases (second stage) of labor. In the latent phase of labor, the UCs are 30 to 45 seconds with a frequency of 5 to 30 minutes, and the mucus is pale pink. In the transition phase, the UCs are 45 to 90 seconds with a frequency of 2 to 3 minutes, and the mucus appears bloody. In the active pushing phase of the second stage of labor, the UCs are 90 seconds with a frequency of 2 to 2.5 minutes.)

A patient in labor exhibits flushed cheeks. The nurse records the uterine contractions in the patient as being 3 to 5 minutes apart and lasting for about 1 minute. What nursing intervention is most effective to assess the patient's status during this phase of labor? A. Check blood pressure every 2 hours. B. Note patient's appearance and mood every 15 minutes. C. Assess the patient's temperature every 2 hours until membranes rupture. D. Document

B (The patient is experiencing uterine contractions that are 3 to 5 minutes apart and last for about 60 seconds (1 minute). The patient also exhibits flushed cheeks. These findings indicate that the patient is in the active phase of the first stage of labor. The nursing assessment in the active stage of labor is to check the patient's appearance and mood every 15 minutes, or 4 times in an hour. The patient's mood and energy levels fluctuate, and therefore the nurse should constantly assess them to ensure effective patient care. The patient's blood pressure should be assessed every 30 minutes. The nurse should assess the patient's body temperature every 4 hours until membrane rupture and thereafter every 2 hours. Test-Taking Tip: You have at least a 25% chance of selecting the correct response in multiple-choice items. If you are uncertain about a question, eliminate the choices that you believe are wrong and then call on your knowledge, skills, and abilities to choose from the remaining responses.)

Which patients are more susceptible to soft-tissue damage with vaginal deliveries? Select all that apply. A. Multiparous patients B. Nulliparous patients C. Patients needing forceps delivery D. Patients with fetal vertex presentation E. Patients with fetal breech presentatio

B, C, E (A nulliparous patient has rigid perineal tissue making it susceptible to injury. Fetal breech presentation exerts undue pressure on the tissues, increasing the risk of injuries. Forceps delivery also increases the risk of injury due to undue stretch of the perineum. Multiparous patients have stretchable perineal tissues, which are less likely to get injured during childbirth. Fetal vertex presentation causes the least amount of tissue damage.)

The nurse is caring for a Southeast Asian patient who gave birth to a child. What interventions can the nurse perform to promote bonding between the newborn and the family? Select all that apply. A. Placing the hand on the infant's head B. Encouraging the sibling to hold the baby C. Explaining the molding of the infant's head D. Praising the infant's appearance and health E. Explaining the dusky appearance of the infant

B, C, E (The parents may be worried about the newborn's dusky appearance. Therefore the nurse should properly explain to the parents that the baby may initially appear dusky. The color may become normal once the circulation is established. Siblings may be encouraged to hold the newborn to promote bonding between them. The infant's head is molded due to the narrowness of the birth canal and the pelvic structures. This is to be explained to the parents. Southeast Asian patients consider the head to be the sacred part of the human body and should not be touched. Hence, the nurse should avoid placing hand on the infant's head. The Southeast Asian population considers any praise of the infant as harmful, as they believe the jealous spirits will take away the baby.)

It is paramount for the obstetric nurse to understand the regulatory procedures and criteria for admitting a woman to the hospital labor unit. Which guideline is an important legal requirement of maternity care? a. The patient is not considered to be in true labor (according to the Emergency Medical Treatment and Active Labor Act [EMTALA]) until a qualified health care provider says she is. b. The woman can have only her male partner or predesignated "doula" with her at assessment. c. The patient's weight gain is calculated to determine whether she is at greater risk for cephalopelvic disproportion (CPD) and cesarean birth. d. The nurse may exchange information about the patient with family members.

C (According to EMTALA, a woman is entitled to active labor care and is presumed to be in "true" labor until a qualified health care provider certifies otherwise. A woman can have anyone she wishes present for her support. The risk for CPD is especially great for petite women or those who have gained 16 kg or more. All patients should have their weight and BMI calculated on admission. This is part of standard nursing care on a maternity unit and not a regulatory concern. According to the Health Insurance Portability and Accountability Act (HIPAA), the patient must give consent for others to receive any information related to her condition.)

Which description of the phases of the second stage of labor is accurate? a. Latent phase: Feeling sleepy, fetal station 2+ to 4+, duration 30 to 45 minutes b. Active phase: Overwhelmingly strong contractions, Ferguson reflux activated, duration 5 to 15 minutes c. Descent phase: Significant increase in contractions, Ferguson reflux activated, average duration varied d. Transitional phase: Woman "laboring down," fetal station 0, duration 15 minutes

C (The descent phase begins with a significant increase in contractions; the Ferguson reflex is activated, and the duration varies, depending on a number of factors. The latent phase is the lull, or "laboring down," period at the beginning of the second stage. It lasts 10 to 30 minutes on average. The second stage of labor has no active phase. The transition phase is the final phase in the second stage of labor; contractions are strong and painful.)

Which action is correct when palpation is used to assess the characteristics and pattern of uterine contractions? a. Place the hand on the abdomen below the umbilicus and palpate uterine tone with the fingertips. b. Determine the frequency by timing from the end of one contraction to the end of the next contraction. c. Evaluate the intensity by pressing the fingertips into the uterine fundus. d. Assess uterine contractions every 30 minutes throughout the first stage of labor.

C (The nurse or primary care provider may assess uterine activity by palpating the fundal section of the uterus using the fingertips. Many women may experience labor pain in the lower segment of the uterus that may be unrelated to the firmness of the contraction detectable in the uterine fundus. The frequency of uterine contractions is determined by palpating from the beginning of one contraction to the beginning of the next contraction. Assessment of uterine activity is performed in intervals based on the stage of labor. As labor progresses this assessment is performed more frequently.)

The primary health care provider (PHP) advised the nurse to assess the maternal temperature and vaginal discharge of a pregnant patient every 2 hours. What is the reason behind this advice? A. To evaluate fetal status B. To know the onset of labor C. To assess for potential risk for infection D. To prevent fetal hypertension

C (When the membranes rupture, there is a possible risk of infection, as the microorganisms can ascend form the vagina to the uterus. Ruptured membranes can be assessed by monitoring the body temperature and vaginal discharge every 2 hours. The assessment is not used for knowing the onset of labor because it does not indicate the progress of labor. The fetal status is not known by the assessment of the temperature and vaginal show; it may be known by another procedure called Leopold maneuvers. This measure is not done to prevent fetal hypertension, because the maternal body temperature and vaginal discharge does not indicate fetal blood pressure.)

A patient sustained a first-degree laceration during childbirth. What physical finding should the nurse infer from this? The laceration: A. Also involves the anterior rectal wall. B. Continues through the anal sphincter muscle. C. Extends through muscles of the perineal body. D. Extends through the skin and structures superficial to muscles

D (A first-degree laceration extends through the skin and structures superficial to muscles. A second-degree laceration extends through muscles of the perineal body. A third-degree laceration continues through the anal sphincter muscle. A fourth-degree laceration involves the anterior rectal wall.)

When a nulliparous woman telephones the hospital to report that she is in labor, the nurse initially should: a. Tell the woman to stay home until her membranes rupture. b. Emphasize that food and fluid intake should stop. c. Arrange for the woman to come to the hospital for labor evaluation. d. Ask the woman to describe why she believes she is in labor.

D (Assessment begins at the first contact with the woman, whether by telephone or in person. By asking the woman to describe her signs and symptoms, the nurse can begin the assessment and gather data. The amniotic membranes may or may not spontaneously rupture during labor. The client may be instructed to stay home until the uterine contractions become strong and regular. The nurse may want to discuss the appropriate oral intake for early labor such as light foods or clear liquids, depending on the preference of the client or her primary health care provider. Before instructing the woman to come to the hospital, the nurse should initiate the assessment during the telephone interview.)

A pregnant couple has formulated a birth plan and is reviewing it with the nurse at an expectant parent's class. Which aspect of their birth plan is considered unrealistic and requires further discussion with the nurse? A. "My husband and I have agreed that my sister will be my coach because he becomes anxious with regard to medical procedures and blood. He will be nearby and check on me every so often to make sure everything is OK." B. "We plan to use the techniques taught in the Lamaze classes to reduce the pain experienced during labor." C. "We want the labor and birth to take place in a birthing room. My husband will come in the minute the baby is born." D. "We do not want the fetal monitor used during labor because it will interfere with movement and doing effleurage."

D (Because monitoring is essential to assess fetal well-being, it is not a factor that can be determined by the couple. The nurse should fully explain its importance. The option for intermittent electronic monitoring could be explored if this is a low-risk pregnancy and as long as labor is progressing normally. Having the woman's sister as her coach, using Lamaze techniques to reduce pain, and using a birthing room are realistic plans for the birth.)

The nurse palpates the abdomen of a pregnant patient and reports that the fetus lies in longitudinal position with cephalic presentation. Which observation enabled the nurse to report about the fetal position? A. The presenting part has deeply descended in the pelvis. B. The cephalic prominence is on the same side as the back. C. The head is presenting to the true pelvis and is not engaged. D. The head feels round, firm, freely movable, and palpable by ballottement

D (Leopold maneuvers (abdominal palpation) help identify the degree of descent into the pelvis of the presenting part in a pregnant patient. The head feels round, firm, freely movable, and palpable by ballottement when the fetus has a cephalic or breech presentation. Based on the descent of the presenting part, it may be difficult to infer the fetal position, as the presenting part can be head or buttock. The cephalic prominence on the same side as the back shows that the fetal head is extended and the face is the presenting part. This maneuver is not related to identification of fetal position. If the head is presenting to the true pelvis and is not engaged, then it determines the attitude of fetal head whether flexed or extended. It does not indicate the fetal position.)

During an assessment, the nurse is instructed to determine the position of the fetal head in a pregnant patient. What should the nurse do to determine whether the fetal head is flexed or extended? A. Palpate the fetal head with the palmar surface of the fingertips of the right hand. B. Identify the fetal part that occupies the fundus in the uterus of the pregnant patient. C. Palpate the smooth convex contour of the fetal back using the palmar surface of one hand. D. Grasp the lower pole of the uterus between the thumb and fingers, pressing in slightly.

D (Leopold maneuvers (abdominal palpation) help identify the degree of descent into the pelvis of the presenting part in a pregnant patient. Therefore the nurse should grasp the lower pole of the uterus between the thumb and fingers, pressing in slightly in order to determine whether the fetal head is flexed or extended. Identifying the fetal part that occupies the fundus of the patient helps to identify the fetal position. The fetal head is palpated with the palmar surface of the fingertips using both hands, but not with only the right hand to determine the cephalic prominence. Palpation of the smooth convex contour of the fetal back and irregularities using the palmar surface of one hand is not used to determine the attitude of the fetal head. This maneuver helps identify the feet, hands, and elbows of the fetus.)

The student nurse finds that the patient who is in labor has sweat on the upper lip, is shivering in the extremities, and is vomiting. What would the student nurse interpret from these observations? The patient has symptoms of: A. Postural hypotension. B. Respiratory depression. C. Onset of the first stage of labor. D. Onset of the second stage of labor.

D (Sudden appearances of sweat on the upper lip, shaking of the extremities, and vomiting indicate the onset of the second stage of labor. Irregular and mild to moderate uterine contractions (UCs) indicate the onset of the latent phase of the first stage labor. Postural hypotension is characterized by a sudden fall in the blood pressure while changing the position. Respiratory depression is characterized by a decreased rate of respiration.)

The patient reports severe lower back pain during labor. Which position does the nurse plan for the patient during childbirth? A. Lateral position B. Upright position C. Semirecumbent position D. Hands-and-knees position

D (The hands-and-knees position is suitable for patients with back pain and for patients experiencing back labor, because it reduces stress on the back. The lateral position can be used when the patient is receiving a back rub, but this position does not offer relief from back pain. An upright position may not have a significant effect on back pain. Therefore this position is not planned for childbirth. The semirecumbent position does not support the back, so back pain may not be relieved.)

The nurse who performs vaginal examinations to assess a woman's progress in labor should: a. Perform an examination at least once every hour during the active phase of labor. b. Perform the examination with the woman in the supine position. c. Wear two clean gloves for each examination. d. Discuss the findings with the woman and her partner.

D (The nurse should discuss the findings of the vaginal examination with the woman and her partner and report them to the primary care provider. A vaginal examination should be performed only when indicated by the status of the woman and her fetus. The woman should be positioned to avoid supine hypotension. The examiner should wear a sterile glove while performing a vaginal examination for a laboring woman.)

A multiparous woman has been in labor for 8 hours. Her membranes have just ruptured. The nurse's initial response would be to: a. Prepare the woman for imminent birth. b. Notify the woman's primary health care provider. c. Document the characteristics of the fluid. d. Assess the fetal heart rate and pattern.

D (The umbilical cord may prolapse when the membranes rupture. The fetal heart rate and pattern should be monitored closely for several minutes immediately after ROM to ascertain fetal well-being, and the findings should be documented. Rupture of membranes (ROM) may increase the intensity and frequency of the uterine contractions, but it does not indicate that birth is imminent. The nurse may notify the primary care provider after ROM occurs and fetal well-being and the response to ROM have been assessed. The nurse's priority is to assess fetal well-being. The nurse should document the characteristics of the amniotic fluid, but the initial response is to assess fetal well-being and the response to ROM.)


Kaugnay na mga set ng pag-aaral

Chapter 10, Leading, Managing and Care Delegating

View Set

#15 Quantitative Comparison - Algebra and Applied Mathematics

View Set

Chapter 26 AP EURO Example Questions (UPDATED!!!!)

View Set